RD MSK formatted Flashcards

1
Q
  1. A patient with a known Charcot foot has an ulcer with a red swollen foot, the clinicians ask your advice to identify osteomyelitis at 7-10 days after the onset of swelling?
    a. MRI
    b. bone scan
    c. PET
    d. Xray
    e. CT
A

ANSWER a. MRI= T, but often difficulta. MRI= T, but often difficult

  1. A patient with a known Charcot foot has an ulcer with a red swollen foot, the clinicians ask your advice to identify osteomyelitis at 7-10 days after the onset of swelling (SK)?
    a. MRI= T, but often difficulta. MRI= T, but often difficult
    b. Bone scan= F = Charcot joint will likely be positive whether infected or not (best NM test is combined leucocyte and sulfur colloid marrow subtraction study)
    c. PET= F = see below
    d. Xray= F = would do, but low diagnostic yield
    e. CT= FACR guidelines state do MRI + x-rays (both rate 9, N/M rates

4)StatDx
• Radiograph is appropriately the first-line test; relatively insensitive; MR is gold standard
• Even MR may be nonspecific for osteomyelitis in the presence of Charcot joint changes
• Recent meta-analysis suggests high accuracy of FDG PET to diagnose chronic osteomyelitis (but see below)J Nucl Med July 1, 2011 vol. 52 no. 7 1012-1019
• 18F-FDG PET/CT, even with sequential imaging, has a low diagnostic accuracy for osteomyelitis and cannot replace WBC scintigraphy in patients with diabetic foot.

How well did you know this?
1
Not at all
2
3
4
5
Perfectly
2
Q
  1. Runner with fusiform increased uptake on bone scan. Most likely:
    a. Shin splints
    b. Stress fracture
    c. Tumour
    d. Normal variant
A

ANSWER b. Stress fracture

  1. Runner with fusiform increased uptake on bone scan. Most likely:

a. Shin splints
b. Stress fracture
c. Tumour
d. Normal variant

Stress fracture (StatDx, Mettler)
• Tibia: typically proximal posterior cortex
• NM:
o Usually focal, fusiform or oval configuration of increased activity (more focal than shin splints)
o Typically positive on all 3 phases of bone scan

• MRI: highly specific
o T1 shows linear fracture line
o Fluid sensitive sequences: fracture line, surrounding marrow edema, & circumferential soft tissue signal

Shin splints (medial tibial stress syndrome) – spectrum of stress response to tibia (i.e periostitis – with no # formation yet)
• NM: Metter p273
o Normal blood flow & blood pool images
o On delayed images get linear increased activity along > one-third length of posteromedial tibial cortex at insertion of soleus muscle

• MRI: StatDx
o Linear hyperintense periosteal oedema/fluid in direct contact with medial tibial cortex (T2WI). Hyperintense edema/fluid extends anteromedial to posteromedial, and extends to orgin of soleus posteromedially (soleus bridge),
o Marrow hyperintensityMRI is the most sensitive test for stress injury, followed by NM.

How well did you know this?
1
Not at all
2
3
4
5
Perfectly
3
Q
  1. Runner with shin pain and transverse, linear uptake posteromedially on bone scan. Most likely:

a. Shin splints
b. Stress fracture
c. Tumour
d. Normal variant

A

*LW: possibly poor recall (transverse linear uptake, if stated vertical linear uptake, would favour shin splints, however based on stem I think stress fracture would be best option)

Shin splints show: Linear, superficial posterior medial tibial cortex, ≥ 1/3 of tibial length
Angiographic phase hyperemia absent/minimal.
Bone scan:
Normal blood flow & blood pool images
On delayed images get linear increased activity along > one-third length of posteromedial tibial cortex at insertion of soleus muscle
MRI:
Linear hyperintense periosteal oedema/fluid in direct contact with medial tibial cortex (T2WI). Hyperintense edema/fluid extends anteromedial to posteromedial, and extends to orgin of soleus posteromedially (soleus bridge),

Stress fracture / Fatigue Fracture:
Tibia: typically proximal posterior cortex
NM:
o Usually focal, fusiform or oval configuration of increased activity (more focal than shin splints)
o Typically positive on all 3 phases of bone scan
MRI:
o T1 shows linear fracture line
o Fluid sensitive sequences: fracture line, surrounding marrow edema, & circumferential soft tissue signal

ANSER:a. Shin splints T see above

  1. Runner with shin pain and transverse, linear uptake posteromedially on bone scan. (?? Transverse – maybe bad recall, meant longitudinal possibly)

a. Shin splints T see above
b. Stress fracture
c. Tumour
d. Normal variant

How well did you know this?
1
Not at all
2
3
4
5
Perfectly
4
Q
  1. A 60 year old man. Hot warm knee, swelling. Acute. Symmetrical loss of joint space with erosions on plain film. No other joints involved.

a. Trauma
b. Infection
c. Rheumatoid arthritis
d. Seronegative arthropathy
e. Gout
f. OA

A

ANSER:b. Infection= T (must be excluded)

  1. A 60 year old man. Hot warm knee, swelling. Acute. Symmetrical loss of joint space with erosions on plain film. No other joints involved.
    a. Trauma= F
    b. Infection= T (must be excluded)
    c. Rheumatoid arthritis = F
    d. Seronegative arthropathy = possible, but unlikely – the most common (AS) starts with sacroiliitis, and then tends to be a asymmetric polyarthropathy
    e. Gout = F clinically possible, but a key feature is preservation of the joint space until relatively late
    f. OA= F

Single joint = suspect infection;
DDx is crystal-deposition, PVNS, seronegative

Symmetric JS loss = inflammatory or crystal-deposition (although with gout often have partial preservation of JS); JS destruction in septic arthritis

Erosions = septic, inflammatory incl. seronegative, goutMust rule out infection

How well did you know this?
1
Not at all
2
3
4
5
Perfectly
5
Q
  1. Young man, acute pain in knee, afebrile, no other joints affected, no recent history. Imaging showed symmetrical jt narrowing and osteophytes. Most likely?

a. Trauma
b. Seronegative arthropathy
c. CPPD
d. Rheumatoid arthritis

A

ANSER:b. Seronegative arthropathy T most likely is reactive (Reiter) arthritis – men 20-40 years, uniform JS narrowing, erosive changes, osteophytes
*LW: agree this is most likely, however “no recent Hx” would semi argue against it, as would expect some form of Hx of a GI GU infection.

  1. Young man, acute pain in knee, afebrile, no other joints affected, no recent history. Imaging showed symmetrical jt narrowing and osteophytes. Most likely?
    a. Trauma F if secondary to trauma would be asymmetric JS narrowing
    b. Seronegative arthropathy T most likely is reactive (Reiter) arthritis – men 20-40 years, uniform JS narrowing, erosive changes, osteophytes
    c. CPPD F occurs > 50 years
    d. Rheumatoid arthritis F
How well did you know this?
1
Not at all
2
3
4
5
Perfectly
6
Q
  1. Melorrheostosis - NOT complicated by:
    a. Malignant transformation
    b. Muscle atrophy
    c. Contractures (skin)
    d. Sclerotomal distribution
A

ANSER:Probably Aa. Malignant transformation = ?F – isolated cases reported (bonetumor.org), not in Dahnert though

  1. Melorrheostosis - NOT complicated by:
    a. Malignant transformation = ?F – isolated cases reported (bonetumor.org), not in Dahnert though
    b. Muscle atrophy = T (present in some cases – F. Chew)
    c. Contractures (skin) = T (flexion contractures)
    d. Sclerotomal distribution = TDahnert, MSK Req p628, MSK Companion (F. Chew), StatDx & http://www.bonetumor.org/tumors-foot-and-ankle/melorheostosis

• Non-hereditary, idiopathic
• Assoc/ w/ osteopoikilosis, osteopathia striata, vascular malformations/tumours
• Monomelic in sclerotomal distribution
• Dense bone deposited along cortex (usually periosteal, but can be endosteal), usually involving only one side of the involved bone/s
• Candle-wax dripping
• May cross joint
• Flexion contractures
• Isolated cases of malignancy have been reported in association with melorheostosis, one osteosarcoma and one malignant fibrous histiocytoma.
StatDx says “Rarely associated with other bone malignancy such as giant cell tumor or osteosarcoma”.
• Bone scan: Increased blood flow on angiographic, blood pool, and delayed images

How well did you know this?
1
Not at all
2
3
4
5
Perfectly
7
Q
  1. 40 year old woman. Chronic pain, acutely worse. US shoulder shows anechoic region in thickened supraspinatus extending from humeral head surface to bursa
    a. Partial tear at joint side
    b. Full thickness tear
    c. Partial tear bursal side
    d. Calcific tendinitis
    e. CPPD
A

anser: b. Full thickness tear
7. 40 year old woman. Chronic pain, acutely worse. US shoulder shows anechoic region in thickened supraspinatus extending from humeral head surface to bursa

a. Partial tear at joint side
b. Full thickness tear
c. Partial tear bursal side
d. Calcific tendinitis
e. CPPD

Terminology (MSK Req p92)
• Full thickness tear = perforation that extends from bursal to articular surfaces
• Complete tear = disruption of entire tendon

MSK cases – US criteria for full-thickness supraspinatus tear
• Nonvisualization of the tendon or focal tendon defect filled with fluid and debris, with loss of the normal outward convexity of the tendon and dipping of the deltoid muscle into the tendon gap.
• The uncovered cartilage sign or naked cartilage sign is the hyperechoic interface between the joint fluid and the cartilage covering the humeral head.
• Other US signs are bone irregularity of the greater tuberosity, joint effusion, and fluid in the subdeltoid bursa

How well did you know this?
1
Not at all
2
3
4
5
Perfectly
8
Q
  1. SCFE . Slippage of femoral head is:
    a. Posterior and medial
    b. Posterior and lateral
    c. Anterior and medial
    d. Anterior and lateral
A

a. Posterior and inferomedial

How well did you know this?
1
Not at all
2
3
4
5
Perfectly
9
Q

O For which condition is US NOT the best first best

a. rotator cuff tear
b. instability
c. adhesive capsulitis
d. impingement
e. bursitis

A

B
According to Imaging pathways (WA)

a. rotator cuff tear = T = US + XRs
b. instability = F = XR then MR arthrography or CT arthrography
c. adhesive capsulitis F = MR arthrography modality of choice
d. impingement = T = US + XRsE = T = US
e. bursitis likely true

How well did you know this?
1
Not at all
2
3
4
5
Perfectly
10
Q

Oblique spiral fibula fracture from joint level

A

Webber B???

How well did you know this?
1
Not at all
2
3
4
5
Perfectly
11
Q
SUFE
a. rare diagnosed on AP 
B. usually 5-10 years of ages
C. usually bilateral
d. usually posteromedial slippage
E. early slippage frog leg view is best
A

So D or E??

A = F, can be seen often
B = F, typically 10-16 years (growth spurt), esp. obese, males & blacks
C = F, bilateral in 25-33%
D = T, epiphysis is rotated posteriorly & inferomedially
E = T 

Emedicine: The lateral radiograph demonstrates slippage earliest because the slippage begins with posterior displacement and progresses with medial rotation.

How well did you know this?
1
Not at all
2
3
4
5
Perfectly
12
Q

Which is a recognised feature of OA

a. subchondral sclerosis
b. osteopneia
c. subluxation
d. erosions

A

Probably A
A = T (subchondral sclerosis, asymmetric JS narrowing, osteophytes, subchondral cysts = OA)
B = F (inflammatory arthropathy)
C = T (can occur at 1st CMCJ, hip joint, patellofemoral joint, glenohumeral joint – StatDx)
D = ?F (erosions do not occur in the hand in pure OA, do occur in erosive OA – StatDx)

How well did you know this?
1
Not at all
2
3
4
5
Perfectly
13
Q
  1. Male, 50 year old patient, distal arthropathy including the (middle & index) PIPJ and DIPJ, soft tissue swelling, some erosions, most likely

a. Rheumatoid arthritis
b. Erosive arthritis
c. Seronegative arthritis
d. OA

A

ANSWER:c. Seronegative arthritis T – psoriasis most likely, but usually DIPJ predominance and asymmetric; against it is age & lack of ST swelling

  1. Male, 50 year old patient, distal arthropathy including the (middle & index) PIPJ and DIPJ, soft tissue swelling, some erosions, most likely
    a. Rheumatoid arthritis Possibly T, but typically more proximal joints; 3x’s more common in females
    b. Erosive arthritis ? T – gull-wing deformity (central erosions + peripheral osteophytes), soft tissue swelling, distal distribution; main negative is most commonly occurs in post-menopausal females (F»M 12:1, Statdx)
    c. Seronegative arthritis T – psoriasis most likely, but usually DIPJ predominance and asymmetric; against it is age & lack of ST swelling
    d. OA – F – productive only, no erosionse. ?
How well did you know this?
1
Not at all
2
3
4
5
Perfectly
14
Q
  1. 20 year old patient, knee pain, joint space narrowing (symmetric), some osteophyte formation, most correct

a. Previous trauma
b. Infection
c. Rheumatoid
d. OA
e. Seronegative
f. PVNS

A
  1. 20 year old patient, knee pain, joint space narrowing (? Symmetric), some osteophyte formation, most correct ANSWER: ? seronegative (*LW agrees)

Previous trauma ?T = possible, with secondary OA, common site; usually not symmetricg.

Infection = ?T = symmetric JS narrowing; usually no osteophytes until secondary OA occurs – may have been due to previous infection as a child?

h. Rheumatoid = JS narrowing that is uniform, erosions, effusion, osteopenia; but wrong demographic (usually women 40-70 years)
i. OA= asymmetric JS narrowing; older age
j. ? Seronegative = most likely is reactive (Reiter) arthritis – men 20-40 years, uniform JS narrowing, erosive changes, osteophytes
k. ? PVNS = no JS narrowing

How well did you know this?
1
Not at all
2
3
4
5
Perfectly
15
Q
  1. Soft tissue mass near the ankle, 30 year old female, High T2, peripheral enhancement, most correct
    a. Synovial sarcoma
    b. Myxoid sarcoma
    c. Ganglion
    d. Giant cell tumour
A

ANSWER: c. Ganglion – T2 very bright; can have thin enhancement of wall; usually about the wrist, also occur about knee & ankle

  1. Soft tissue mass near the ankle, 30 year old female, High T2, peripheral enhancement, most correct (StatDx)A
    a. Synovial sarcoma – T2 heterogeneous hyperintensity; most common around knee & foot; age 16-36; marked heterogeneous enhancement
    b. Myxoid sarcoma – most common in lower extremity, esp. in thigh; T2 intermediate-high signal (myxoid = high T2 signal in general); variable enhancement (myxoid tissue typically hypoenhancing)
    c. Ganglion – T2 very bright; can have thin enhancement of wall; usually about the wrist, also occur about knee & ankle
    d. Giant cell tumour- can involve tendon sheaths of ankle/foot (although favours hand & wrist); variable T2 hypo-hyperintensity (internal hypointensities due to haemorrhage); intense enhancement +/- heterogenous
How well did you know this?
1
Not at all
2
3
4
5
Perfectly
16
Q
  1. Child sustains a forearm injury, pain at the wrist with a deformity at the mid forearm, most likely?

a. Distal radius fracture with ulna bowing
b. Distal radius fracture with dislocation at the distal radial ulnar joint
c. Ulnar midshaft fracture with radial head dislocation
d. Fracture scaphoid

A

ANSWER:b. Distal radius fracture (?metaphysis) with dislocation at the distal radial ulnar joint

  1. Child sustains a forearm injury, pain at the wrist with a deformity at the mid forearm, most likely?
    a. Distal radius fracture (?metaphysis) with ulnar bowing
    b. Distal radius fracture (?metaphysis) with dislocation at the distal radial ulnar joint
    c. Ulnar midshaft fracture with radial head dislocation

d. Fracture scaphoidReferring to Galeazzi fracture (see pic below) = fracture of radial shaft (mid-distal) with DRUJ subluxation
(FROG = fractured radius of galeazzi)c.f. Monteggia fracture = fracture of proximal ulnar shaft with radial head dislocation

How well did you know this?
1
Not at all
2
3
4
5
Perfectly
17
Q
  1. 41 year old runner presents with leg pain. Bone scan show linear uptake in the posteromedial tibia. Which is MOST likely?
    a. Shin splint
    b. Stress fracture
    c. Normal variant
    d. Primary bone tumour
    e. Metastasis
A

B. shint splint

Stress fracture (StatDx, Mettler)
• Tibia: typically proximal posterior cortex
• NM:
o Usually focal, fusiform or oval configuration of increased activity (more focal than shin splints)
o Typically positive on all 3 phases of bone scan
• MRI: highly specific
o T1 shows linear fracture line
o Fluid sensitive sequences: fracture line, surrounding marrow edema, & circumferential soft tissue signal

Shin splints (medial tibial stress syndrome)
• NM: Metter p273
o Normal blood flow & blood pool images
o On delayed images get linear increased activity along > one-third length of posteromedial tibial cortex at insertion of soleus muscle
• MRI: StatDx
o Linear hyperintense periosteal oedema/fluid in direct contact with medial tibial cortex (T2WI). Hyperintense edema/fluid extends anteromedial to posteromedial, and extends to orgin of soleus posteromedially (soleus bridge),
o Marrow hyperintensityMRI is the most sensitive test for stress injury, followed by NM.

How well did you know this?
1
Not at all
2
3
4
5
Perfectly
18
Q
  1. Patient presents with shoulder pain. Plain x-ray is normal. Clinically subacromial bursitis is suspected. Which is the NEXT most appropriate investigation?
    a. CT
    b. MRI
    c. USS
    d. NM Bone Scan
    e. Repeat x-ray in 7-10 days
A

Ultrasound. As per WA imaging guideline. Imaging, 2003 (UK)
The primary purpose of imaging in impingement is to demonstrate if there is a tear and to assess its size. The degree of atrophy of the rotator cuff muscle has been cited as an indicator of predicting the rate of rerupture but it is unclear whether this is an important consideration in the younger population with small or medium sized tears that make up the major of those patients who undergo surgery [7].

US and MRI are the two main tests used in impingement. US has the advantage of being a rapid and accurate method of diagnosing rotator cuff tears and is suitable for one-stop combined clinics with instant access to scanning.
MRI is also an accurate technique for tears and gives a broader overview of the shoulder. MRI is expensive, often disliked by patients and not amenable to providing an instant access service. If the question to be answered is “is there a rotator cuff tear” then US is the preferred technique. Bursal abnormalities, including dynamic signs of impingement, calcific deposits, and irregularity of the greater tuberosity are other common findings that are clearly identified on US.UTD, 2011 (USA)Magnetic resonance imaging — MRI is the preferred imaging study for patients with suspected impingement and rotator cuff injury. A normal MRI suggests that the likelihood of a rotator cuff tear is less than 10 percent [30-32]. On the other hand, MRI findings for rotator cuff tears are not highly specific, particularly in older patients [33]. The sensitivity and specificity of MRI for the diagnosis of impingement are approximately 93 and 87 percent, respectively [34]. MRI is also useful in the evaluation of avascular necrosis, biceps tendinopathy and rupture, inflammatory processes, and tumors [35].Ultrasonography — In the hands of skilled operators, the diagnostic accuracy of ultrasound has been found to be the equivalent of MRI in identifying rotator cuff tears, labral tears, and biceps tendon tears and dislocations [36-42]. Ultrasound is less expensive than MRI and preferred by patients [42,43].

How well did you know this?
1
Not at all
2
3
4
5
Perfectly
19
Q
  1. 20 yo. Round soft tissue mass around knee. CT shows lots of calc. Most likely
    a. fibroma
    b. desmoid
    c. gouty tophus
    d. synovial sarcoma
A

Probably Synovial sarcoma ( can calcify)

A – fibroma (?T) – possibly a fibroma of tendon sheath – low-intermediate T1, heterogenous low-high signal on T2, variable enhancement (StatDx); Dahnert says fibroma of soft tissue hypointense nodule on all MR sequences. Have seen one at PAH immediately above upper pole of patella which was T2 hypointense. However can’t find calcification in any reference.

B – desmoid = deep fibromatosis – T2 iso to muscle, T2 intermediate to hyperintense, no calcs

C – tophus = unlikely in 30 yo male; usually not calcified unless renal failure; gouty tophus low-intermediate T2 signal (Dahnert; StatDx says variable signal)

D = synovial sarcoma (?T) = best answer = 20’s to 40’s, most occur about knee or thigh; calcification in 30%; usually T2 heterogenously hyperintense though (hypointense components).Soft tissue lesion with calcification near a joint → think of SS.

How well did you know this?
1
Not at all
2
3
4
5
Perfectly
20
Q
  1. optimal position to image infrapsinatous on US.
    b. internal rotation, arm behind back
    c. internal rotation, touching opposite shoulder
A

*LW:
Radiopedia state hand on opposite shoulder
radiographics: start hand on lap supinated, and if required progress to hand on opposite shoulder.

Suprapinatus position: hand reaches to get wallet from back pocket position.
Biceps: Patient position: arm in neutral position, elbow flexed 90 degrees, forearm supinated (palm up).
Subscapularis: Patient position: arm in neutral position, elbow flexed 90 degrees, forearm supinated (palm up), and then externally rotated.

Probably C.
• Infraspinatus
o Back of hand placed behind the back (shoulder extended in IR) or forearm supinated & placed on thigh +/- slight IR (Radiology 2011) or place hand on opposite shoulder (ESR)
o From SST pan posteriorly to IST – tendons difficult to separate at US
o Can also image IST by moving transducer posteriorly, often aided by passive IR & ER

How well did you know this?
1
Not at all
2
3
4
5
Perfectly
21
Q
Production line worker. Shoulder pain. Suspect impingement. Xray normal. Next best test:
A. US
B. MRI
C. CT
D. Re-xray in 10 days
A

US- in train specialist these can be as effective as MRI

How well did you know this?
1
Not at all
2
3
4
5
Perfectly
22
Q

Fracture proximal fibula + talar shift =

A

maisonneuve.note. dupentures # - think of bimalleolar # with tibiofibular ligament rupture and talar shift

How well did you know this?
1
Not at all
2
3
4
5
Perfectly
23
Q

What is a toddler #

A

Toddler fracture: Clinically subtle lower extremity fracture in a toddler or young child that results in refusal to bear weight, gait disturbance, or inability to walk. Possible sites of toddler’s fractures are midshaft of tibia (spiral), proximal tibia, distal fibular metadiaphysis, cuboid, calcaneum, talus & metatarsals.

How well did you know this?
1
Not at all
2
3
4
5
Perfectly
24
Q
Woman with increasing pain in shoulder. US thickened supraspinatous. Echogenic stuff in bursa or tend? with some shadowing. Most likely 
A. CPPD. 
b. HADD. 
C. partial tear
D. full tear
A

b. HADD (calcified tendinosis of supraspinatous tendon)

How well did you know this?
1
Not at all
2
3
4
5
Perfectly
25
Q
Woman with increasing pain in shoulder. US thickened supraspinatous. Echogenic stuff in bursa or tend? with some shadowing. Most likely 
A. CPPD. 
b. HADD. 
C. partial tear
D. full tear
A

b. HADD (calcified tendinosis of supraspinatous tendon)

How well did you know this?
1
Not at all
2
3
4
5
Perfectly
26
Q

Definition of foot malformations

a. club foot is tailpipes equinovarus
b. def of pes.
c. valgus is foot turned medially

A
A = T
B = ? (pes means foot)
C = F (turned laterally)

• Pes: the foot
• Talipes: a congenital deformity in which the foot is twisted out of shape or position
• Hindfoot equinus: fixed plantarflexion of the calcaneus (distal end pointing down)
o Evaluated only on lateral WB film
o Decreased calcaneal pitch (< 200)
o Calcaneal-tibial angle > 90deg
• Hindfoot calcaneus: calcaneus excessively dorsiflexed (calcaneal-tibial angle < 60deg)
o Seen in cavus & spastic deformities
• Hindfoot varus: distal bone of hindfoot (calcaneum) is angled too far medially in relation to the more proximal bone of the hindfoot (talus)
o Assessed with DP & lateral WB films
o AP talocalcaneal angle < 150 (approaching parallelism of bones); talus will point lateral to MT1 (normally passes thru or just medial to MT1)
o Lateral talocalcaneal angle < 250 (bones approach parallelism)
• Hindfoot valgus: calcaneum angled too far laterally in relation to the taluso AP talocalcaneal angle > 40deg; talus will point medial to MT1 (normally bisects MT1)
o Lateral talocalcaneal angle > 45deg (talus has increased plantarflexion)
• Forefoot varus: forefoot angled medially relative to the hind/midfoot – forefoot is inverted & often slightly supinatedo Forefoot appears narrowed on DP radiograph, with increased convergence of the bases of the metatarsals
o On lateral view metatarsals have a ladder-like arrangement, with the 5th MT in the most plantar position
• Forefoot valgus: forefoot angled laterally relative to the hind/midfoot – forefoot is everted & often pronatedo Forefoot appears broadened on DP radiograph, with a decrease in overlap of bases of metatarsals
o On lateral view the metatarsals are more superimposed & the 1st MT becomes the most plantar (normally 5th MT is most plantar)
• CTEV consists of
o Hindfoot equinus
o Hindfoot varus
o Forefoot varus
o AKA “clubfoot”

How well did you know this?
1
Not at all
2
3
4
5
Perfectly
27
Q

RA

a. age of presentation is 20-40
b. effusion happens late
c. peri-articular osteopenia is a sign of early disease

A

ANSWER: c. hallmark (periarticular erosion = early disease)- age of presentation is 40-70- effusion is an early sign

**LJS - recent Robbins, age 20-50yr

*LW: RP states Onset is generally in adulthood, peaking in the 4th and 5th decades = 30s-40s, effusions happen EARLY, peri articular osteopenia is relatively early that progresses to more diffuse osteopenia…. so maybe question was which is false?

How well did you know this?
1
Not at all
2
3
4
5
Perfectly
28
Q

27) Dwarf. Low back pain radiating down both legs. Most likely cause:
i) Overexaggerated lordosis of sacrum
ii) Short pedicles
iii) Scalloping of vertebral bodies
iv) Scoliosis
v) Sacrosciatic action

A

ANSWER:ii) Short pedicles T – short pedicles with lumbar spinal canal stenosis and disc prolapse can result in severe neurologic deficits (Apley

)i) Overexaggerated lordosis of sacrum = F

ii) Short pedicles T – short pedicles with lumbar spinal canal stenosis and disc prolapse can result in severe neurologic deficits (Apley)
iii) Scalloping of vertebral bodies = F – this does occur, but doesn’t account for S&S
iv) Scoliosis = F
v) Sacrosciatic action= F, although small sacrosciatic notches are a feature

How well did you know this?
1
Not at all
2
3
4
5
Perfectly
29
Q

28) Chance fracture following MVA. Most relevant:

i) Almost universally has neurological deficit
ii) Vertebral disc distraction
iii) Spinal cord injury
iv) Flexion compression fracture of middle column

A

ANSWER:iii) Spinal cord injury T – cord injury can occur due to retropulsion of posterior vertebral body cortex (StatDx), but as above it is uncommon.

2) Chance fracture following MVA. Most relevant:
i) Almost universally has neurological deficit F neurologic damage is uncommon, although the injury is unstable (Apley). MSK CRS – acutely unstable – pure osseous fractures heal well with long-term stability likely, while pure ligamentous injuries have high risk of residual instability. Neurologic injury in 20% (Dahnert)

.ii) Vertebral disc distraction F anterior compression to disc/body (StatDx)

iii) Spinal cord injury T – cord injury can occur due to retropulsion of posterior vertebral body cortex (StatDx), but as above it is uncommon.
iv) Flexion compression fracture of middle column F - Compression injury of anterior column with distraction of middle & posterior columns (StatDx)15-80% (> 50%) have significant abdominal injuries (bowel & mesentery most common)

How well did you know this?
1
Not at all
2
3
4
5
Perfectly
30
Q

29) Spondyloepiphyseal dysplasia. NOT a feature:

i) Atlanto-occipital instability
ii) Kyphosis
iii) Normal sized hands and feet
iv) Coxa vara
v) Normal femoral head ossification

A

ANSWER:v) Normal femoral head ossification F epiphyseal dysplasia – ossification of epiphyses delayed, often being irregular & fragmented; early OA

3) Spondyloepiphyseal dysplasia. NOT a feature: MSK Req p636
i) Atlanto-occipital instability T C1-2 instability with dens hypoplasia
ii) Kyphosis T progressive kyphoscoliosis (Dahnert)
iii) Normal sized hands and feet T normal or slightly shortened limbs, with multiple accessory epiphyses in hands & feet
iv) Coxa vara T severe coxa vara + genu valgum
v) Normal femoral head ossification F epiphyseal dysplasia – ossification of epiphyses delayed, often being irregular & fragmented; early OA

How well did you know this?
1
Not at all
2
3
4
5
Perfectly
31
Q
30)	15yo male.  Generalised bone pain. Cardiomegaly.  Broad ribs. Flattened thoracic vertebrae.  Most likely
\:i)	Glycogen storage disease
ii)	Sickle cell
iii)	Thalassemia
iv)	Fibrous dysplasia
v)	Mets
A

iii) Thallasaemia T get expansion of the marrow cavity, osteopaenia, crush fractures, H-shaped vertebra, anaemia leading to high output cardiac failure

How well did you know this?
1
Not at all
2
3
4
5
Perfectly
32
Q

31) Which is the most correct regarding a cortical desmoid?
i) Posteromedial condyle
ii) Lateromedial condyle
iii) Increased signal on STIR imaging
iv) Increased uptake on bone scan in 70%
v) Biopsy required for diagnosis

A

ANSWER:iii) Increased signal on STIR imaging T high T2/STIR signal at site

5) Which is the most correct regarding a cortical desmoid? (= distal femoral metaphyseal irregularity)
i) Posteromedial condyle F posteromedial femoral metaphysis just lateral to adductor tubercle
ii) Lateromedial condyle F
iii) Increased signal on STIR imaging T high T2/STIR signal at site
iv) Increased uptake on bone scan in 70% F normal or slightly increased uptake
v) Biopsy required for diagnosis F don’t touch lesion

Cortical desmoid is a benign shallow cortical irregularity seen in older children or adolescents. It is recognised at specific metaphyseal sites, the most common being the medial posterior aspect of the distal femoral metaphysis at the insertion of the adductor magnus aponeurosis. Other sites include the proximal tibia, medial aspect proximal humerus and distal radius. The sites generally coincide with muscle or tendon insertions and it has been suggested that the lesion may reflect a chronic avulsion injury. The lesions are 1 to 3 cm. in size, irregular, with areas of lucency and areas of sclerosis. There may be some mineralization within the lesion. The margin may be latent or active. Bone scan will show uptake due to the presence of reactive bone, then become “cold” as the lesion involutes. CT scan will differentiate this lesion from osteoid osteoma and show the complex nature of the lesion, with areas of cortical thinning and thickening, some possible small “cystic” araes, and the surrounding mild sclerotic bone reaction. MRI in general is helpful but can be confusing. Biopsy is not indicated or warranted. A lesion whose nature is in doubt should be referred by an orthopaedic oncologist.

How well did you know this?
1
Not at all
2
3
4
5
Perfectly
33
Q

32) 80yo man with rib pain. Bone scan shows fusiform increased posterolaterally in left 7th rib. Most likely:

i) Myeloma
ii) Met
iii) Fracture
iv) Osteoid osteoma
v) Infection

A

ANSWER:iii) Fracture T Usually focal (punctate foci of ↑ activity) rather than infiltrative or elongated. More likely if ≥ two consecutive ribs are involved.

6) 80yo man with rib pain. Bone scan shows fusiform increased posterolaterally in left 7th rib. Most likely:
i) Myeloma F Unlikely to be single; sensitivity for lesion detection 75-85%, but poor sensitivity for lytic/trabecular lesions
ii) Met ? T More linear-type increased uptake, usually multiple (single lesions less commonly neoplastic, however metastastic disease not excluded). Mettler – a single focus of increased activity in a rib is due to metastases in only 10% of cases. A key exception is single sternal focus in woman with breast cancer (80% chance of being a met).
iii) Fracture T Usually focal (punctate foci of ↑ activity) rather than infiltrative or elongated. More likely if ≥ two consecutive ribs are involved.
iv) Osteoid osteoma F
v) Infection F

How well did you know this?
1
Not at all
2
3
4
5
Perfectly
34
Q

33) 20yo jogger with left shin pain. Focused left lower limb bone scan shows increased uptake in the proximal fibula. Most likely:

i) Normal variant for age
ii) Osteoid Osteoma
iii) Shin Splints
iv) Stress fracture
v) Metastasis
vi) Primary bone tumour

A

ANSWER:iv) Stress fracture – StatDx NM: Usually focal, fusiform or oval configuration of increased activity; positive on all three phases. More focal/fusiform c.f. medial tibial stress syndrome. Fibular stress fracture usually distal third but can be in proximal third.

7) 20yo jogger with left shin pain. Focused left lower limb bone scan shows increased uptake in the proximal fibula. Most likely:A

i) Normal variant for age
ii) Osteoid Osteoma Typically have intense uptake on bone scan. Should have pain at the site, worse at night & relieved by aspirin/NSAIDs.
iii) Shin splint – F – Shin splint = medial tibial stress syndrome. StatDx: Radionuclide activity posteromedial border tibia on delayed images (involves ≥ 1/3 length of tibia), but normal blood flow & blood pool. MRI = Hyperintense edema/fluid signal (T2WI) medial tibial border. Often bilateral. See Mettler NM p270-. Occurs at insertion of soleus muscle
.iv) Stress fracture – StatDx NM: Usually focal, fusiform or oval configuration of increased activity; positive on all three phases. More focal/fusiform c.f. medial tibial stress syndrome. Fibular stress fracture usually distal third but can be in proximal third.
v) Metastasis - F
vi) Primary bone tumour - F

How well did you know this?
1
Not at all
2
3
4
5
Perfectly
35
Q
34)	Least correct regarding osteoarthritis
\:i)	Reduced mineralization
 ii)	Erosions
 iii)	Unilateral / asymmetrical 
 iv)	Subluxation  
v)	Osteophytes
A

ANSWER:i) Reduced mineralization F usually no osteopenia, usually have sclerosis

8) Least correct regarding osteoarthritis:
i) Reduced mineralization F usually no osteopenia, usually have sclerosis
ii) Erosions T/F can occur in erosive OA, subchondral cysts in conventional OA
iii) Unilateral / asymmetrical T
iv) Subluxation T – atypical, but can occur in SLAC wrist
v) Osteophytes T

How well did you know this?
1
Not at all
2
3
4
5
Perfectly
36
Q

35) A 7 y.o girl comes in with hip pain and fever. Normal x-ray. What would be the next most appropriate examination to perform?
i) Bone scan
ii) MR
Iiii) Repeat films in 7 - 10 days time
iv) Bilateral Hip US

A

ANSWER:iv) Bilateral Hip US

9) A 7 y.o girl comes in with hip pain and fever. Normal x-ray. What would be the next most appropriate examination to perform?
i) Bone scan Only if XR & US unhelpful & usually only if MRI not available
ii) MRI May be indicated, but not as next step
iii) Repeat films in 7 - 10 days time
iv) Bilateral Hip US

How well did you know this?
1
Not at all
2
3
4
5
Perfectly
37
Q
36)	Elite rugby player with recurrent dislocations. GP rings wanting to know best assessment. You say
\:i)	US
ii)	Xray
iii)	CT
iv)	MRI
v)	Bone scan
A

ANSWER: MRI

10) Elite rugby player with recurrent dislocations. GP rings wanting to know best assessment. You say:

i) US
ii) Xray
iii) CT
iv) MRI
v) Bone scan

UTD 2011MR arthrography is the method of choice for imaging the labrocapsular structures. MR arthrography is the most accurate imaging modality for sports injuries of the shoulder and in evaluating instability.

How well did you know this?
1
Not at all
2
3
4
5
Perfectly
38
Q

37) MRI shoulder in young guy with shoulder pain. Area of high signal in supraspinatus tendon only visualised on T1 sequences.
i) Full thickness supraspinatus tear
ii) Magic angle
iii) Partial thickness supraspinatus tear
iv) Anisotropy of tendon

A

ANSWER: MAGIC ANGLE

Magic angle refers to spuriously increased signal intensity that may occur within any tissue containing highly structured collagen fibers (tendon, ligament, meniscus, labrum), depending on its position within the magnetic field. This magic angle effect is due to the orientation of the collagen bundles and occurs when the structure lies at an angle near 55 degrees to the main magnetic field. The resulting increased signal is seen on images obtained with a short TE (T1, proton density, and most gradient echo sequences), but disappears on long TE (T2W) images. This latter feature allows for differentiation from true tendon pathology. Other supportive signs include a lack of tendon enlargement or peritendinous edema.In the shoulder, this phenomenon occurs commonly about 1 cm proximal to the insertion of the supraspinatus tendon on the greater tuberosity, which is the hypovascular region of the tendon, also known as the critical zone.The intermediate signal intensity from the magic angle phenomenon disappears with long TE sequences, such as T2W images, making it possible to differentiate magic angle from an abnormal tendon. A good rule to distinguish the magic angle phenomenon from a tear on T2* images is that the signal intensity within the tendon is never higher than the signal intensity within the adjacent muscle if it is from the magic angle phenomenon (see Fig. 10-2), whereas with a tendon tear, the signal intensity is higher than that of muscle.

How well did you know this?
1
Not at all
2
3
4
5
Perfectly
39
Q

38) US shoulder in young guy with shoulder pain. Supraspinatus tendon normal morphology/thickness and no fluid in subdeltoid/subacromial bursa. Decreased echogenicity and no visualisation of internal fibres at insertion. Likely due to:
i) Full thickness supraspinatus tear
ii) Magic angle
iii) Partial thickness supraspinatus tear
iv) Anisotropy of tendon

A

ANSWER: ANISOTROPY OF TENDONAnisotropy is the property of tendons, nerves and muscles to vary in their ultrasound appearance depending on the angle of insonation of the incident ultrasound beam. Loss of reflectivity in tendons may also denote underlying disease. A lesion can only be confirmed if a poorly reflective area remains when the angle of insonation is perpendicular to the long axis of the tendon.

How well did you know this?
1
Not at all
2
3
4
5
Perfectly
40
Q

39) Morton’s neuroma

A
Morton neuroma (StatDx):
•	Best diagnostic clue: Soft tissue mass between ± distal to 3rd + 4th metatarsal heads (3rd intermetatarsal space)
•	Location: 3rd common digital branch of medial plantar nerve, plantar to deep transverse intermetatarsal ligament
•	Intermediate to hyperintense on FS PD FSE + STIR images
•	Effacement of plantar subcutaneous fat by convex border of teardrop-shaped mass
How well did you know this?
1
Not at all
2
3
4
5
Perfectly
41
Q

40) SLE arthopathy is NOT associated with:
a. Subluxations and soft tissue calcification
b. Bilateral and symmetrical distribution
c. Periarticular osteopaenia
d. Erosions
e. Osteonecrosis
f. Soft tissue swelling

A

ANSWER:d. Erosions

14) SLE arthopathy is NOT associated with:
a. Subluxations and soft tissue calcification – T Prominent MCPJ subluxations; 10% have SC calcifications
b. Bilateral and symmetrical distribution T
c. Periarticular osteopaenia T may be seen
d. Erosions F – no erosions, JS narrowing uncommon
e. Osteonecrosis T high incidence of AVN due to steroids & the vasculitis (acute necrotising vasculitis of small arteries)
f. ? Soft tissue swelling T

How well did you know this?
1
Not at all
2
3
4
5
Perfectly
42
Q

41) YOUNG MAN WITH RIB LESION. destruction and pleural effusion.
a) askin tumour
b) fibrous dysplasia
c) neuroblastoma metastasis

A

ANS = A = Askin tumour (StatDx):
• Primitive neuroectodermal tumor (PNET) arises in pleura or chest wall (i.e. related to Ewing & other PNET type tumours)
• Although rare, most common pleural tumor in teenagers & young adults (especially females)• Often large
• Involves rib 23-60% of time

How well did you know this?
1
Not at all
2
3
4
5
Perfectly
43
Q

42) Which of the following is not associated with melorheostosis…
a. Muscle atrophy
b. Contractures
c. Sclerotomal distribution
d. Juxta-articular
e. malignant transformation
f. Soft tissue calcification

A

e. Malignant transformation T/F – some case reports of this… (but not mentioned in Dahnert)

Which of the following is not associated with melorheostosis…

a. Muscle atrophy T common
b. Joint contractures T flexion contractures of hip & knee
c. Sclerotomal distribution T monomelic
d. Juxta-articular T predominantly affects the diaphysis, but may extend into epiphysis & even cross the joint
e. Malignant transformation T/F – some case reports of this… (but not mentioned in Dahnert)
f. Soft tissue calcification T para-articular soft tissue ossification is uncommon, but can occur in severe cases (Dahnert says ossified soft-tissue masses in 27%)

How well did you know this?
1
Not at all
2
3
4
5
Perfectly
44
Q

43) 40 yo man with painful arc. 30-60 yo. Most likely U/S finding
a. complete supraspinatous tendon tear
b. florid subacromial bursitis
c. partial thickness supraspinatous tear
d. HADD

A

*LW: Likely C, although B possible is felt less likely.
If was tendinosis as stated, this would be preferred option in absence of dynamic impingement manoeuvres.

(Long explanation sorry)
Clinical Hx is describing subacromial impingement.
“Subacromial impingement is a clinical syndrome of anterolateral shoulder and/or lateral upper arm pain that occurs during elevation of the arm as a mid‐range “painful arc” that, in lesions of the rotator cuff, is believed to reflect compression of the rotator cuff and/or subacromial‐subdeltoid (SA‐SD) bursa by the overlying coraco‐acromial arch.

This definition captures the key features of subacromial impingement:

■ The condition is a clinical syndrome of pain (not a static or dynamic grayscale ultrasound appearance)
■ The pain is felt in the shoulder and/or upper arm through an “arc” of mid‐range elevation of the arm
■ Pain is due to mechanical compression of the rotator cuff and/or SA‐SD bursa by the coraco‐acromial arch.

US diagnosis is usually achieved via dynamic impingement examinations, e.g. Needs test.
Additiona signs of subacromial impingement:
(a) “bunching” or fluid distension of the SA‐SD bursa lateral to the impingement point at coraco‐acromial arch
b) “bunching” of the supraspinatus tendon lateral to the impingement point at coraco‐acromial arch
(c) bulge of the coraco‐acromial ligament ;
(d) less commonly, complete “blocking” of supraspinatus tendon motion due to “migration of the humeral head upward to prevent its passage beneath the acromion”

Thus:

1) complete US tear is usually associated with chronic impingement, in the elderly.
2) Florid sub acromial bursitis is possible, but by itself is not specific.
3) Partial thickness supraspinatus tear, possible, especially according to Gartsman sub classification of Neer’s pathological class-action into Type 2B which shows partial tears in patients 25-40yrs.
4) HADD, different clinical presentation.

Previous Answer:
answer: C

A = F
B = less likely
C = T = partial thickness SST tear (partial tears are more painful than full thickness tears!)
D = F = aching pain, not painful arc

Apley Ortho.
• Painful arc syndrome = subacute tendinosis: usually pain at 60-120 degrees of abduction
• Cuff disruption: when complete tear patient is unable to abduct the arm; when complete tear pain subsides over weeks but abduction remains absent
• Acute calcific tendinosis = aching pain

How well did you know this?
1
Not at all
2
3
4
5
Perfectly
45
Q

44) MRi shoulder. True
a. fluid in subldeltoid bursa is diagnostic of supraspinatous tear.
b. fluid in bicipital bursa means biceps pathology
c. magic angle affect more likely with long TE.
d. labral tear has high association with instability

A

ANSWER: D

A = F = a sign of SST tear, but also occurs in bursitis
B = F = can occur in joint effusion
C = F = less likely on longer TE sequences (T2)
D = T = most labral tears occur in association with shoulder dislocation & instability
How well did you know this?
1
Not at all
2
3
4
5
Perfectly
46
Q

40 yo lady with 3m shoulder pain. U/S shoulder shows thickened hypo echoic supraspinatous tendon. Most likly

a. complete supraspinatous tendon tear
b. rheumatoid tendinopathy
c. partial thickness supraspinatous tear
d. traumatic tendinopathy

A

ANSWER: ?D. Traumatic tendinopathy ? ?? Tendon degeneration shows thickening & heterogenous echoes at US; at MRI increased signal on PD that does not increase on T2

  1. 40yr female with 3m shoulder pain. US shoulder shows thickened hypoechoic supraspinatus tendon. Most likely

A. Complete supraspinatus tendon tear F – defect in tendon, flattening or concavity of echogenic subdeltoid fat, subdeltoid bursal fluid, irregular GT, tendon retraction

B. Rheumatoid tendonopathy F cuff atrophy in RA

C. Paritial thickness supraspinatus tear ? F – focal thinning/defect in tendon; decreased echogenicity &thinning at tear site

D. Traumatic tendinopathy ? ?? Tendon degeneration shows thickening & heterogenous echoes at US; at MRI increased signal on PD that does not increase on T2

Rotator cuff tendinopathy
• Degenerative changes in SST• US = thickened hypoechoic tendon +/- tears• MR = thickened tendon of T1 intermediate & T2 increased signal

How well did you know this?
1
Not at all
2
3
4
5
Perfectly
47
Q

46) Regarding radiographs of the shoulder which is most correct?
a. Acromiohumeral distance is normally < 7mm
b. Calcification in the supraspinatus tendon are best seen in external rotation
c. Hill-Sachs lesion is best seen in external rotation
d. Rotator cuff calcification is the most common radiographic finding in impingement

A

ANSWER: b. Calcifications in the supraspinatus tendon are best seen in external rotation T

*LW agrees: Ca++ in supraspinatus best seen in profile over GT on AP in external rotation.
To assess infraspinatus and tires minor; internal rotation is best suited to bring the more posteriorly based muscles into profile.

  1. Regarding radiographs of the shoulder which is most correct?
    a. Acromiohumeral distance is normally < 7mm F normally 7-14mm; if < 6mm suggests chronic rotator cuff tear
    b. Calcifications in the supraspinatus tendon are best seen in external rotation T
    c. Hill-Sachs lesion is best seen in external rotation F internal rotation – this defect is visualized on AP x-ray w/ arm in internal rotation & Stryker Notch view & may be missed on routine AP views (Wheeless)
    d. Rotator cuff calcification is the most common radiographic finding in impingement F

Best views (radiographs)
• ER → supraspinatus +/- infraspinatus
• IR → subscapularis & teres minor +/- infraspinatus; Hill-Sachs lesion

How well did you know this?
1
Not at all
2
3
4
5
Perfectly
48
Q

47) AVN OF HIP, not a cause?
a. gaucher
b. SLE
c. untreated and spondylitis
d. pancreatitis

A

c. untreated and spondylitis

* LW: presume this means untreated ankylosing spondylitis

How well did you know this?
1
Not at all
2
3
4
5
Perfectly
49
Q

48) Radiation-induced bone malignancy
a. Chondrosarcoma most common radiation-induced tumour
b. 20-25 yrs average lag time of radiation-induced malignancy
c. Radiation osteitis & radiation-induced malignancy can be differentiated by change in pattern
d. Radiation osteoradionecrosis hot on bone scan

A

PROBABLY C:c. Radiation osteitis & radiation-induced malignancy can be differentiated by change in pattern ?F osteoradionecrosis can be extremely difficult to differentiate from recurrent tumour, although frank bone destruction in the radiation portal without a soft tissue mass is most suggestive of ORN (AJR 10). If an associated ST mass is seen, biopsy is needed. Key sign is new destructive osseous change in radiated field – can be difficult to recognise (StatDx)

  1. Radiation-induced bone injury/malignancy
    a. Chondrosarcoma most common radiation-induced tumour F – most common benign XRT-induced tumour is osteochondroma; most common malignant tumours are osteosarcomas (90%) & fibrosarcomas/MFH, with one-third arising in pre-existing lesions (RG 1998; Dahnert)
    b. 20-25 yrs average lag time of radiation-induced malignancy F average lag-time of 11-14 years, can occur from 4-42 years post XRT
    c. Radiation osteitis & radiation-induced malignancy can be differentiated by change in pattern ?F osteoradionecrosis can be extremely difficult to differentiate from recurrent tumour, although frank bone destruction in the radiation portal without a soft tissue mass is most suggestive of ORN (AJR 10). If an associated ST mass is seen, biopsy is needed. Key sign is new destructive osseous change in radiated field – can be difficult to recognise (StatDx)
    d. Radiation osteoradionecrosis hot on bone scan F - decreased uptake (Dahnert)Radiation necrosis is dose-dependant, with “radiation osteitis” seen at around 30 Gy and osteoradionecrosis assoicated with doses of 50 Gy or higher. Radiation osteitis is a term used to describe potentially reversible changes such as temporary cessation of growth, periostitis, bone sclerosis and increased fragility, ischemic necrosis and infection.

Radiographs will show bone which is mottled demonstrating both osteopenia and sclerosis and areas of coarse trabeculation. Some investigators and clinicians believe that radiation osteitis is the set-up for osteoradionecrosis and that a patient does not progress to necrosis unless infection is present. [http://uwmsk.org/residentprojects/radiationchanges.html].

How well did you know this?
1
Not at all
2
3
4
5
Perfectly
50
Q

49) MRI and ACL injuries, which is false:

a. Meniscocapsular separation of the posterior horn of the medial meniscus
b. Bone contusion of the posterolateral tibial plateau
c. Iliotibial band avulsion
d. Bone contusion anterior aspect of the medial femoral condyle
e. Posterior horn medial meniscus tear

A

ANSWER: d. Bone contusion anterior aspect of the medial femoral condyle . -> should be posterior aspect medial femoral condyle in countercoup injury.
*LW: Out of all options this is probably least correct.

a. Meniscocapsular separation of the posterior horn of the medial meniscus T meniscocapular separation assoc/ w/ ACL &/or MCL tears. Sagittal T2 shows widening of meniscocapsular interval ≥ 5mm. Most common at posterior horn MM (StatDx).
b. Bone contusion of the posterolateral tibial plateau T standard is posterolateral corner tibia, lateral femoral condyle +/- posteromedial tibial condyle
c. Iliotibial band avulsion T Gerdy tubercle avulsion (anterolateral tibial plateau), RG 08. Some say Segond fracture is ITB avulsion.
d. Bone contusion anterior aspect of the medial femoral condyle ?F usually lateral femoral condyle, but can occur in anterior condyles following hyperextension injury

e. Posterior horn medial meniscus tear ?T (LM torn > MM)
* LW: Still possible, so hence can be associated with ACL injury: In acute ACl injury lateral meniscal tear > > than medial, while in chronic ACL injury, both medial and lateral occur near similar frequency. Conversely, Lateral meniscal tear was commonly associated with acute ACL injury, while medial meniscal tear with chronic ACL injury. (https://www.ncbi.nlm.nih.gov/pubmed/26286641)
* O’Donoghue’s unhappy triad = ACL tear, MM tear and MCL tear. Agree with above comments regarding lateral being more common though (lateral joint gets squashed in pivot-shift)

f. ? Concavity of the anterior ligamentous border

Associated abnormalities – ACL tear (StatDx)
•	Marrow oedema/contusions:
o	Posterolateral corner tibia
o	Lateral femoral condyle
o	+/- posteromedial tibial condyle
•	Bone trabecular injuries or impaction fractures of the posterolateral tibia and weight-bearing surface of lateral femoral condyle (sulcus)
•	Meniscal tears (lateral greater than medial)
•	Posterolateral corner injuries
o	Lateral collateral ligament
o	Arcuate ligament
o	Popliteus tendon
o	Posterolateral capsule
o	Popliteofibular ligament
How well did you know this?
1
Not at all
2
3
4
5
Perfectly
51
Q

50) 30yo male with anterior right shoulder injury. MRI arthrogram shows a contrast cleft between the anteroinferior glenoid labrum and bony glenoid margin. MOST LIKELY explanation?
a. SLAP
b. Bankart
c. Sublabral foramen
d. Buford complex
e. Perthes

A

ANSWER: b. Bankart T = Tear of the anteroinferior glenoid labrum with torn anterior scapular periosteum. May have an associated fracture of the anteroinferior glenoid rim.

30yo male with anterior right shoulder injury. MRI arthrogram shows a contrast cleft between the anteroinferior glenoid labrum and bony glenoid margin. MOST LIKELY explanation?

a. SLAP = F = Superior labrum tear propagating anterior and posterior to the biceps anchor
b. Bankart T = Tear of the anteroinferior glenoid labrum with torn anterior scapular periosteum. May have an associated fracture of the anteroinferior glenoid rim

.c. Sublabral foramen F = absent fixation of labrum at 1-3 o’clock

d. Buford complex F = absent anterosuperior labrum with thickened MGHL
e. Perthes = F = Detached inferior glenohumeral ligamentous complex with intact scapular periosteum, which is stripped medially.

Minimally-displaced, avulsed anteroinferior labrum; IGHL is non-displaced in neural positioning, but will displace with ABER positioning (“non-displaced Bankart”). ⇒ NB another recalled “contrast cleft b/w anteroinferior labrum & glenoid cartilage” that was “not displaced” – in this case the answer would be E PERTHES lesion.

How well did you know this?
1
Not at all
2
3
4
5
Perfectly
52
Q
  1. MRI knee, which is false?

a. Normal ACL is perpendicular to Blumenstaadts line
b. Tears of the posterior horn of the MM are more common than the anterior horn
c. The lateral facet of the patella is typically larger than the medial facet
d. Normal PCL is subject to “magic angle” effect
e. MCL and ACL disruption are associated

A

ANSWER:a. Normal ACL is perpendicular to Blumenstaadts line F Parallel

  1. MRI knee, which is false?
    a. Normal ACL is perpendicular to Blumenstaadts line F Parallela. Normal ACL is perpendicular to Blumenstaadts line F Parallel
    b. Tears of the posterior horn of the MM are more common than the anterior horn T
    c. The lateral facet of the patella is typically larger than the medial facet T
    d. Normal PCL is subject to “magic angle” effect T In the magic angle phenomenon, increased signal intensity may be present on the upward sloping portion of the PCL on short TE images, mimicking a tear. It is present in anatomic components of the ligament oriented 55° to the main static magnetic field, along the long axis of the magnetic bore. The phenomenon can be distinguished from a true PCL tear using long TE-weighted imaging sequences. When using proton-density imaging, the artifact may persist if the TE is 20 milliseconds or less. If the abnormal signal focus persists on T2 (long TE), a true PCL abnormality is present. In knee can also occur in posterior horn of lateral meniscus
    e. MCL and ACL disruption are associated T as part of O’Donoghue’s triad
How well did you know this?
1
Not at all
2
3
4
5
Perfectly
53
Q
  1. Which of the following statements regarding congenital tarsal coalition is FALSE?
    a. Calcaneo-navicular coalition is readily detectable on oblique radiographs
    b. The condition is bilateral in 50%
    c. 90% include the talo-calcaneal or calcaneo-navicular joint
    d. Talo-calcaneal coalition are best seen on axial CT
A

ANSWER:d. Talo-calcaneal coalition are best seen on axial CT F coronal.

  1. Which of the following statements regarding congenital tarsal coalition is FALSE?
    a. Calcaneo-navicular coalition is readily detectable on oblique radiographs T best seen on oblique view as direct connection (bony coalition) or close proximity & irregularity of the joint margins (fibrous coalition)
    b. The condition is bilateral in 50% T bilateral in 50%
    c. 90% include the talo-calcaneal or calcaneo-navicular joint T
    d. Talo-calcaneal coalition are best seen on axial CT F coronal.
    e. Calc-nav coalition is best visualised on sagittal T2W MRI. T calcaneonavicular coalitions best visualized on sagittal & axial MRI imageshttp://radiographics.rsna.org/content/20/2/321.full (tarsal coalition RG 2000 – direct from)
How well did you know this?
1
Not at all
2
3
4
5
Perfectly
54
Q
  1. Which of the following statements regarding cortical desmoids is MOST correct?

a. Occur only on the posterior medial epicondyle of the femur
b. Occur only on the posterior lateral epicondyle of the femur
c. Occur on the posterior cortex of the
d. They do not exhibit periosteal new bone
e. Biopsy is the only certain method of diagnosis

A

Cortical desmoid is a benign shallow cortical irregularity seen in older children or adolescents. It is recognised at specific metaphyseal sites, the most common being the medial posterior aspect of the distal femoral metaphysis at the insertion of the adductor magnus aponeurosis.

How well did you know this?
1
Not at all
2
3
4
5
Perfectly
55
Q
  1. Pediatric foot, which of the following statements is incorrect?
    a. ‘Talipes’ refers to congenital foot deformity
    b. ‘Pes’ refers to acquired foot deformity
    c. Valgus refers to distal foot angulation medially
    d. ‘Equinus’ refers to plantar flexion of the calcaneus
    e. ‘Clubfoot’ refers to Talipes equinovarus
A

ANSWER:c. Valgus refers to distal foot angulation medially F = valgus means lateral angulation

  1. Pediatric foot, which of the following statements is incorrect?

a. ‘Talipes’ refers to congenital foot deformity T
b. ‘Pes’ refers to acquired foot deformity ??T (pes = foot)
c. Valgus refers to distal foot angulation medially F = valgus means lateral angulation
d. ‘Equinus’ refers to plantar flexion of the calcaneus T
e. ‘Clubfoot’ refers to Talipes equinovarus T

How well did you know this?
1
Not at all
2
3
4
5
Perfectly
56
Q
  1. 8yo boy presents to DEM complaining of severe pain after a fall while playing soccer. On examination – marked pain and swelling over wrist. Xrays of wrist, which is the LEAST common expected radiographic finding?

a. Fracture of the scaphoid waist
b. Fracture of the ulnar styloid process
c. Salter Harris 2 fracture of the distal radius
d. Dislocation of the distal radio-ulnar joint

A

ANSWER:a. Fracture of the scaphoid waist F uncommon at this age.

  1. 8yo boy presents to DEM complaining of severe pain after a fall while playing soccer. On examination – marked pain and swelling over wrist. Xrays of wrist, which is the least common expected radiographic finding?
    a. Fracture of the scaphoid waist F uncommon at this age.
    b. Fracture of the ulnar styloid process
    c. Salter Harris 2 fracture of the distal radius
    d. Dislocation of the distal radio-ulnar joint

Paediatric fractures about the wrist (Donnelly & Handbook of Fractures)
• Most common fracture site in children
• Most fractures of the distal forearm are buckle or transverse fractures of the distal radial metaphysis +/- fracture of the distal ulnar metaphysis
• The distal radius is the most common area of physeal (SH) fracture (28% of physeal injuries occur in the distal radius)
• Carpal injuries are very rare before the teenage years (injury force transmitted to distal radial physis rather than carpus). Injuries to the scaphoid are extremely rare in the 1st decade.Rule of thumb – wrist fractures
• 5 years = torus
• 15 years = Salter-Harris of DR
• 25 years = scaphoid #
• 45 years = Colles #

How well did you know this?
1
Not at all
2
3
4
5
Perfectly
57
Q
  1. Which of the following is LEAST LIKELY to be present in association with lateral patella dislocation?
    a. Patella alta
    b. Posterolateral corner injury
    c. Bone oedema in the lateral femoral condyle
    d. Bone oedema in the medipatella
    e. Hemarthrosis
A

b. Posterolateral corner injury

How well did you know this?
1
Not at all
2
3
4
5
Perfectly
58
Q
  1. A 64 yo woman sees her GP with an 8 month history of bony pain. Investigations indicate hyperparathyroidism. The GP contacts you for advice regarding the next most appropriate investigation?
    a. US and sestamibi scan
    b. Thyroid and neck US
    c. Parathyroid sestamibi
    d. Surgical neck exploration
    e. CT neck and chest
A

ANSWER:a. US and sestamibi scan T if primary hyperparathyroidism & minimally-invasive surgery considered.

  1. A 64yo woman sees her GP with an 8 month history of bony pain. Investigations indicate hyperparathyroidism. The GP contacts you for advice regarding the next most appropriate investigation?ANSWER:
    a. US and sestamibi scan T if primary hyperparathyroidism & minimally-invasive surgery considered
    b. Thyroid and neck US
    c. Parathyroid sestamibi
    d. Surgical neck exploration T if minimally-invasive surgery not being considered
    e. CT neck and chest

WA Imaging Pathways
• Imaging for pre-operative localisation of the parathyroid glands remains controversial although it is generally recommended for minimally invasive or unilateral neck surgery.
• Ultrasound and Sestamibi scans are sensitive methods used as first line investigations, frequently in combination.
• CT and MRI have the advantage of superior anatomical localisation but are usually reserved for equivocal or negative ultrasound and nuclear medicine studies
.• Preoperative imaging is required for recurrent or persistent hyperparathyroidism to minimise the risks of repeat surgery and to maximise the chances of succesful treatment.

How well did you know this?
1
Not at all
2
3
4
5
Perfectly
59
Q
  1. 65yo female. Acute onset knee pain after walking dog. MRI obtained within 1 week later demonstrates intense marrow oedema in the medial femoral condyle. Which of the following is the MOST LIKELY associated MRI finding?
    a. Displaced bucket handle tear of the medial meniscus
    b. Medial collateral ligament tear
    c. Radial tear medial meniscus
    d. Large chondral defect
    e. Intra-articular loose body
A

ANSWER: NOT SURE PROBABLY D:
Large chondral defect ?T with subchondral insufficiency fracture (SONK), however with this condition the overlying articular cartilage is often intact

  1. 65yo female. Acute onset knee pain after walking dog. MRI obtained within 1 week later demonstrates intense marrow oedema in the medial femoral condyle. Which of the following is the MOST LIKELY associated MRI finding?
    a. Displaced bucket handle tear of the medial meniscus F usually adolescents & young active adults
    b. Medial collateral ligament tear F would cause lateral knee contusions
    c. Radial tear medial meniscus ?T middle-aged to older adult; often caused by acute trauma with axial loading; large radial tears may be a/w marrow oedema
    d. Large chondral defect ?T with subchondral insufficiency fracture (SONK), however with this condition the overlying articular cartilage is often intact
    e. Intra-articular loose body
How well did you know this?
1
Not at all
2
3
4
5
Perfectly
60
Q
  1. Which of the following is NOT associated with ACL tears?

a. Bone contusion postero-lat tibial plateau
b. Bone contusion at anterior aspect of med femoral condyle
c. Meniscocapsular separation of the posterior horn of MM
d. MCL injury
e. Ilio-tibial band avulsion

A

ANSWER:b. Bone contusion anterior aspect of the medial femoral condyle ?F usually lateral femoral condyle; bone contusions can occur in anterior condyles following hyperextension injury

  1. Which of the following is NOT associated with ACL tears?
    a. Bone contusion of the posterolateral tibial plateau T standard is posterolateral corner tibia, lateral femoral condyle +/- posteromedial tibial condyle
    b. Bone contusion anterior aspect of the medial femoral condyle ?F usually lateral femoral condyle; bone contusions can occur in anterior condyles following hyperextension injury
    c. Meniscocapsular separation of the posterior horn of the medial meniscus T meniscocapular separation assoc/ w/ ACL &/or MCL tears (StatDx)
    d. MCL injury T (although LM torn > MM)
    e. Ilio-tibial band avulsion T Gerdy tubercle avulsion (anterolateral tibial plateau), RG 08.

Some say Segond fracture is ITB avulsion – StatDx says Segond fracture is avulsion of lateral capsular ligament at lateral tibial rim cortex, which can involve ITB more anteriorly.

Associated abnormalities – ACL tear (StatDx
)•	Marrow oedema/contusions:
o	Posterolateral corner tibia
o	Lateral femoral condyle 
o	+/- posteromedial tibial condyle
•	Bone trabecular injuries or impaction fractures of the posterolateral tibia and weight-bearing surface of lateral femoral condyle (sulcus)
•	Meniscal tears (lateral greater than medial)
•	Posterolateral corner injuries
o	Lateral collateral ligament
o	Arcuate ligament
o	Popliteus tendon
o	Posterolateral capsule
o	Popliteofibular ligament

Imaging 2003
There are a number of associated injuries involving other structures which are quite specific for ACL tear. At the time of injury there is usually valgus strain, external rotation and anterior translation of the tibia relative to the femoral condyle which frequently results in an impaction injury of the posterior lip of the lateral tibial plateau against the femoral condyle [7]. The typical appearance of this injury on MRI is oedema in the posterior portion of the plateau, occasionally accompanied by a small fracture of the posterior lip, and a focal osteochondral impaction fracture of the mid portion of the lateral femoral condyle (Figure 13 ). The latter may result in a loose body. The combination of these two injuries is diagnostic of ACL tear. Similar bony impaction may be seen on the medial side which is thought to be as result of rebound from the valgus force [8]. There are often associated meniscal tears, the most characteristic of which is a vertical circumferential tear of the posterior third of the lateral meniscus.

How well did you know this?
1
Not at all
2
3
4
5
Perfectly
61
Q
  1. Which of the following findings best correlates with osteomyelitis in the setting of Charcot foot?
    a. Intraarticular bodies
    b. Sinus tract in bone
    c. Subchondral cysts
    d. Thin rim of enhancement in relation to joint effusion
    e. Joint effusion
A

b. Sinus tract in bone T

How well did you know this?
1
Not at all
2
3
4
5
Perfectly
62
Q
  1. Which is not a feature of rickets?
    a. Scoliosis
    b. Fraying and cupping of metaphyses
    c. Dental abnormalities
    d. Premature closure of anterior fontanels
    e. Genu valgus
A

ANSWER:d. Premature closure of anterior fontanels F (often delayed closure), JAOA 2002

  1. Which is not a feature of rickets?
    a. Scoliosis T
    b. Fraying and cupping of metaphyses T
    c. Dental abnormalities T
    d. Premature closure of anterior fontanels F (often delayed closure), JAOA 2002
    e. Genu valgus T usually genu varus (bow legs) but can be genu valgus (esp. in hypophosphataemic rickets – Wheeless, orthonet)
How well did you know this?
1
Not at all
2
3
4
5
Perfectly
63
Q
  1. 80yo female with Hx left THR 5yo ago. Left hip pain for 10 months. Mildly increased CRP / ESR. Xray shows periprosthetic sclerosis and lucency. Bone scan shows increased tracer uptake around femoral component of prosthesis. Infection vs loosening, best investigation is?

a. Repeat bone scan 3/12 following antibiotic therapy
b. Left hip arthrography and aspiration
c. Contrast enhanced hip MRI
d. CT hip
e. Radioisotope labelled white cell scan or Gallium scan.

A

ANSWER:b. Left hip arthrography and aspiration T
Most diagnostic

**LJS - aspiration yes but ?not arthogram. Would be USS guided aspiration only, without imaging of joint I think
Orthobullets says aspiration is diagnostic, if can’t dx on aspiration or bloods = NM scan
In-111 labelled WBC scan combined with suphur colloid is the investigation of choice in this scenario.

**expert orthopaedic SMO opinion - aspiration, but increasing evidence for NM studies (*A pre-part 2 ortho reg also agrees with this)

ADDIITIONAL INFO

e. Radioisotope labeled white cell scan or Gallium scan. F Mettler: generalized ↑ activity around hip prosthesis on MDP bone scan may be indicative of OM. A normal Gallium scan effectively excludes OM. If the Galium & MDP bone scan distributions are spatially incongruent, or if they are spatially congruent but gallium activity exceeds technetium activity, OM should be considered. An infected joint replacement is more specifically diagnosed by comparing an 111-Indium or 99mTc labeled leucocyte images with a technetium colloid marrow (substraction) scan – when there is periprosthetic leucocyte accumulation without corresponding marrow activity on the colloid images, the study is positive for infection.

How well did you know this?
1
Not at all
2
3
4
5
Perfectly
64
Q
  1. 24yo athlete. Worsening hip pain and groin pain, brought on by running, relieved by rest. Now limiting her running. GP verifies a ‘click’ over right greater trochanter on clinical examination and appropriate stress maneuvers. Plain radiographs normal. MOST LIKELY?
    a. Snapping gluteus maximum
    b. Femoral head AVM
    c. Trochanteric bursitis
    d. Osteochondritis dessicans
    e. Rectus femoris tear
A

ANSWER:a. Snapping gluteus maximus T (see below)a. Snapping gluteus maximus T (see below)

  1. 24yo athelete. Worsening hip pain and groin pain, brought on by running, relieved by rest. Now limiting her running. GP verifies a ‘click’ over right greater trochanter on clinical examination and appropriate stress maneuvers. Plain radiographs normal. MOST LIKELY?
    a. Snapping gluteus maximus T (see below)
    b. Femoral head AVM F – no snapping
    c. Trochanteric bursitis F but can occur with/as a result of the snapping hip
    d. Osteochondritis dessicans F
    e. Rectus femoris tear F

Apley Orthopaedics‘Snapping hip’ is a disorder in which the patient (usually a young woman) complains of the hip ‘jumping out of place, or ‘catching’, during walking. The snapping is caused by a thickened band in the gluteus maximus aponeurosis flipping over the greater trochanter. EmedicineSnapping hip syndrome is characterized by an audible snap or click that occurs in or around the hip. This syndrome is well recognized but poorly understood. Snapping hip syndrome may be due to an external cause (eg, snapping of the iliotibial band or gluteus maximus over the greater trochanter) or an internal cause (eg, snapping of the iliopsoas tendon over the iliopectineal eminence, acetabular labral tear, intra-articular loose body). Acetabular labral tears and intra-articular loose bodies are relatively uncommon causes of internal snapping hip syndrome

How well did you know this?
1
Not at all
2
3
4
5
Perfectly
65
Q
  1. 30yo male with anterior right shoulder pain. MRI arthrogram shows a contrast cleft between the anteriosuperior glenoid labrum and bony glenoid margin. MOST LIKELY explanation?
    a. Buford complex
    b. Labral tear
    c. Sublabral foramen
    d. SLAP tear
A

c. Sublabral foramen sublabral sulcus/recess at 12 o’clock at site of biceps tendon attachment; sublabral foramen (= sublabral hole) 2 o’clock position ⇒ the two may co-exist

How well did you know this?
1
Not at all
2
3
4
5
Perfectly
66
Q
  1. AVN of femoral head, which is TRUE?

a. Posterior position of femoral head more commonly affected than anterior portion
b. Bone changes on both side of joint occur in the majority
c. Early loss of articular cartilage
d. Subchondral lucent line in femoral head is early sign on plain radiographs
e. MRI is more sensitive for the Dx than Tc99m scintigraphy

A

ANSWER:e. MRI is more sensitive for the Dx than Tc99m scintigraphy T MRI 98% sensitive & 85% specific; bone scan with SPECT is 85% sensitive

  1. AVN of femoral head, which is TRUE?
    a. Posterior position of femoral head more commonly affected than anterior portion F anterior weight bearing portion of femoral head early
    b. Bone changes on both side of joint occur in the majority F AVN is not a primary articular process – joint remains intact until secondary OA occurs
    c. Early loss of articular cartilage F cartilage intact until late (secondary OA
    d. Subchondral lucent line in femoral head is early sign on plain radiographs F early sign is relative sclerosis of femoral head; intermediate timing for subchondral fracture line (crescent sign)
    e. MRI is more sensitive for the Dx than Tc99m scintigraphy T MRI 98% sensitive & 85% specific; bone scan with SPECT is 85% sensitive
How well did you know this?
1
Not at all
2
3
4
5
Perfectly
67
Q
  1. Regarding erosive osteoarthritis, FALSE?

a. Distribution tends to be symmetrical
b. Condition characterised by synovitis superimposed on changes of degeneration
c. Tends to be hereditary
d. Predominantly affects middle aged women
e. MCP joints are most frequently affected

A

ANSWER:e. MCP joints are most frequently affected F identical distribution to non-inflammatory OA (distal > proximal joints, i.e. DIPJs & PIPJs)

  1. Regarding erosive osteoarthritis, FALSE?
    a. Distribution tends to be symmetrical T
    b. Condition characterised by synovitis superimposed on changes of degeneration T
    c. Tends to be hereditary T some evidence in general for a role in OA
    d. Predominantly affects middle aged women T? post-menopausal women (Dahnert)
    e. MCP joints are most frequently affected F identical distribution to non-inflammatory OA (distal > proximal joints, i.e. DIPJs & PIPJs)

The anatomic distribution of involvement in primary osteoarthritis and erosive osteoarthritis is the same: DIP, PIP, and first CMC joints, generally with sparing of the other joints of the hand and wrist. This pattern of distribution is characteristic and, in well-established disease, differentiates osteoarthritis from inflammatory forms of arthritis such as rheumatoid arthritis, psoriatic arthritis,” and septic arthritis and from crystal-mediated forms of arthritis such as pyrophosphate arthropathy and gouty arthritis.

How well did you know this?
1
Not at all
2
3
4
5
Perfectly
68
Q

67.64 year old male presents with worsening back pain. Otherwise well. Normal examination. Sclerotic lesion T8 vertebral body without loss in height. The most appropriate next investigation is:

  1. Skeletal survey
  2. CT T-spine
  3. MRI T-spine
  4. Bone scan
  5. No further investigation required at this stage
A
  1. Bone scan - T - would evaluate if solitary / multifocal, and if active.
  2. 64 year old male presents with worsening back pain. Otherwise well. Normal examination. Sclerotic lesion T8 vertebral body without loss in height. The most appropriate next investigation is: (GC) answer:
  3. Skeletal survey - F
  4. CT T-spine - F - need to image whole spine, whereby MRI would give more information regarding discs etc. as other potential causes for back pain.
  5. MRI T-spine - F - need to image whole spine.
  6. Bone scan - T - would evaluate if solitary / multifocal, and if active.
  7. No further investigation required at this stage - F - at least check PSA.

Bone tumours favouring vertebral bodies:
CALL HOME
Chordoma, ABC, Leukaemia, Lymphoma, Haemangioma, Osteoid osteoma/Osteoblastoma, Myeloma/Mets, EG.

DDx Ivory vertebra (sclerotic vertebra)
• Infection (↓ disc space height & endplate changes)
• Metastasis (blastic) – breast, prostate, carcinoid
• Lymphoma (Hodgkin>NHL; paraspinal mass)
• Paget disease (trabeculated)
• Mastocytosis (small bowel thickening; hepatosplenomegaly)
• Haemangioma (trabeculated)
• Vertebroplasty

Ddx widespread sclerotic lesions: 
metastases - prostate, (breast), lung, bladder, pancreas, stomach, colon, carcinoid, brain 
Paget's disease 
sarcoma 
myelofibrosis
mastocytosis    [Dahnert]
How well did you know this?
1
Not at all
2
3
4
5
Perfectly
69
Q
  1. Multiple crush fractures in a patient with known osteopenia. Pain. The fractures at T8 and T11 levels are hot on a bone scan. Which of the following is the most correct explanation
  2. Acute or malunited fractures
  3. Metastases
  4. Lymphoma
  5. Myeloma
  6. Healed fractures
A
  1. Acute or malunited fractures - hot on all 3 phases
  2. Multiple crush fractures in a patient with known osteopenia. Pain. The fractures at T8 and T11 levels are hot on a bone scan. Which of the following is the most correct explanation (TW/SK) answer:
  3. Acute or malunited fractures - hot on all 3 phases
  4. Metastases – could be pathologic fractures; most mets show increased uptake
  5. Lymphoma – lymphoma does have increased uptake +/- central necrosis causing photopenia
  6. Myeloma – F – positive in 75% of patients & 24-55% of sites (Dahnert)
  7. Healed fractures
Hot bone lesions:: 
Infection, 
Neoplasm, 
Trauma, 
autoimmune 
Arthropathy (e.g. Charcot), , 
Aseptic Necrosis  

Fracture on bone scan (Mettler p270)
• Hot at 1-3 days
• Returns to normal usually by 1 year, may take 3 years

Emedicine: Myeloma is a disease that results in overactivity of osteoclasts, with resultant liberation of bone and suppression of osteoblasts. Nuclear medicine bone scans rely on osteoblastic activity (bone formation) for diagnosis. As such, standard technetium-99m (99m Tc) bone scans have underestimated the extent and severity of disease and should not be used routinely.[20]

How well did you know this?
1
Not at all
2
3
4
5
Perfectly
70
Q
  1. Which of the following is false regarding SLE?
  2. Erosions
  3. Calcification of the periarticular soft tissues
  4. Subluxation
  5. Juxta-articular osteoporosis
  6. Osteonecrosis
A

answer: 1.Erosions - F - subluxation and/or dislocation without erosive disease is the hallmark of SLE
3. Which of the following is false regarding SLE? (TW)
1. Erosions - F - subluxation and/or dislocation without erosive disease is the hallmark of SLE
2. Calcification of the periarticular soft tissues - T - calcification may be present in the subcutaneous tissue in SLE. (MSK Req – 10%, linear/streaky/nodular/amorphous)
3. Subluxation - T - see 1.
4. Juxta-articular osteoporosis - T
5. Osteonecrosis - T - said to occur in 6-40% of pts with SLE. Usually occurs bilaterally and asymetrically. Femoral heads, humeral heads, femoral condyles, and tibial plateaus, and tali are most common sites.

How well did you know this?
1
Not at all
2
3
4
5
Perfectly
71
Q

70.Which of the following is false regarding chordomas?

  1. Spinal metastases
  2. Destructive lesion with soft tissue
  3. Less than 30 years old
  4. Occurs in both ends of the spine
A

*LW:
Assuming spinal means osseous spine mets, then preferred answer is C - less than 30yrs old, given the majority occur in patients 30-70yrs

Previous answer:
answer:1.Spinal metastases – T - metastases (in 5-43%) to liver, lung, bone, regional lymph node, peritoneum, skin (late) heart.SK – depends what “spinal means” – with chordoma can get intradural extramedullary & bony metastases, but spinal cord metastases extremely rare

  1. Which of the following is false regarding chordomas? (TW/SK) answer:
  2. Spinal metastases – T - metastases (in 5-43%) to liver, lung, bone, regional lymph node, peritoneum, skin (late) heart.SK – depends what “spinal means” – with chordoma can get intradural extramedullary & bony metastases, but spinal cord metastases extremely rare
  3. Destructive lesion with soft tissue - T - lobulated tumor contained within pseudocapsule. Most frequent radiographic appearance of chordoma is that of a destructuve lesion of a vertebral body centered in the midline, with a large, associated soft tissue mass. SK lytic, destructive lesion – may involve disc, contiguous vertebrae, epidural space, etc.
  4. Less than 30 years old – F?? - can occur at any age, but mainly 30-70yo (mean 50yo). M>F 2:1. SK – StatDx = generally > 30 years, extremely rare in patients < 20 yrs; < 5% of chordomas present in childhood; young patients tend to occur in clivus
  5. Occurs in both ends of the spine - T - 50% in sacrum, 35% in clivus, 15% in vertebrae. Chordoma is the most common primary malignant tumor of the spine in adults excluding lymphproliferative neoplasms. Originates from embryonic remnants of notochord / ectopi cordal foci (Hensens node / primitive knot). Histo - cords & clusters of large bubblelike vacuolated (physaliferous) cells.
How well did you know this?
1
Not at all
2
3
4
5
Perfectly
72
Q

71.Which of the following is more suggestive of tuberculous rather than pyogenic infection of the spine?

  1. Multifocal
  2. Low signal on T1 and T2
  3. Disc space narrowing
  4. Subligamentous spread
  5. Normal chest xray rules it out
A

answer: 4.Subligamentous spread - T - infection spreads beneath the ALL or PLL to adjacent vertebrae; may see skip lesions. Pyogenic infection spreads contiguously involving disc and subchondral bone; begins in disc in kids (highly vascularised); in adults begins in endplate with secondary disc invovlement.
5. Which of the following is more suggestive of tuberculous rather than pyogenic infection of the spine? (GC) Primer p586, MSK Req p558
1. Multifocal - F - In TB there is typically more than one (up to 5-10) vertebrae involved due to subligamentous spread; upper lumber + lower thoracic (L1 most common). Pyogenic infection may involve multiple levels in 20% (esp. immunocompromised).
2. Low signal on T1 and T2 - F - non discriminating feature of spondylitis - low SI of marrow on T1, contrast enhancement of marrow +/- disc, high SI of disc (+/- marrow) on T2.
3. Disc space narrowing - F - relative preservation of disc space because TB lacks proteolytic enzymes; disc itself is preserved but fragmented (cf. rapid destruction in pyogenic infection).
4. Subligamentous spread - T - infection spreads beneath the ALL or PLL to adjacent vertebrae; may see skip lesions. Pyogenic infection spreads contiguously involving disc and subchondral bone; begins in disc in kids (highly vascularised); in adults begins in endplate with secondary disc invovlement.
5. Normal chest xray rules it out - F - no pulmonary lesions in 50%. Other features of TB spondylitis: almost always bone destruction evident at the time of imaging, rather than just marrow oedema gibbus deformity due to preferential anterior involvement in adults posterior elements often involvedepidural and paraspinous abscesses are common and large at time of presentation; psoas abscess may be calcified absence of reactive sclerosis or periosteal reaction vertebra plana in kids; vertebra within a vertebra, ivory vertebra [Dahnert; Kaplan; Castillo’s notes] Changed option 3 from “disc space narrowing/involvement late” so only one true answer.

How well did you know this?
1
Not at all
2
3
4
5
Perfectly
73
Q
  1. Regarding a destructive sacral lesion, the least likely cause would be:
  2. Osteoblastoma
  3. Giant cell tumour
  4. Chordoma
  5. Chondrosarcarcoma
A

answer:

  1. Osteoblastoma - rarely found in sacrum (Dahnert – very rare). May be blastic (large osteoid osteoma) or expansile & lytic (similar to ABC). Tends involve the posterior vertebral elements.
    * LW statDx states 15-20% osteoblastomas in sacrum, tend to be young 2nd-3rd decade, thus if age was in question stem this would likely confirm this as the least likely cause.
  2. Regarding a destructive sacral lesion, the least likely cause would be: (GC)
  3. Osteoblastoma - rarely found in sacrum (Dahnert – very rare). May be blastic (large osteoid osteoma) or expansile & lytic (similar to ABC). Tends involve the posterior vertebral elements.
  4. Giant cell tumour - T - only 7% of GCTs involve the spine, but with respect to spinal involvement, the sacrum is the most common site. 2nd-4th decade. Locally aggressive, eccentric; involves subchondral bone, may grow across SIJ. 5-10% are malignant. SK – usually no mineralisation (purely lytic); heterogeneous due to haemorrhage, necrosis and fibrous tissue.; 2nd most common primary sacral tumour after chordoma; locally aggressive;
  5. Chordoma - T - arise from notochordal rests, therefore always midline/paramedian in relation to spine. Most common primary sacral malignancy (excl. lymphoproliferative), mostly 4th-7th decades, 50-60% in sacrum (35% in clivus). Locally aggressive, amorphous calcifications, may cross SIJ. SK - most common primary sacral malignancy; > 70% have intratumoural calcification; T1 & T2 hetero; 70% in sacrum have T2 low signal foci of haemosiderin.
  6. Chondrosarcoma - T - lytic lesion with assocd soft tissue mass and calcifications; adults. Will have chondroid matrix calcification.
  7. Plasmacytoma - T lytic, destructive lesions

Differential for destructive sacral lesion:- tumour: -> chordoma (1st), GCT (2nd), –> met, myeloma, lymphoma, leukaemia-> ewing, neuroblastoma- infection

How well did you know this?
1
Not at all
2
3
4
5
Perfectly
74
Q
  1. Which of the following is the least specific regarding atlanto-occipital instability?
  2. Occipital condyle fracture
  3. > 12mm basion dens distance
  4. > 12mm distance from basion to posterior axial line
  5. Extradural haemorrhae along the anterior aspect of the spinal canal
  6. Flexion/extension views and survival of the fittest!
A
  • LW: per same question with LJS detailed logic, in the setting of trauma, flexion extension views are least specific.
    answer: 4.Extradural haemorrhae along the anterior aspect of the spinal canal
    7. Which of the following is the least specific regarding atlanto-occipital instability? (GC)
    1. Occipital condylar fracture - T - classified as impaction fractures, extensions of occipital skull fractures or avulsion fractures at the insertion of the alar ligaments. The latter are potentially unstable fractures, esp. if displaced, and when assocd with tectorial memebrane injury can result in gross atlanto-occipital discontinuity. May be unilateral or bilateral, may extend in a ringlike pattern around the foramen magnum, and are extemely difficult to identify on xray. May have a lower CN palsy (most commonly CNXII due to fracture extension into hypoglossal canal).
    2. >12mm basion-dens distance - T
    3. >12mm distance from basion to posterior axial line - T - anteriorly, see below.
    4. Extradural haemorrhage along the anterior aspect of the spinal canal - non specific.
    5. Flexion / extension views and survival of the fittest! - T

Normal dimensions of the CC junction at lateral xray:
Basion-dens interval <12mm
Basion-posterior axial line interval <12mm posterior to basion; <4mm anterior to basion
Prevertebral soft tissues <6mm at C2, flat or concave
Anterior atlanto-dens interval <2mm
Lateral atlanto-dens interval <2-3mm side-to-side difference
Atlanto-occipital articulation 1-2mm
Atlantoaxial articulation 2-3mm
[Radiologic spectrum of CC distraction injuries, RG 2000]

How well did you know this?
1
Not at all
2
3
4
5
Perfectly
75
Q
  1. 40 year old male with lower back pain. Xray shows a destructive sacral lesion. Which of the following is the least correct possibility?
  2. Chordoma
  3. Plasmacytoma
  4. Giant cell tumour
  5. Chondrosarcoma
  6. Osteoblastoma
A
  1. 40 year old male with lower back pain. Xray shows a destructive sacral lesion. Which of the following is the least correct possibility? (GC) see question 33.
  2. Chordoma - T
  3. Plasmacytoma - T
  4. Giant cell tumour - T
  5. Chondrosarcoma - T
  6. Osteoblastoma - F - rarely found in sacrum.
    * LW: 15-20% of osteoblastomas occur in sacrum, however 90% occurr in teenagers - 20s, thus wrong age group.

Sacral destructive lesion- chordoma, GCT- met myeloma lymphoma- infection

How well did you know this?
1
Not at all
2
3
4
5
Perfectly
76
Q
  1. Which of the following is the most correct regarding rheumatoid arthritis?
  2. 20-40 peak age
  3. Osteopenia early in the course of the disease
  4. Pleural effusion late in the course of the disease
  5. Synovial tendon sheath spared
  6. Plain xray superior to MRI for detection of erosion defects
A

ANSWER:2.Osteopenia early in the course of the disease - T - early xray signs include fusiform soft tissue swelling, periarticular osteopaenia, widened joint space, subcortical cysts. Earliest changes are seen in MCP 2+3 and PIP 3; first erosion is classically the base of the proximal 4th phalanx (bare area).

  1. Which of the following is the most correct regarding rheumatoid arthritis? (GC)
  2. 20-40 peak age - F - affects 1% of population; 40-70yo. (5th to 8th decade). Females 3:1 if <40yo. (after 40, M=F).
  3. Osteopenia early in the course of the disease - T - early xray signs include fusiform soft tissue swelling, periarticular osteopaenia, widened joint space, subcortical cysts. Earliest changes are seen in MCP 2+3 and PIP 3; first erosion is classically the base of the proximal 4th phalanx (bare area).
  4. Pleural effusion late in the course of the disease - F - mostly unilateral, without change for months, usually not assocd with parenchymal disease.
  5. Synovial tendon sheath spared - F - tenosynovitis causes subluxations and rupture.
  6. Plain xray superior to MRI for detection of erosion defects - F - MR & US are the methods of choice in detecting early RA: synovial hyperaemia, synovial swelling, pannus, pre-erosive subcortical cysts, joint effusion, marrow oedema.
How well did you know this?
1
Not at all
2
3
4
5
Perfectly
77
Q
  1. Which of the following is more suggestive of a Charcot joint than osteomyelitis?
  2. Intraarticular loose body
  3. Sinus tract to bone
  4. Subchondral cyst
  5. Joint effusion
  6. Thin rim enhancement of fluid collection
A

ANSWER:1.Intraarticular loose body - T - six D’s: dense, degeneration, destruction, deformity, debris, dislocation. Loose bodies due to fragmentation of eburnated subchondral bone. Extensive osseous fragmentation is not an expected imaging finding in typical bacterial vertebral OM. The only thing that could potentially resemble a loose body in OM would be a sequestrum that may have fallen out of its cloaca…?

  1. Which of the following is more suggestive of a Charcot joint than osteomyelitis? (GC, TW)
  2. Intraarticular loose body - T - six D’s: dense, degeneration, destruction, deformity, debris, dislocation. Loose bodies due to fragmentation of eburnated subchondral bone. Extensive osseous fragmentation is not an expected imaging finding in typical bacterial vertebral OM. The only thing that could potentially resemble a loose body in OM would be a sequestrum that may have fallen out of its cloaca…?
  3. Sinus tract to bone - F - suggests OM.
  4. Subchondral cyst - F
  5. Joint effusion - T (but prob not most correct) - a persistent joint effusion is often the first sign of Charcot’s; joint distension may later occur due to fluid, hypertrophic synovitis, osteophytes, subluxation. Joint effusion can be seen in OM if infection spreads to joint (septic arthritis); usually involves a single joint.
  6. Thin rim enhancement of fluid collection - F - abscess, non-specific.
How well did you know this?
1
Not at all
2
3
4
5
Perfectly
78
Q
  1. Which of the following is the least correct regarding osteoarthritis?
  2. Reduced mineralization
  3. Erosions
  4. Unilateral / asymmetrical
  5. Subluxation
  6. Osteophytes
A

ANSWER:1.Reduced mineralization - F - bone density preserved; others include psoriasis, charcot’s, gout/pseudogout, sarcoidosis.

  1. Which of the following is the least correct regarding osteoarthritis? (GC/SK)
  2. Reduced mineralization - F - bone density preserved; others include psoriasis, charcot’s, gout/pseudogout, sarcoidosis.
  3. Erosions - T - may see erosions of SIJ, AC, TMJ, symphysis pubis (“SATS”). SK: Erosions occur in erosive OA. Erosions can occur in the “letter joints”, i.e. TMJ, AC & SI joints + the symph (Helms).
  4. Unilateral / asymmetrical - T - unilateral and/or bilateral asymmetrical distribution.
  5. Subluxation - T - eg. radial subluxation of 1st MC base, superolateral subluxation of femoral head (Dahnert)
  6. Osteophytes - T - at articular margin / non stressed area. Other OA features: nonuniform loss of joint space; subchondral sclerosis and cysts; distribution in hands, feet, knees, and hips; sparing of shoulders and elbows.[B&H, pg1133]
How well did you know this?
1
Not at all
2
3
4
5
Perfectly
79
Q

78.Regarding lumbar discs, which is the most correct?

  1. Annular tear is secondary to trauma
  2. Focal herniation < 25% of the disc
  3. Broad-based herniation >50% of the disc
  4. Far lateral disc at L4/5 level affects L5 nerve
  5. Posterolateral disc (paracentral) at L3/4 level affects L3 nerve
A

ANSWER:2.Focal herniation < 25% - T - localised herniation in the axial plane can be focal, ie less than 25% of the disk circumference or broad-based, meaning between 25 and 50% of the circumference. If it is >50% it is considered bulging, not herniation

  1. Regarding lumbar discs, which is the most correct? (JS)
  2. Annular tear is secondary to trauma - F - Disruption of concentric collagenous fibers comprising the anulus fibrosus. Abnormal signal focus (HIZ) at posterior disc margin on MRI. Direct association with disc degeneration, often due to repetitive trauma.
  3. Focal herniation < 25% - T - localised herniation in the axial plane can be focal, ie less than 25% of the disk circumference or broad-based, meaning between 25 and 50% of the circumference. If it is >50% it is considered bulging, not herniation
  4. Broad-based herniation >50% - F - see above
  5. Far lateral disc at L4/5 level affects L5 nerve - F - affects the L4 nerve which has already exited
  6. Posterolateral disc at L3/4 level affects L3 nerve - F - affects the L4 nerve root in the lateral recess
How well did you know this?
1
Not at all
2
3
4
5
Perfectly
80
Q

79.Male with anterior shoulder pain. MR arthrogram shows cleft between anterior superior labrum and bony glenoid margin. Which of the following is the most correct?

  1. Buford complex
  2. Bankart lesion
  3. SLAP lesion
  4. Superior labral foramen
  5. Perthe’s
A

*LW:
Based on stem info, i.e. just anterosuperior labrum cleft, and not stating extension posteriorly (beyond biceps insertion), I would favor this to represent a Superior labral foramen normal variant.
**LJS agree. *AJL also agree

Previous answer
SLAP LESION

  1. Male with anterior shoulder pain. MR arthrogram shows cleft between anterior superior labrum and bony glenoid margin. Which of the following is the most correct? (JS)
  2. Buford complex - Anatomical variant with absent anterosuperior labrum associated with a thickened cord like middle glenohumeral ligament. Seen in about 3% of the population.
  3. Bankart lesion - Refers to a complete labral tear at the origin of the inferior GH ligament, resulting in disruption of the scapular periosteum and detachment of the labrum from the glenoid rim. An osseous bankart lesion indicates an osteochondral fracture at this site
  4. SLAP lesion - Superior labrum anterior to posterior = seen in throwing athletes secondary to the pull of the long head of biceps which inserts into the superior labrum. A SLAP is best seen on the oblique coronal view and is irregular and extends sueriorly or laterally (cf sublabral recess which is thin and smooth and extends medially)
  5. Superior labral foramen - Anatomical variant which is a opening beneath the anterior superior labrum and bony glenoid that mimics a detachment. Seen in up to 20% of the population. SK: sublabral sulcus/recess at 12 o’clock at site of biceps tendon attachment; sublabral foramen (= sublabral hole) 2 o’clock position ⇒ the two may co-exist
  6. Perthe’s - Similar to Bankart where the inferior labral-ligamentous complex remains attached to the scapular periosteum which is stripped medially on the glenoid neck
How well did you know this?
1
Not at all
2
3
4
5
Perfectly
81
Q
  1. Young man with cleft between anterior inferior labrum and bony glenoid margin on MRI arthrogram. The most likely cause is
  2. Buford complex
  3. Bankart lesion
  4. SLAP lesion
  5. Superior labral foramen
A

BANKART

82
Q
  1. Which of the following is not associated with haemochromatosis
  2. Beaked osteophytes
  3. Symmetrical distribution
  4. Most marked 4th and 5th metacarpals
  5. Generalised osteoporosis is a rare finding
A

ANSWER: (hopefully incomplete recall given wording of options 3 and 4).

PROBABLY 3.
*LW: haemochromotosis causes beaked liked osteophytes radial aspect 2-5th MCPs, but most pronounced 2nd and 3rd, so agree this is likely the least correct.

(Previous answer)
Most marked 4th and 5th metacarpals F Most marked in 2nd & 3rd MCPJs [Dahnert, 6th ed. P101] Characteristic involvement of the 2nd & 3rd Metacarpals. I’ve put this as false as Burgener says Less commonly in the 1st, 4th and 5th metacarpals [Burgener 3rd P162] but this stem may be true it if it is the more unlikely between two stems, as in “most common”

  1. Which of the following is not associated with haemochromatosis (CC)
  2. Beaked osteophytes T this is a classic feature of haemochromatosis [Dahnert 6th ed.]
  3. Symmetrical distribution T Haemochromatosis is similar in distribution to CPPD, ie symmetrical distribution [www.learningradiology.com] and has uniform symmetric joint space narrowing [Burgener 3rd]
  4. Most marked 4th and 5th metacarpals F Most marked in 2nd & 3rd MCPJs [Dahnert, 6th ed. P101] Characteristic involvement of the 2nd & 3rd Metacarpals. I’ve put this as false as Burgener says Less commonly in the 1st, 4th and 5th metacarpals [Burgener 3rd P162] but this stem may be true it if it is the more unlikely between two stems, as in “most common”
  5. Generalised osteoporosis is a rare finding F?, Haemochromatosis is usually associated with generalised osteoporosis [D]. StatDx says normal bone density.
    * LW: Radiopedia states 25% osteoporosis, and 40% osteopenia….so unsure that seems common to me…
83
Q

83.Which of the following is not associated with gout

  1. The back is a common site of involvement
  2. Subchondral cysts
  3. Well defined erosions with sclerotic margins
A

ANSWER:1.The back is a common site of involvement F hands and feet are the most commonly affected. Involvement of hip & spine is rare (SI joints involed in 15%) [Dahnert 6th ed. P96] 2.Subchondral cysts T

  1. Which of the following is not associated with gout (CC)
  2. The back is a common site of involvement F hands and feet are the most commonly affected. Involvement of hip & spine is rare (SI joints involed in 15%) [Dahnert 6th ed. P96]
  3. Subchondral cysts T subchondral cysts/geodes are seen in gout [Dahnert 6th ed.]
  4. Well defined erosions with thin sclerotic margins T para-articular scalloped erosions with sclerotic margins are seen in gout
84
Q
  1. Which of the following is not associated with CPPD
  2. Erosions
  3. Uniform joint space loss
  4. Chondrocalcinosis
  5. Brown tumours
  6. Asymmetrical distrubution
A

*LW: Favoured answer is 5 - asymmetrical distribution.

  1. Erosions: true
  2. Uniform joint space loss: true
  3. Chondrocalcinosis: true
  4. Brown tumours: CPPD occurs with an increased frequency in primary hyperPTH, which can also cause Brown tumours, so the association is correct.
  5. Asymmetrical distrubution: false according to StatDx, states majority (2/3) shows a polyarticular and symmetrical distribution.

ANSWER:??? PROBABLY 4.Brown tumours - T - primary HPTH is assocd with CPPD; and brown tumours are more common in primary. SK – out of these, the least DIRECTLY associated with CPPD.

18.Which of the following is not associated with CPPD (CC, GC)

1.Erosions – T? - crystals cause DJD by eroding cartilage, however, true ‘erosions’ are not a feature. Fragmentation of an articular surface may result in a defect that can mimic a true erosion. Large subchondral cysts are a hallmark feature of CPPD.
See below for ddx erosions.SK – opinions:
• Dahnert – no
• StatDx – yes (“pressure” erosions from the crystals; 1/8 will show erosions)
• Requisites (Manaster) – early disease shows erosions, more advanced disease appears primarily productive
• Nivene Saad – yes

  1. Uniform joint space loss - T - uniform joint space loss is a feature; as is normal mineralisation.
  2. Chondrocalcinosis - T - polyarticular calcification in fibro- and hyaline cartilage
  3. Brown tumours - T - primary HPTH is assocd with CPPD; and brown tumours are more common in primary. SK – out of these, the least DIRECTLY associated with CPPD.

5.Asymmetrical distrubution - T - bilateral asymmetric distribution or symmetric, depending on the reference! (SK). bilateral symmetrical distribution of PIP & MCP jts occurs in 5% (pseudo-RA).
The distribution pattern for CPPD is usually bilateral, and may be symmetric or asymmetric. [gentilli.net]

Other characteristic features of CPPD:

knee: most commonly involved, 80% show chondrocalcinosis, preferential narrowing of patellofemoral joint space
- hand/wrist: esp. radiocarpal joint (stepladder configuration, chondrocalcinosis of the TFCC), SLAC, MCP 2+3, may have hook-like osteophytes (ddx haemochromotosis)
- non-weight bearing joints (unlike OA): shoulder, elbow- spine: multilevel degen disc changes, vacuum phenomenon, annulus fibrosis of L-spine, apophyseal jts, pubic symphysis.

85
Q
  1. Which of the following is not associated with osteoarthritis
  2. Osteoporosis
  3. Subluxation
  4. Scapho-trapeziotrapezoidal articulation complex
  5. Carpometacarpophalangeal joints
  6. Distal interphalangeal joints
A

NSWER:1.Osteoporosis - F, normal bone density is associated with osteoarthritis [MSK requisites P304]

  1. Which of the following is not associated with osteoarthritis (CC) A
  2. Osteoporosis - F, normal bone density is associated with osteoarthritis [MSK requisites P304]
  3. Subluxation - T, Subluxation can be found in a several joints, commonly hip joint, spine [MSK requisites P304]
  4. Scapho-trapeziotrapezoidal articulation complex - T, common location of OA [MSK, requisites P304]
  5. Carpometacarpophalangeal joints - T, common location of OA [MSK, requisites P304]
  6. Distal interphalangeal joints - T, common location of OA [MSK, requisites P304]
86
Q

86.Which of the following is not associated with DISH

  1. Flowing osteophytes
  2. Acetabular osteophytes
  3. Disc space narrowing
  4. Retinoid therapy
  5. Preferential fusion of the inferior portion of the SI joint.
A

ANSWER: 3 AND 5
3.Disc space narrowing – F. Disc spaces are relatively preserved, no apophyseal ankylosis.

  1. Preferential fusion of the inferior portion of the SI joint - F, Preferential fusion of the superior (non-synovial) part of SIJ and superior aspect of the symphysis pubis. Inferior SIJ fusion (synovial part) is seen in ankylosing spondylitis. [Dahnert 6th ed. P68]
  2. Which of the following is not associated with DISH T/F? (GC, TW)
  3. Flowing osteophytes - T, commonly found in DISH
  4. Acetabular osteophytes - T, broad osteophytes at the lateral acetabular edge are seen [Dahnert 6th ed. P68]
  5. Disc space narrowing – F. Disc spaces are relatively preserved, no apophyseal ankylosis.
  6. Retinoid therapy – T?, Retinoid therapy can result in similar appearance to DISH [MSK requisities]. SK – not really a cause of DISH, more a DDx.
  7. Preferential fusion of the inferior portion of the SI joint - F, Preferential fusion of the superior (non-synovial) part of SIJ and superior aspect of the symphysis pubis. Inferior SIJ fusion (synovial part) is seen in ankylosing spondylitis. [Dahnert 6th ed. P68]
87
Q

87.Which of the following is not associated with SLE

  1. Ulnar deviation
  2. Erosions
  3. may have positive antinuclear antibodies and rheumatoid factor
  4. Deformity is worse on Norgaard views
  5. Soft tissue calcification is occurs in 10% of cases.
A

ANSWER:2.Erosions F, erosions are not associated with SLE – a “non-erosive polyarthritis” [MSK requisites P328]

  1. Which of the following is not associated with SLE ? (CC)
  2. Ulnar deviation - T, ulna deviation and subluxation is commonly seen in SLE [MSK requisites P328]
  3. Erosions F, erosions are not associated with SLE – a “non-erosive polyarthritis” [MSK requisites P328]
  4. may have positive antinuclear antibodies and rheumatoid factor – T?, antinuclear antibodies (100%). Rheumatoid factor may be positive in SLE [MSK requisites P328] – can’t find this is Robbins or UTD though (SK)
  5. Deformity is worse on Norgaard views - T, Deformity is commonly worse on Norgaard views [MSK requisites P328]
  6. Soft tissue calcification is occurs in 10% of cases - T, soft tissue calcification occurs in only 10% of cases and is usually the lower extremity [MSK requisites P328]
88
Q

88.You are a rural radiologist. A 13 y.o obese boy comes in with left hip pain. L hip and knee XRs are normal. What would be the next most appropriate examination to perform?

  1. Bone scan
  2. MRI
  3. Repeat films in 10 days time
  4. AP pelvis and frog leg view
  5. Orthopaedic referral
A

4.AP pelvis and frog leg view

89
Q
  1. A 7 y.o girl comes in with hip pain and fever. Normal x-ray. What would be the next most appropriate examination to perform?
  2. Bone scan
  3. MRI
  4. Repeat films in 7 - 10 days time
  5. Bilateral Hip US
A

ANSWER: ULTRASOUND

23.A 7 y.o girl comes in with hip pain and fever. Normal x-ray. What would be the next most appropriate examination to perform? (CC)

  1. Bone scan
  2. MRI
  3. Repeat films in 7 - 10 days time
  4. Bilateral Hip US

Stem 4: The next investigation would be bilateral hip ultrasound to examine for effusions. Although more sensitive than plain radiographs in detecting effusions, a negative scan does not exclude septic arthritis. [Donnelly, Fund of Paed Rad P206]

90
Q

90.Which is false regarding MRI L-spine?

  1. Annular tears are usually traumatic in origin
  2. A far lateral disc at L4/5 impinges the exiting L4 nerve root
  3. A paracentral disc at L3/4 impinges the L4 nerve root
A

1.Annular tears are usually traumatic in origin - F, although this depends on the interpretation of trauma being separate from microtrauma from spinal motion (“wear and tear”). [Brant-Zawaski et al., Spine]

91
Q
  1. Which is true regarding Tarlov cysts?
  2. They cause bone erosion
  3. They are symptomatic in 50%
  4. They are most common at S1 and S4
A
  • LW: bone remodelling and scalloping can be considered and seen as pressure erosions, like that in PVNS for example, so I would favor “erosions” to be true, (if the remaining options were all false)
  • *LJS now agree

ANSWER: 1.They cause bone erosion - T - maybe associated with bone erosion [Neuroradiology Requisites 2nd ed P806-7]

  1. Which is true regarding Tarlov cysts? (CC) = perineural cyst, extradural meningeal cyst, spinal nerve root diverticulum
  2. They cause bone erosion - T - maybe associated with bone erosion [Neuroradiology Requisites 2nd ed P806-7]
  3. They are symptomatic in 50% - F - symptomatic in one fifth [Neuroradiology Requisites 2nd ed P806-7]
  4. They are most common at S1 and S4 – F - most common at S2 and S3 [Neuroradiology Requisites 2nd ed P806-7]

TARLOV CYST- define: dilated spinal nerve root sheath, containing nerve fibre

92
Q

92.Which is false regarding discitis?

  1. Staph aureus is the most likely cause of discitis in a 70 y.o female
  2. Discitis can mimic Modic Type 1 changes
  3. Early Mycobacterial involvement typically collapses the intervertebral disc space
A

answer: 3.Early Mycobacterial involvement typically collapses the intervertebral disc space - False - Disc height is initially relatively spared (Pott’s disease) [Neuroradiology Requisites 2nd ed p796]
26. Which is false regarding discitis? (CC)
1. Staph aureus is the most likely cause of discitis in a 70 y.o female - T - Staphylococcus is the most common organism [Neuroradiology Requisites 2nd ed P795]
2. Discitis can mimic Modic Type 1 changes - T - T1 hypointense and T2 hyper intense (marrow oedema) is a typical findings [Neuroradiology Requisites 2nd ed P796]
3. Early Mycobacterial involvement typically collapses the intervertebral disc space - False - Disc height is initially relatively spared (Pott’s disease) [Neuroradiology Requisites 2nd ed p796]

93
Q

93.Which is false regarding Modic changes?

  1. Low T1, high T2 equals Modic Type 1 changes
  2. High T1, low T2 equals Modic Type 2 changes
A
  • Type I: Hypointense on T1WI, hyperintense on T2WI (water – fibrovascular marrow)
  • Type II: Hyperintense on T1WI, isointense on T2WI (fat – fatty marrow)
  • Type III: Hypointense on T1WI and T2WI (sclerosis – bony sclerosis with little residual marrow)
94
Q
  1. Which is false regarding femoroacetabular impingement?
  2. A CT is needed for confirmation
  3. Cam type
  4. Pincer type
  5. Typical age = 20-40
A

answer: 1.A CT is needed for confirmation - F - CT may be required to exclude other pathologies if there is clinical doubt but rarely “needed” to confirm FAI, but is can be useful in surgical planning
28. Which is false regarding femoroacetabular impingement? (CC) MSK Req p204
1. A CT is needed for confirmation - F - CT may be required to exclude other pathologies if there is clinical doubt but rarely “needed” to confirm FAI, but is can be useful in surgical planning. [Leunig, Clin Orthop Res 2009, p618]
2. Cam type - T [Dahnert 6th ed.]
3. Pincer type - T [Dahnert 6th ed.]
4. Typical age = 20-40 - T [Dahnert 6th ed.]

95
Q

95.Which is false regarding Morton’s neuroma?

  1. 2nd metatarsal interspace
  2. Causes an entrapment neuropathy of the digital nerve
  3. Well defined, hypoechoic on US
  4. 1% bilateral
  5. Low on T1
A

answer: 4. 1% bilateral - False - 10 % bilateral [Munk P309]; bilateral in 0-12% (StatDx), *LW - radopedia also states 10% bilateral, so this is least correct.
29. Which is false regarding Morton’s neuroma? (CC) MSK Req p502; Munk
1. 2nd metatarsal interspace - T - [Dahnert 6th ed.] *LW: 2nd MT space is the 2nd most common location after 3rd MT space. (would depend on wording)
2. Causes an entrapment neuropathy of the digital nerve - T - [Dahnert 6th ed.]
3. Well defined, hypoechoic on US - T - [Munk P309] – hypoechoic, well-defined ovoid mass, at or proximal to metatarsal head
4. 1% bilateral - False - 10 % bilateral [Munk P309]; bilateral in 0-12% (StatDx)
5. Low on T1 - True - Low to iso-intense on T1 [Munk P310], T2 intermediate to high, intense enhancement (StatDx)

96
Q
  1. Which is not a cause of a pelvic bone lesion with soft tissue mass?
  2. Chordoma
  3. Ewings sarcoma
  4. Chondroblastoma
A

answer:
*LW: Chondroblastoma - F - [ Dahnert 6th ed P56]. Less common in flat bones. StatDx – can occur in epiphyseal equivalents in iliac bone. Usually no ST mass, and rare in iliac bones, compared to usual occurance in long bone epiphysis.

  1. Which is not a cause of a pelvic bone lesion with soft tissue mass? (CC/SK)
  2. Chordoma - T? - originates from embryonic remnants of notochord. Myxoid type can have soft tissue mass [ Dahnert 6th ed P200] . Occur in sacrum (not really “pelvic bone”)
  3. Ewings sarcoma - T - usually have soft tissue mass[ Dahnert 6th ed P75]
  4. Chondroblastoma - F - [ Dahnert 6th ed P56]. Less common in flat bones. StatDx – can occur in epiphyseal equivalents in iliac bone. Usually no ST mass.
97
Q

97.With respect to scoliosis, which is false?

  1. Typically convex to right (thoracic) and left (lumbar)
  2. A right scoliosis is one which is convex to the right
  3. There is asymmetry of the iliac apophyses
  4. Wedging suggests early onset osteoporosis
A

ANSWER:3.There is asymmetry of the iliac apophyses - F, asymmetry of iliac apophyses is not a criterion in scoliosis assessment. Iliac apophyses are commented on regarding skeletal maturation

31.With respect to scoliosis, which is false? (CC)

.1.Typically convex to right (thoracic) and left (lumbar) - T – typical for adult idiopathic scoliosis.

  1. A right scoliosis is one which is convex to the right - T “S-shape” on AP view
  2. There is asymmetry of the iliac apophyses - F, asymmetry of iliac apophyses is not a criterion in scoliosis assessment. Iliac apophyses are commented on regarding skeletal maturation.
  3. Wedging suggests early onset osteoporosis - T - seems inherently true, finally found a reference that associated idiopathic scoliosis with osteoporosis as many of the early bone densitometry in adolescent females was fraught with lack of normal values. [Szalay, Adolescents with idiopathic scoliosis are not osteoporotic, 2008, P806]
98
Q

98.Lucent metaphysis, which is false?

  1. Scurvy
  2. Hypothyroidism
  3. Leukaemia
  4. Rickets
  5. TORCH
A

ANSWER:
2.Hypothyroidism - False - Can cause generalised demineralisation and stippled epiphyses in childeren but Dahnert says no skeletal changes in adult onset [Dahnert 6th ed p107]

  1. Lucent metaphysis, which is false? (CC/SK)
  2. Scurvy - True - “LINING” [Dahnert 6th ed. P5]
  3. Hypothyroidism - False - Can cause generalised demineralisation and stippled epiphyses in childern but Dahnert says no skeletal changes in adult onset [Dahnert 6th ed p107]
  4. Leukaemia - True [Dahnert 6th ed. P5]
  5. Rickets - True - [Dahnert 6th ed. P5]
  6. TORCH - True - [Eisenberg 873]

LUCENT METAPHYSEAL BAND DDX (Lining)- same as below

DENSE METAPHYSEAL BAND DDX

  • infection (torch)
  • tumour : leukaemia, lymphoma, neuroblastoma met
  • trauma/fracture
  • metabolic-> RICKETS (lucent in active disease)-> SCURVY -> renal dystrophy-> treated hypoparathyroidism
  • normal (physiologic or weight bearing bone)
  • lead poisoning
99
Q
  1. Not a feature of a toddlers fracture
  2. Diaphyseal
  3. Calcaneal
  4. Cuboid
  5. Fibula
  6. Navicular
A

ANSWER:5.Navicular - F - [Not yet described in the radiology literature]

  1. Not a feature of a toddlers fracture ? (CC)
  2. Diaphyseal - T - [Dunbar, 1964, J Can, Assoc Radiology]
  3. Calcaneal - T - [Laliotis, Injury, 1993]
  4. Cuboid - T - [Blumberg, Radiology, 1991]
  5. Fibula - T - [Donnelly, AJR 2000]
  6. Navicular - F - [Not yet described in the radiology literature]
100
Q

100.Which is false regarding imaging the Achilles tendon tears ?

  1. Repeated microtrauma can produce hyperintensity within the middle of the tendon
  2. Peritendinous fluid around the tendon on MRI
  3. Achilles tendon thickness may not be assessable in hyperlipidaemia
  4. Frequently tear distal calcaneal insertion
A

NSWER:4.Frequently tear distal calcaneal insertion - False - usually 2 -6 cms proximal to calcaneal insertion (a relatively avascular zone) [all stems adjusted from MSK requisites]

  1. Which is false regarding imaging the Achilles tendon tears ? (CC/SK) MSK Req p259A
  2. Repeated microtrauma can produce hyperintensity within the middle of the tendon - True
  3. Peritendinous fluid around the tendon on MRI - True
  4. Achilles tendon thickness may not be assessable in hyperlipidaemia - True
  5. Frequently tear distal calcaneal insertion - False - usually 2 -6 cms proximal to calcaneal insertion (a relatively avascular zone) [all stems adjusted from MSK requisites]
101
Q

101.32 yr old man with calcified mass below lesser trochanter on plain XRAY , but with no cortical involvement lucent centre and low uptake on MDP bone scan, which is the most likely diagnosis

  1. Osteosarcoma.
  2. Juxtacortical chondroma.
  3. Osteochondroma.
  4. Metastases.
A

**LJS ?juxtacortical chondroma. 50% have matrix calcification
Can’t be osteochondroma if no cortical involvement, mets uncommon <40 yr. Extraskeletal osteosarcoma - less likely “lucent center” - typically has calcification in center of lesion

Not sure about this one, but sounds like there should be another option of ‘myositis ossificans’.

Osteosarcoma - would have to be parosteal or extraskeletal - both about 4% of OS.
Parosteal - cauliflower-like homogeneous ossific mass, cortical thickening.
Both parosteal and estraskeletal periphery is less dense than centre.
ESOS does not have bone attachment.Juxtacortical chondroma - no periosteal reaction, but may lift it causing columns or buttresses - “saucerisation”.

Osteochondroma - usually attached. Can fracture, but tend to heal again.

Mets - 32yo and soft tissue met? Bit low for a Ca++ node. Myositis ossificans would probably be a more suitable option.

102
Q
  1. Causes of Acro-osteolysis (which is false?)
  2. Reiters syndrome.
  3. Leprosy.
  4. Ergotamine
  5. Scleroderma.
  6. Phenytoin toxicity
  7. Prostaglandin
A

6.Prostaglandin - F - is a cause of periosteal reaction

PINCHFO

  • psoriasis, pyknodysosstosis
  • injury : thermal (burn/frostbite
  • neuropathy : DM, leprosy
  • Collagen vascular disease - Scleroderma
  • Hyperparathyroidism
  • familial : hadju cheney
  • others: PVC , progeria
  • others: reiter, ergotamine, phenytoin
103
Q
  1. Which of the following are associations?
  2. Hypoparathyroidism and ulcers.
  3. Myxoedema and Cushings.
  4. Hyperparathyroidism and Basal ganglia calcification.
A

3.Hyperparathyroidism and Basal ganglia calcification.

104
Q
  1. Subarticular lesions are seen in the following except?
  2. Chondromyxoid fibroma.
  3. PVNS.
  4. GCT.
  5. CPDD.
  6. Chondroblastoma.
A

ANSWER:1.Chondromyxoid fibroma. F occur eccentrically in metaphyses, esp. in proximal tibia

  1. Subarticular lesions are seen in the following except? RA/SK
  2. Chondromyxoid fibroma. F occur eccentrically in metaphyses, esp. in proximal tibia
  3. PVNS.T large erosions, subchondral cysts
  4. GCT. T meta-epiphysis → subchondral bone
  5. CPDD. T subchondral cysts
  6. Chondroblastoma. T epiphyseal
105
Q

105.Lesions involving the Knee?

  1. CPPD most commonly affects patello-femoral joint.
  2. RA usually involves patello-femoral joint.
  3. Lack of bone proliferation is more suggestive of Reiters
  4. Haemochromatosis produces widening of the intracondylar notch
A

ANSWER:1.CPPD most commonly affects patello-femoral joint.T a favoured site

39.Lesions involving the Knee? RA

  1. CPPD most commonly affects patello-femoral joint.T a favoured site
  2. RA usually involves patello-femoral joint. F usually medial & lateral compartments
  3. Lack of bone proliferation is more suggestive of reiters. F - Poriatic and Reiters usually have evidence of bone proliferation (bone excrescences at ligamentous and tendinous attachments or a periostitis).
  4. Haemachromatosis produces widening of the intracondylar notch. F- hemophilia and JRA causes widening from overgrowth of ends of bones.
106
Q

106.Regarding Osteosarcoma which is false?

  1. 10-20% are extra-skeletal.
  2. Telangetic variant has fluid-fluid levels on MRI.
  3. 40-60% occur around the knee.
  4. osteosarcomas of jaw are more common than in other sites in the elderly.
  5. Arises in the metaphyseal medullary canal in the majority of cases
A

ANSWER:1.10-20% are extraskeletal - F - 4% of osteosarcomas

  1. Osteosarcoma, which is false: (TW)
  2. 10-20% are extraskeletal - F - 4% of osteosarcomas
  3. Telangectatic variant has fluid fluid level on MRI - T - fluid-fluid levels in 90%.
  4. 40-60% occur about the knee - T - conventional OSA 50-55%, other subtypes vary and although low-grade intraosseous osteosarcoma if often about the knee, still ~50% of all osteosarcoma occur near the knee.
  5. Osteosarcomas of the jaw are more common than in other sites in the elderly - T - This answer is T as per the CME’s. Gnathic osteosarcoma - avg age 34yo. Is generally said 3rd-4th decade. Of note Dahnert says parosteal osteosarcoma peak age 38yo..
    * AJL - I think this is false. Axial and mandibular is more likely than in the younger population however is not more common than other sites.

5.Arises in the metaphyseal medullary canal in the majority of cases - T – conventional osteosarcoma - origin in metaphysis 90-95% / diaphysis 2-11%. Can be intramedullary (most common in conventional OS), intracortical or juxtacortical/surface (peri-/parosteal).

107
Q

107.The following are true concerning unilateral facet dislocation?

  1. Mechanism of injury is simultaneous hyperextension and rotation.
  2. There is disruption of anterior ligament complex.
  3. There is forward subluxation of the involved vertebra of a distance equal to or grater than half of the AP diameter of the body.
  4. The optimum plain radiographic view for the diagnosis is the ipsilateral oblique.
  5. Injury is mechanically unstable.
A

ANSWER:4.The optimum plain radiographic view for the diagnosis is the ipsilateral oblique.T

  1. The following are true concerning Unilateral facet dislocation? RA
  2. Mechanism of injury is simultaneous hyperextension an rotation. F flexion-rotation injury
  3. There is disruption of anterior ligament complex.F
  4. There is forward subluxation of the involved vertebra of a distance equal to or grater than half of the AP diameter of the body. F less than 1/4 of the AP diameter of the body
  5. The optimum plain radiographic view for the diagnosis is the ipsilateral oblique.T
  6. Injury is mechanically unstable. F stable
108
Q

108.Which is true re BMD estimation using DEXA scan?

  1. T score compares individuals BMD to age matched mean.
  2. BMD abnormal if at least 1.5 SD below the mean of the young normal population.
  3. Tscore -2.2 indicates Osteoporosis.
  4. For each SD decrease in BMD there is a doubling of fracture risk.
A

*LW: multiple answers appear true:

  1. BMD abnormal if at least 1.5 SD below the mean of the young normal population. True
    * LW: T score = density relative to young adult. T score above -1.0 is normal. Between -1.0 and -2.5 is Osteopenia, and below -2.5 is osteoporosis. Thus 1.5 below normal young population average is saying T score = -1.5, which equals osteopenia, and hence abnormal BMD. Thus, I favour this answer also true.
  2. For each SD decrease in BMD there is a doubling of fracture risk. T. See this Medscape article:
  3. Which is true re BMD estimation using DEXA scan? [AB]A
  4. T score compares individuals BMD to age matched mean. F. Z-score is age-matched. T-score compares BMD to a healthy young (30 year-old) adult. Both scores are sex-matched.
  5. BMD abnormal if at least 1.5 SD below the mean of the young normal population. F. Normal is defined as T-score > -1.
    * LW: T score = density relative to young adult. T score above -1.0 is normal. Between -1.0 and -2.5 is Osteopenia, and below -2.5 is osteoporosis. Thus 1.5 below normal young is saying T score = -1.5, which equals osteopenia, and hence abnormal BMD. Thus, I favour this answer also true.
  6. Tscore -2.2 indicates Osteoporosis. F. Osteopenia (between - 1 to - 2.5. Osteoporosis T-score < -2.5.
  7. For each SD decrease in BMD there is a doubling of fracture risk. T. See this Medscape article: http://www.medscape.com/viewarticle/472681_2
109
Q
  1. Fatty marrow most likely site?
  2. Femoral metaphyses.
  3. Femoral diaphyses.
  4. Ribs.
  5. Pelvis.
  6. Proximal humerus.
A

2.Femoral diaphyses. Most likely to have fatty marrow (SK) pelvis least likely

110
Q

110.Regarding rib notching, which is false?

  1. NF causes superior and inferior rib notching.
  2. Unilateral right rib notching may be seen with coarctation and left subclavian artery arising distal to coarct
  3. Superior rib notching may be seen with SVC obstruciton
  4. Rib notching related to connective tissue diseases tends to be superior
A

ANSWER:3.Superior margin rib notching may be seen with SVC obstruction - F - inferior margin due to enlargement of intercostal veins. Other venous causes: AV malformation of chest wall, SVC obstruction (D 6th ed)

  1. Regarding rib notching, which is false? (TW)
  2. NF causes superior and inferior rib notching - T - Chapman & Nak
  3. Unilateral right rib notching may be seen with coarctation and left subclavian artery arising distal to coarct -T - Blood via right innominate (right SCA) to intercostal aa which function as collaterals to descending aorta. As coarct before left SCA, no left sided shunting. Other uniltareal causes - posteroperative Blalock-Taussig shunt (SCA to PA), coarct prox to aberrant SCA (D 6th ed)
  4. Superior margin rib notching may be seen with SVC obstruction - F - inferior margin due to enlargement of intercostal veins. Other venous causes: AV malformation of chest wall, SVC obstruction (D 6th ed)
  5. Rib notching related to connective tissue diseases tends to be superior - T - Rheumatoid, SLE, scleroderma, Sjogren’s syndrome (C & N).
111
Q

111.Regarding pediatric bone disorder (which is false?)

  1. Periosteal reaction can be normal in a femur of a 2m old
  2. In 8yo child, distal femoral epiphyseal irregularity is seen in hypothyroidism
  3. Os tibiale externum is on the lateral aspect of the navicular
A

ANSWER:3.Os tibiale externum is on the lateral aspect of the navicular. F medial

  1. Regarding pediatric bone disorder. (RA)
  2. Perisoteal reaction can be normal in a femur of a 2m old. T Bilateral and symmetrical
  3. In 8yo child, distal femoral epiphyseal irregularity is seen in hypothyroidism.T fragmented and stippled
  4. Os tibiale externum is on the lateral aspect of the navicular. F medial dahnert
112
Q

112.Which is true regarding rotator cuff tears:

  1. Commonly associated with type 1 acromion
  2. Most often in posterior third of supraspinatus tendon
  3. Typically echogenic
  4. Full thickness tears found in at least 10% of asymptomatic individuals >80 years old
  5. Absence of fluid in subacromial/subdeltoid bursa excludes diagnosis
A
  1. Full thickness tears found in at least 10% of asymptomatic individuals >80 years old T - The precise incidence of symptomatic rotator cuff injuries is not known. Many individuals with full-thickness tears are not only asymptomatic but they have minimal functional disability. The most accepted figure is 20-30%. Cadaver studies of elderly persons have estimated full-thickness tears as high as 30%. (eMedicine)
  2. Which is true regarding rotator cuff tears: (GC)
  3. Full thickness tears found in at least 10% of asymptomatic individuals >80 years old
  4. Commonly associated with type 1 acromion F - Type I has a flat undersurface. Type II is curved downward (concave), type III is hooked downward, type IV is curved upward (convex). Check this at MRI in the sagittal plane. Type III has a significantly higher incidence of rotator cuff tears. Other factors that may contribute to RC impingment (and subsequent degeneration-tear) include AC osteophytes and os acromiale. (Rotator cuff RG 2006)
  5. Most often in posterior third of supraspinatus tendon F - “critical zone” is 1cm medial to tendon insertion (area of relative hypovascularity), tears usually start in anterior portion and propagate posteriorly. *LW: conflicting literature, Helms states anterior most fibres are where tears usually begin.
  6. Typically echogenic F - typically hypoechoic area, represents thickened bursa filling the defect (Dahnert)
  7. Full thickness tears found in at least 10% of asymptomatic individuals >80 years old T - The precise incidence of symptomatic rotator cuff injuries is not known. Many individuals with full-thickness tears are not only asymptomatic but they have minimal functional disability. The most accepted figure is 20-30%. Cadaver studies of elderly persons have estimated full-thickness tears as high as 30%. (eMedicine)
  8. Absence of fluid in subacromial/subdeltoid bursa excludes diagnosis F - most sensitive sign (Dahnert) but may not be seen in articular-surface partial thickness tears.
113
Q

113.Which statement is most correct:

  1. Tear of the Achilles tendon most commonly occurs at the origin.
  2. Tear of the Achilles tendon most commonly occurs 4cm above the insertion
  3. Tenosynovitis may be an MRI feature of tendon strain or degeneration
  4. Anterior to the Achilles tendon is a triangular fat pad called Haglund’s triangle
  5. When normal it has a convex anterior margin on axial T1 weighted images
A

ANSWER:2.Tear of the Achilles tendon most commonly occurs 4-8cm above the insertion T - usually occur about 4cm above its calcaneal insertion (2-6cm), but may also occur anywhere along the length of the tendon. Tears may also occur at the musculotendinous junction so must have a large enough FOV to include this region on the sagittal images. Acute injury = haemorrhage and oedema; chronic = muscle atrophy.

  1. Achilles tendon, which is most correct: (GC)
  2. Tear of the Achilles tendon most commonly occurs at the origin F
  3. Tear of the Achilles tendon most commonly occurs 4-8cm above the insertion T - usually occur about 4cm above its calcaneal insertion (2-6cm), but may also occur anywhere along the length of the tendon. Tears may also occur at the musculotendinous junction so must have a large enough FOV to include this region on the sagittal images. Acute injury = haemorrhage and oedema; chronic = muscle atrophy.
  4. Tenosynovitis may be an MRI feature of tendon strain or degeneration F - Achilles does not have a tendon sheath because it does not come into close contact with other structures along its length; therefore it cannot have changes of tenosynovitis, but only of paratendinitis. There is a paratenon present on the dorsal, medial and lateral aspects of the tendon that allow smooth gliding of the tendon in lieu of a tendon sheath.
  5. Anterior to the Achilles tendon is a triangular fat pad called Haglund’s triangle F - it is called Kager’s triangle. Haglund’s triad (or “pump bumps”) refers to retro-Achilles bursitis, retrocalcaneal bursitis, and thickening of the distal Achilles tendon.
  6. When normal it has a convex anterior margin on axial T1 weighted images F - normally flat or concave; if it becomes diffusely convex, an abnormally thickened tendon exits. (see images Qu 40, April 2004)
114
Q

114.With regard to imaging of the scaphoid bone (T/F):

  1. Plain film misses <5% of scaphoid fractures
  2. In the majority of normal subjects the periscaphoid fat plane is present on plain radiography
  3. The optimal timing of scintigraphy to detect a fracture is < 24 hours from injury
  4. Approximately 70% of fractures occur through the waist
  5. MR imaging is the most sensitive modality to assess for avascular necrosis
A
  1. Plain film misses <5% of scaphoid fractures F - misses 25-65%. If initial xray negative, reexamine in 2 + 6 weeks after treatment with short-arm cast (Dahnert), B& H say reimage in 1 week, or MRI for definitive short term answer.
  2. In the majority of normal subjects the periscaphoid fat plane is present on plain radiography T - Absence or displacement of avicular fat stripe should raise suspicion of scaphoid fracture.
  3. The optimal timing of scintigraphy to detect a fracture is < 24 hours from injury F - Bone scan PPV 93% after 2-3 days.
  4. Approximately 70% of fractures occur through the waist T - Proximal third 20% (failure to reunite 90%), middle third 70% (failure to reunite 30%), distal third 20% (usually fragments reunite).
  5. MR imaging is the most sensitive modality to assess for avascular necrosis T - MRI more sensitive and specific than radiography or CT. Evaluates for proximal pole T2 hypointensity (AVN), and T1 C+ can document viability of proximal pole with enhancement.
115
Q

115.Ulnar deviation of the metacarpophalangeal joint is a recognised feature of:

  1. Reiter’s syndrome
  2. Systemic lupus erythematosus
  3. Psoriasis
  4. Calcium pyrophosphate deposition disease (CPPD)
  5. Haemochromatosis
A

2.Systemic lupus erythematosus “nondeforming, nonerosive arthropathy”. Joints are affected in >80% of SLE patients. 10% develop a chronic deforming synovitis; Jaccoud’s arthropathy primarily involves the hands - ulnar deviation at the MCPJs, later may have swan-neck and boutonniere deformities. Periarticular soft tissue swelling, juxta-articular osteoporosis are similar to RA, but usually no erosions or joint space narrowing. Jaccoud’s has been described in a variety of disorders but is most commonly seen in patients with SLE. (RG 2004, Ann Rh Dis 1992)

116
Q

`116.The following are features of pyogenic infection of the spine, except:

  1. Staphylococcus aureus is the most common causative organism
  2. The thoracic spine is the most common site
  3. The characteristic radionuclide bone scan is increased uptake in two adjacent vertebral bodies
  4. Rapid loss of disc height favours pyogenic over tuberculous infection
  5. Osteoblastic reaction with sclerosis is more common than with tuberculosis
A

ANSWER:2.The thoracic spine is the most common site F - L3/4, L4/5, unusual above T9 (with regard to discitis, assumed disc-osteomyelitis)

  1. The following are features of pyogenic infection of the spine, except: (GC)
  2. Staphylococcus aureus is the most common causative organism T - usually from haematogenous spread, less often iatrogenic/penetrating injury. Other pyogenic causes include Salmonella (associated with sickle cell disease), GNB - E.coli, Klebsiella, Pseudomonas (in IVDU or immunocompromised).
  3. The thoracic spine is the most common site F - L3/4, L4/5, unusual above T9 (with regard to discitis, assumed disc-osteomyelitis)
  4. The characteristic radionuclide bone scan is increased uptake in two adjacent vertebral bodies T - vertebra usually seeded haematogenously, with subsequent spread to disc space and adjacent VB.
  5. Rapid loss of disc height favours pyogenic over tuberculous infection T - S.aureus produces enzymes that rapidly ‘digest’ the discs. Pearl: destruction of disc space implies pyogenic infection. TB spondylitis in contrast, often spares the disc.
  6. Osteoblastic reaction with sclerosis is more common than with tuberculosis T - reactive sclerosis and periosteal reaction typically absent in TB; instead may see demineralisation (resorption of dense margin), or “gouge defect” (mild contour irregularity of anterior/lateral aspect of VB due to erosion from subligamentous extension of tuberculous abscess).
117
Q

117.Characteristic articular changes seen in haemophilia, which is false:

  1. Radiodense soft tissue swelling
  2. Overgrowth of epiphyses
  3. Early uniform joint space loss is uncommon
  4. Commonest in distribution distal to elbow and ankle
  5. Secondary to chronic repetitive haemarthrosis and intraosseous bleeding
A

ANSWER:4.Commonest in distribution distal to elbow and ankle F - knee > ankle > elbow

  1. Characteristic articular changes seen in haemophilia, which is false: (GC)
  2. Radiodense soft tissue swelling T - may see radiodense joint effusion, and haemosiderin deposition in soft tissues may also account for this. Other causes of haemarthrosis include trauma, myeloproliferative disease, antocoagulant/bleeding diathesis, scurvy.
  3. Overgrowth of epiphyses T - due to synovial inflammation and hyperaemia. DDx JRA, paralysis.
  4. Early uniform joint space loss is uncommon T - joint space narrowing occurs secondary to cartilaginous denudation, a late feature of pannus erosion.
  5. Commonest in distribution distal to elbow and ankle F - knee > ankle > elbow
  6. Secondary to chronic repetitive haemarthrosis and intraosseous bleeding T - intra- and periarticular bleeding results in synovial hypertrophy and inflammation (pannus). Osseous changes may be due to elevated intra-articular and intramedullary pressures, as well as pannus causing erosion of cartilage with loss of subchondral bone plate and formation of subarticular cysts.
118
Q

118.Unicameral bone cysts, which is true:

  1. Are more common in the femur than humerus
  2. Involve the epiphysis in about 30% of cases
  3. Contain fluid-fluid levels on MRI in greater than 50% of cases
  4. Arise eccentrically in the metaphysis
  5. Migrate into the diaphysis with time
A

ANSWER:5.Migrate into the diaphysis with time T

  1. Unicameral bone cysts, which is true: (GC)
  2. Are more common in the femur than humerus F - humerus 55-65%, femur 25-30% in patients younger than 20 yrs. Over 20 yo. more common in flat bones (iliac, calcaneus).
  3. Involve the epiphysis in about 30% of cases F - involvement of the epiphysis is rare. Epihyseal involvement may represent a distinct clinical and radiographic entity. Tend to be older patients, and the epiphyseal plates are closed in 50% of these patients. Because of the close proximity to the physeal plate, a greater association with growth arrest exists. (eMedicine)
  4. Contain fluid-fluid levels on MRI in greater than 50% of cases F - Ddx fluid-fluid levels: GCT, ABC, telangiectatic OSA
  5. Arise eccentrically in the metaphysis F - it is the only lesion in FEGNOMASHIC that is always central. Located adjacent to physeal cartilage (during active phase), migrating into diaphysis with growth (during latent phase).
  6. Migrate into the diaphysis with time T
119
Q

119.Fibrous dysplasia, which of the following is incorrect?

  1. It is a benign developmental anomaly of mesenchyme precursors
  2. The mono-ostotic form presents later than the polyostotic variant
  3. The leontiasis ossea variant is not assoc with extracranial lesions
  4. About 10% of patients have spinal involvement
  5. Changes in Mc Cune Albright are typically unilateral
  6. Reactivation of skeletal lesions is assoc with pregnancy CME 03.72 / 02.14
A

ANSWER:4.About 10% of patients have spinal involvement - F - Monostotic FD (70-80% of FD) involving the spine is rare (clinical radiology 1988). In polyostotic FD there are reports of up to 60-65% spinal involvement. Mostly lumbar spine and thoracic spine. Less frequent in sacrum and cervical spine (JBJS).

  1. Fibrous dysplasia, which of the following is incorrect? (TW)
  2. It is a benign developmental anomaly of mesenchyme precursors - T - benign fibroossous developmental anomaly of the mesenchymal precursor of bone, manifested as a defect in osteoblastic differentiation and maturation
  3. The mono-ostotic form presents later than the polyostotic variant - T - usually asymptomatic till 2nd-3rd decade. Polyostotic form 2/3rd symptomatic by 10yo (mean 8y).
  4. The leontiasis ossea variant is not assoc with extracranial lesions - T - No extracranial lesions. AKA craniofacial form. (4 disease patterns of FD are: monostotic, polyostotic, carniofacial, cherubism)
  5. About 10% of patients have spinal involvement - F - Monostotic FD (70-80% of FD) involving the spine is rare (clinical radiology 1988). In polyostotic FD there are reports of up to 60-65% spinal involvement. Mostly lumbar spine and thoracic spine. Less frequent in sacrum and cervical spine (JBJS)

.5.Changes in Mc Cune Albright are typically unilateral - T - MAS - almost exclusively in girls. Polyostotic unilateral FD, “coast of Maine” cafe-au-lait spots, endocrinopathy (peripheral sexual precocity).

6.Reactivation of skeletal lesions is assoc with pregnancy - T - this assoc is more commonly seen with polyostotic form.

120
Q
  1. MRI Achilles
  2. Fluid in the retrocalcaneal bursa is a normal finding
  3. Normal convex anterior margin
  4. Rupture of tendon at insertion
  5. Any high signal T2 is abnormal
  6. T2 signal is seen in all cases of chronic teniopathy
A

*LW:
Appears multiple correct answers, or poor recall:
Correct answers:

1.Fluid in the retrocalcaneal bursa is a normal finding T - normal bursa reported to contain betwwn a fraction of 1ml of fluid to slightly more than 1ml (AJR).
*LW: the normal retrocalcaneal bursa is visible on MR imaging but should measure less than 6 mm superior to inferior, 3 mm medial to lateral, and 2 mm anterior to posterior
“On MR imaging, the asymptomatic retrocalcaneal bursa normally contains detectable high-signal-intensity fluid or synovium or both. A bursa larger than 1 mm anteroposteriorly, 11 mm transversely, or 7 mm craniocaudally is abnormal”

4.Any high signal T2 is abnormal: true (presumed intra tendon signal).
*LW: magic angle refers to signal abnormality on short TE sequences (e.g. T1, GRE, PD), sequences with a longer TE (e.g. T2 including FSE T2) can be used to avoid this artifact. Thus the below previous explanation does not explain this answer.
Most achilles tendon pathology presents as increased T2 signal, with no normal internal T2 hyperintensity reported, so hence this answer is also favoured to be true.

5.T2 signal is seen in all cases of chronic tendinopathy: likely true, however the use of the word “all” makes me suspicious that there may be some cases where there is not increased T2 signal, so this one is dubious, could be argued either way, and likely the least correct out of the above 3 options.

(https: //www.tandfonline.com/doi/pdf/10.3109/02841851003627809)
(https: //pubs.rsna.org/doi/full/10.1148/radiographics.20.suppl_1.g00oc26s153)
(https: //www.ajronline.org/doi/full/10.2214/ajr.175.3.1750613)
(https: //www.ajronline.org/doi/10.2214/ajr.170.5.9574592)

Previous answers
54.MRI Achilles (TW)

  1. Fluid in the retrocalcaneal bursa is a normal finding T - normal bursa reported to contain between a fraction of 1ml of fluid to slightly more than 1ml (AJR).
    * LW: the normal retrocalcaneal bursa is visible on MR imaging but should measure less than 6 mm superior to inferior, 3 mm medial to lateral, and 2 mm anterior to posterior

Read More: https://www.ajronline.org/doi/full/10.2214/ajr.175.3.1750613

  1. Normal convex anterior margin F - flat and concave on axial images
  2. Rupture of tendon at insertion F - 2-6cm superior to os calcis.
  3. Any high signal T2 is abnormal F - Achilles tendon is low signal on all MR imaging sequences, however magic angle can produces areas of increased signal as tendon fibers approach an orientation angle of 55 degrees relative to main magnetic field.
    * LW: magic angle refers to signal abnormality on short TE sequences (e.g. T1, GRE, PD), sequences with a longer TE (e.g. T2 including FSE T2) can be used to avoid this artifact. Thus the above explanation does not explain this answer. Most achilles tendon pathology presents as increased T2 signal, with no normal internal T2 hyperintensity reported, so hence this answer is also favoured to be true.

5.T2 signal is seen in all cases of chronic teniopathy: true.

121
Q

121.Meniscal cysts: WHICH IS TRUE

  1. Associated with vertical tears
  2. Medial:lateral 5:1
  3. Anterior horn medial meniscus usual site
  4. Generally T2 hyperintense
  5. Associated with lateral meniscus posterior horn
A

ANSWER:4.Generally T2 hyperintense T – after all, it is cystic!! – direct contact with meniscal tear in 98%

  1. Meniscal cysts (TW)
  2. Associated with vertical tears F – PHMM: most commonly associated with horizontal cleavage tear. AHLM : most commonly assoc with complex tear (AKA radial split tear with radial and horizontal component)
  3. Medial:lateral 5:1 F – usually seen 66% medial, 33% lateral (AJR 2001). See below.
  4. Anterior horn medial meniscus usual site F – posterior horn of medial meniscus
  5. Generally T2 hyperintense T – after all, it is cystic!! – direct contact with meniscal tear in 98%
  6. Associated with lateral meniscus posterior horn F – anterior horn of lateral meniscus AJR 2001: previously thought meniscal cysts were more common in lateral compartment from studies based largely on arthroscopic and surgical findings. AJR large MR series (1402 tears in 2572 examinations, 109 meniscal cysts) showed 66% in medial compartment, 34% in lateral compartment. 90% had horizontal component, and 98% were adjacent to a meniscal tear.Differences thought due to detection at arthroscopy / Sx vs MRI (ie surrounding structures, fat etc).MM cysts - 74% were adjacent to posterior horn. 13% anterior horn. 14% body.LM cysts - 54% adjacent to anterior horn. 30% posterior horn. 16% body. In their population they found M > F 2x. Dahnert says F > M 1-10x.
122
Q

122.Paediatric C spine, which is true

  1. Atlanto axial subluxation is less common than in adults
  2. Anterior wedge of C3 is not a normal variant
  3. Lateral displacement of lateral masses C1/2 by 7mm is a normal finding in a 4 year old
  4. Injuries are more commonly lower cervical than upper
  5. Facet joints are more horizontal in younger children
A

ANSWER:5.Facet joints are more horizontal in younger children

  1. Paediatric C spine, which is true: (TW)
  2. Atlanto axial subluxation is less common than in adults - F - occurs 5x more common in children than adults.
  3. Anterior wedge of C3 is not a normal variant - F – normal variant, however age dependent: eg, anterior wedging in a 3yo at C5 is likely normal variant. However wedged body at C5 in an 8yo this is likely pathologically compressed.
  4. Lateral displacement of lateral masses C1/2 by 7mm is a normal finding in a 4 year old - F -Total offset (sum of both sides) of more than 6mm of the lateral masses of the atlas with respect to the odontoid is highly suggestive of rupture of the transverse ligament or avulsion of its attachments.
  5. Injuries are more commonly lower cervical than upper - F – in patients under 9yo, almost all injures are to the occiput-C2 region (Imaging of Spinal Trauma in Children, Kuhns).
  6. Facet joints are more horizontal in younger children - T - see below.

Differences between adult and paeds spinal injury (Spinal Trauma in Children, Paeds Radiol 2001) Fulcrum of movement located at C2-3 in the child, C5-6 in adult Relatively large head and weak neck muscles Ligamentous and joint capsule laxity Horizontal orientation of the facet joints in younger children Underdeveloped uncinate processes Mild physiological anterior ‘wedging’ of vertebral bodies Incomplete ossification of odontoid process

123
Q

123.Paget’s disease:

  1. Osteolysis is usually at skull base
  2. Cause of ivory vertebrae
  3. Diaphysis before metaphysical involvement
  4. No increase in risk of sarcoma?
  5. Lytic lesion cold?
A

ANSWER:2.Cause of ivory vertebrae T

  1. Paget’s disease: (RA)
  2. Osteolysis is usually at skull base. F at the calvarium anteriorly (osteoporosis circumscripta) and sclerosis at the skull base.
  3. Cause of ivory vertebrae T
  4. Diaphysis before metaphyseal involvement. F - rarely in the diaphysis
  5. No increase in risk of sarcoma? F - increased risk
  6. Lytic lesion cold?F sclerotic burned out lesions
124
Q
  1. Fatty Marrow, most likely site:
  2. Rib
  3. Spine
  4. Femoral diaphysis
  5. Prox humerus
A

3.Femoral diaphysis Adult marrow pattern reached ~25y - at this time see red marrow in axial skeleton (skull, spine, sternum, flat bones) and prox ends of humeri and femurs), yellow marrow elsewhere. May see subchondral islands of red marrow in proximal humeral epiphyses. Orderly and predictable conversion - begins in appendicular/peripheral skeleton and progresses to the axial/central skeleton. In long bones, marrow conversion first in diaphysis, then distal metaphyses, and finally proximal metaphyses.

125
Q
  1. Downs, false:
  2. Flared iliac wings with small acetabular angles
  3. Congenital dislocation of the hips
  4. Slipped capital femoral epiphysis
  5. Atlanto-axial subluxation
  6. 11 pairs of ribs
A
  • LW: Possibly poor recall as all are true, similar question states increased acetabular angles (which is false).
    1. Flared iliac wings with small acetabular angles: true
  1. Congenital dislocation of the hips: Radiopedia states increased incidence of DDH.
    * *LJS edit - increased risk of DDH due to ligamentous laxity, hypotonia, capsular insufficiency. But generally don’t exhibit dysplasia before walking age**
  2. Slipped capital femoral epiphysis: true
  3. Atlanto-axial subluxation: true
  4. 11 pairs of ribs: true

Previous Answers:
ANSWER:2.Congenital dislocation of the hips - F - not increased (CME answer, however data says DS has 5%, vs 1% in Aust population).

  1. Downs, false: (TW
  2. Flared iliac wings with small acetabular angles - T - “Mickey Mouse ears” / “elephant ears” (cf achondroplasia: square flatened iliac bones “tombstone”; and MPS widely flared with inferior tapering & and large acetabular angles)
  3. Congenital dislocation of the hips - F - not increased (CME answer, however data says DS has 5%, vs 1% in Aust population).
  4. Slipped capital femoral epiphysis - T - increased (eMed)
  5. Atlanto-axial subluxation - T
  6. 11 pairs of ribs - T
126
Q
  1. With regard to pediatric skeletal radiographs, which is true:
  2. A spiral tibial fracture in an 6m infant is unlikley to be due to NAI
  3. SH1 type fractures have a high risk of long term complications of growth
  4. ABC’s are the commonest cause of pathological fractures
  5. Radiographically occult fractures are less common than in adults
  6. Periosteal reaction is seen as a physiological phenomenon
A

answer: 5.Periosteal reaction is seen as a physiological phenomenon - T - physiological periosteal new bone formation can commonly be seen in infants during the first few months of life. Supportive features of physiologic aetiology are: symmetric distribution, benign appearance of the periosteal reaction, appropriate ageo fhte child.
60. With regard to pediatric skeletal radiographs, which is true: (TW)
1. A spiral tibial fracture in an 6m infant is unlikley to be due to NAI - F - not weight bearing yet, unlike an older infant/child where a Toddlers fracture would be most likley.
2. SH1 type fractures have a high risk of long term complications of growth -F
3. ABC’s are the commonest cause of pathological fractures - F - SBC 40%, NOF 19%, fibrous dysplasia 16%, osteosarcoma 15%, ABC 10%. Oritz et al 2005.
4. Radiographically occult fractures are less common than in adults - F
5. Periosteal reaction is seen as a physiological phenomenon - T - physiological periosteal new bone formation can commonly be seen in infants during the first few months of life. Supportive features of physiologic aetiology are: symmetric distribution, benign appearance of the periosteal reaction, appropriate ageo fhte child.

127
Q

127.Posterior vertebral scalloping, false

  1. Communicating hydrocephalus
  2. Psoriasis
  3. Achondroplasia
  4. Neurofibromatosis
  5. Ependymoma
A

Answer:2.Psoriasis - F

61.Posterior vertebral scalloping, false (TW)

  1. Communicating hydrocephalus
  2. Psoriasis - F
  3. Achondroplasia
  4. Neurofibromatosis
  5. Ependymoma

Increased intraspinal pressure / mesenchymal tissue laxity (dural ectasia) / Bone softening.
Pressure - lipoma, ependymoma, syringohydromelia, Mening, hydrocephalus
Abnormal bones - Ank spond, Marfans, Acromegaly, Achondroplasia, MPS,Dural ectasia - NF, Marfan, Ehlers Danlos.Variation on CME 98.04

128
Q
  1. Melorheostosis, false:
  2. Soft tissue contractures
  3. Muscle atrophy
  4. Sclerotomal distribution
  5. Malignant transformation
  6. Lymphangiectasisa
A

Answer:4.Malignant transformation - F - but can be associated with tumors

62.Melorheostosis, false: (TW)

.1.Soft tissue contractures - T - limited joint motion. Thickening and fibrosis of overlying skin. Flexion contractures of hip and knee.

  1. Muscle atrophy - T - frequent
  2. Sclerotomal distribution - T - usually monomelic with at least 2 bones involved in a dermatomal distribution
  3. Malignant transformation - F - but can be associated with tumors.
  4. Lymphangiectasisa - TMelorheostosis, rare mesenchymal dysplasia. Sclerosing bone disorder. Due to loss of function mutation in LEMD3 gene (AKA MAN1) which encodes an inner nuclear membrane. Past paper Sept 04
129
Q

129.Lumbar spine, which is true:

  1. Spondylolisthesis at L4 most commonly associated with spondylolysis
  2. Disc herniation is most commonly posterolateral
  3. Degenerative changes are most prominent in the mid lumbar spine
  4. Height of disk is directly proportional to the size of the protrusion
  5. Limbus vertebra is of superior endplate
A

ANSWER:2.Disc herniation is most commonly posterolateral 49% posterolateral

  1. Lumbar spine, which is true: (RA)
  2. Spondylolisthesis at L4 most commonly associated with spondylolysis F not necessarily
  3. Disc herniation is most commonly posterolateral 49% posterolateral
  4. Degenerative changes are most prominent in the mid lumbar spine F lower lumber
  5. Height of disk is directly proportional to the size of the protrusion F
  6. Limbus vertebra is of superior endplate - F intraosseus herniation of disk material at junction of vertebral bony rim of centra + endplate (anterior superior corner) Dah
    * *LJS - typically anteriosuperior corner but can be inferior
130
Q

130.With regards to Osteomyelitis – discitis of the spine, which is false:

  1. The commonest organism is staph
  2. Changes may be mimicked by Modic 1 changes on T1 and T2
  3. Cervical spine is the most common site
  4. Involvement of multiple vertebral bodies is most likely TB
  5. Rapid loss of height favours pyogenic over TB
A

Answer:3.Cervical spine is the most common site F lumber and lower thoracic

  1. With regards to Osteomyelitis – discitis of the spine, which is false: (RA)
  2. The commonest organism is staph T – staph is by far the most common organism; E Coli, Klebsiella & Psuedomonus
  3. Changes may be mimicked by Modic 1 changes on T1 and T2 T -
  4. Cervical spine is the most common site F lumber and lower thoracic
  5. Involvement of multiple vertebral bodies is most likely TB
  6. Rapid loss of height favours pyogenic over TB T – (TB = slow collapse), discs preserved
131
Q
  1. Regarding fibrous cortical defect, which is false:
  2. Longitudinal with bone
  3. Multiloculated
  4. Can present with vague pain
  5. Expansile with thin cortex
  6. Most common age 2-10 years
A

ANSWER:3.Can present with vague pain. F No pain. No periosteal reaction unless fracture

65.Regarding fibrous cortical defect, which is false: (RA)

  1. Longitudinal with bone.T
  2. Multiloculated T
  3. Can present with vague pain. F No pain. No periosteal reaction unless fracture
  4. Expansile with thin cortex -T slightly expansive (B&H), can be hot on nuclear bone scan as they heal
  5. Most common age.T 2-10 years. REF DAH
132
Q

132.Ankle fracture, true/false:

  1. Proximal fibular fracture is related to internal rotation
  2. Talus is displaced if joint widened over 4mm
  3. Shepards fracture mimics os trignoum
  4. Os tibiale externum is lateral to the navicular
A
  • LW:
    1. Proximal fibular fracture is related to internal rotation: False, external rotation.
    2. Talus is displaced if joint widened over 4mm: likely true, most references state medial clear space > 4mm (radio-edina), while others state 4.5mm or >5mm, with upto 4mm considered normal, so technically true according to radiopedia.
    3. Shepards fracture mimics os trignoum: true.
    4. Os tibiale externum is lateral to the navicular: False (Medial to navicular)

66.Ankle fracture, true//false: (RA)
ANSWER: 2 and 3 both true???

  1. Proximal fibular fracture is related to internal rotation. F Pronation-external rotation.
  2. Talus is displaced if joint widened over 4mm .T
  3. Shepards fracture mimics os trignoum.T FRACTURE OF the lateral tubercle of posterior process of talus
  4. Os tibiale externum is lateral to the navicular. F - medial
133
Q

133.Osteosarcoma, which is false:

  1. 40-60% occur about the knee
  2. 10-20% are extraskeletal
  3. Telangectatic variant has fluid fluid level on MRI
  4. Jaw lesions more common in older adults
  5. Arises in the metaphyseal medullary canal in the majority of cases
A

Answer:2.10-20% are extraskeletal - F - 4% of osteosarcomas

  1. Osteosarcoma, which is false: (TW)
  2. 40-60% occur about the knee - T - conventional OSA 50-55%, other subtypes vary and although low-grade intraosseous osteosarcoma if often about the knee, still ~50% of all osteosarcoma occur near the knee.
  3. 10-20% are extraskeletal - F - 4% of osteosarcomas
  4. Telangectatic variant has fluid fluid level on MRI - T - fluid-fluid levels in 90%.
  5. Jaw lesions more common in older adults - T - gnathic osteosarcoma - avg age 34yo. Is generally said 3rd-4th decade. Of note Dahnert says paroeteal osteosarcoma peak age 38yo. This answer is T as per the CME’s.
  6. Arises in the metaphyseal medullary canal in the majority of cases - T – conventional osteosarcoma - origin in metaphysis 90-95% / diaphysis 2-11%. CME 02.01 (14)
134
Q

134.Osteomyelitis, which is false:

  1. Sclerosing Osteomyelitis of garre involves the mandible
  2. Crosses the epiphysis in children
  3. Brodies abscess is usually in the diaphysis
A

Answer:3.Brodies abscess is usually in the diaphysis F - Typically in metaphysis, or in epiphysis in children/infants (Dahnert)CME 02.17 (30)

  1. Osteomyelitis, which is false: (JS)
  2. Sclerosing Osteomyelitis of Garre involves the mandible T – Sterile osteomyelitis, with a low-grade nonnecrotic nonpurulent infection, typically in mandible, focal bulge of thickened cortex with sclerosing periosteal reaction. (Dahnert).
  3. Crosses the epiphysis in childrenT - Typically occurs in the ends of tubular bones, metaphysis but can be in epiphysis in children and infants. (Dahnert) (Probably meant to say “crosses into the epiphysis”)
  4. Brodies abscess is usually in the diaphysis F - Typically in metaphysis, or in epiphysis in children/infants (Dahnert)CME 02.17 (30)
135
Q
  1. Regarding bone age, which is not used:
  2. Sex
  3. Metacarpal index
  4. Width of physis
  5. Size of epiphysis
  6. Shape of epiphysis
A

answer: 2.Metacarpal index - F – Metacarpal index is used to diagnose Marfan’s syndrome (average the 4 ratios of length of 2nd to 5th metacarpals divided by their middiaphyseal width; in Marfan’s >8.8 in male or 9.4 in females) (Dahnert) -> i.e assess arachnodactyly
69. Regarding bone age, which is not used: (JS)

  1. Sex
  2. Metacarpal index - F – Metacarpal index is used to diagnose Marfan’s syndrome (average the 4 ratios of length of 2nd to 5th metacarpals divided by their middiaphyseal width; in Marfan’s >8.8 in male or 9.4 in females) (Dahnert)
  3. Width of physis
  4. Size of epiphysis
  5. Shape of epiphysis
136
Q
  1. Terminal tuft erosions, which is false:
  2. Scleroderma
  3. Ankylosing spondylitis
  4. Hyperparathyroidism
  5. Frostbite
  6. Raynauds disease
A

:2.Ankylosing spondylitis

  1. Terminal tuft erosions, which is false: (JS) answer
  2. Scleroderma
  3. Ankylosing spondylitis
  4. Hyperparathyroidism
  5. Frostbite
  6. Raynaud’s disease

DDx of tuft erosions: Scleroderma, Raynaud’s disease, Psoriatic arthropathy, neuropathic (diabetes, leprosy etc), Thermal injuries, Trauma, Hyperparathyroidism, Epidermolysis bullosa, Porphyria, Phenytoin toxicitiy, snake and scorpion venom (Chapman and Nakielny) PINCHFO

137
Q

137.Posterolateral corner injury, which is least correct:

  1. ACL tear
  2. PCL tear
  3. Reverse segond fracture
  4. Transverse meniscal ligament tear
  5. Meniscocapsular ligament separation
A

*LW:
THink transverse meniscal ligament tear is also false.
Reverse segond borderline.

Answer:3.Reverse Segond fracture - F - Reverse segond medial tibia avulsion fracture is rare with posterolateral corner injuy. Second fracture (cortical avulsion of the tibial insertion of the middle third of the latural capsular ligament +/- avulsion of iliotibial tract and anterior oblique bland) more common [Stoller, Orth, diag imaging]

  1. Posterolateral corner injury, which is least correct: (TW)
  2. ACL tear - T - typical ACL mechanism of injury (int rotation + valgus). ACL tears > PCL.
  3. PCL tear - T - less common than ACL, but an association.
  4. Reverse Segond fracture - F - Reverse segond medial tibia avulsion fracture is rare with posterolateral corner injuy. Second fracture (cortical avulsion of the tibial insertion of the middle third of the latural capsular ligament +/- avulsion of iliotibial tract and anterior oblique bland) more common [Stoller, Orth, diag imaging]
  5. ???? Transverse meniscal ligament tear - T /F?
    * LW: i cant find much literature about tears in this ligament let alone correaltion with OLC injuries. There is an association with the transverse ligemtn resulting in tears in the medial meniscus. So this is alfo favored to be FALSE.

5.Meniscocapsular ligament separation - T - often assoc with ACL type injury.

PLC: Provides posterolateral stabilisation of the knee - important as if missed, can lead to fail in cruciate ligament repair etc.

Posterolateral complex includes - LCL, popliteal tendon, arcuate lig, popliteofibular lig, fabellofibular lig, posterolateral capsule, lateral head of gastroc muscle, biceps femoris tendon and ITB.

  • 4 different types of intermeniscal ligaments - ant transverse meniscomeniscal lig (58%), post transverse meniscomeniscal lig (1-4%), oblique meniscomeniscal ligs (med and lat) are seen in 1-4% of knees
138
Q

138.Blount’s disease, which is true:

  1. Osteonecrosis of the proximal tibia
  2. Normal metaphyseal-diaphyseal angulation at birth 10-12 degrees
  3. Almost always bilateral
  4. Adolescent type more commonly bilateral
A
  • *LJS Answers:
    3. Almost always bilateral: True
  • *LJS: infantile form is commonly bilateral (50-75%) whilst adolescent form is typically unilateral**
  • LW - Radiopedia states Blounts is commonly bilateral.

72.Blount’s disease, which is true: (most common in obese black children) (TW)

  1. Osteonecrosis of the proximal tibia T – medial tibia vara / varus. Osteonecrosis/AVN of the medial tibial condyle.
    * *LJS - NOT osteonecrosis. Is a developmental lesion of disordered endochondral ossification due to compressive forces at medial physis - statdx and radiopedia**

2.Normal metaphyseal-diaphyseal angulation at birth 10-12 degrees - F - 5 degrees +/- 2.8 degrees. Blount disease angle is 16 +/- 4. 3 degrees. Suggested that >11 degrees is Blount. Note - not to be confused with varus angulation (ie genu varus) where 10-15 degrees can be normal at birth.

  1. Almost always bilateral - F - usually unilateral or asymmetric (unlike developmental bowing which is typically symmetric).
    * *LJS: infantile form is commonly bilateral (50-75%) whilst adolescent form is typically unilateral**

4.Adolescent type more commonly bilateral - F – relatively speaking infant type is more commonly bilateral (80%) cf late onset cases (50%). eMed Abnormal stress placed on the posteromedial prox tibial epiphyses that leads to growth suppression. Predisposing factors for development - obesity, early walking, and black ancestry. Black kids noted to have excessive ligamentous laxity, and they being to walk at earlier age.

139
Q

139.ACL injuries, which is false:

  1. Bone bruising ?lateral anterior femoral condyle
  2. MCL
  3. Iliotibial band #
  4. ? Meniscal abnormal
A

*LW: I think all are true:
ITB fracture refers to avulsion at Gerdy’s tubercle. Although Segond fracture is classically associated with ACL, at Lateral tibial plateau, there is a small band of Iltiotibial band inserts here so may be involved in both a segued fracture, as well as in addition to radiographic article states: At MR imaging, avulsion and retraction of the iliotibial band from its distal insertion on the Gerdy tubercle are visualised. Associated injury to the ACL is a common finding in this entity.

Previous answers:
73.ACL injuries, which is false: (JS)

  1. Bone bruising lateral anterior femoral condyle T – occurs in a pivot shift injury. The location of bone bruising on the lateral femoral condyle depends on the degree of flexion during the injury – increased flexion gives more posterior bone bruising. (Radiographics 2000; 20:S135-S151)
  2. MCL T
  3. Iliotibial band # F – not commonly associated with pivot-shift type injuries.
  4. Meniscal abnormality T - can be associated with either medial or lateral mensical tears
140
Q

140.Radiation necrosis and malignant transformation (T/F): which is true

  1. 25 year latency
  2. Chronic necrosis is cold on bone scan
  3. Stability of appearance on Xray suggests necrosis and differentiates from sarcoma
  4. Chondrosarcoma most common
  5. Does not occur in absence of radiation necrosis
A

anwer: 2.Chronic necrosis is cold on bone scan T – bone scan in osteoradionecrois is typically cold in the radiation field
74. Radiation necrosis and malignant transformation T/F: (JS)
1. 25 year latency F ? – average latency is 11-14 years but the range is 5-55 years. Occurs with a minimum dose of 1600 – 3000 rads. (Dahnert)
2. Chronic necrosis is cold on bone scan T – bone scan in osteoradionecrois is typically cold in the radiation field
3. Stability of appearance on Xray suggests necrosis and differentiates from sarcoma. T - bone changes within the radiation field, the absence of soft tissue mass and stability over time will usually allow the accurate diagnosis of osteoradionecrosis. So essentially if there are x-ray change it suggest necrosis - i.e stability on x-ray mean less likelihood of necrosis or sarcoma (Radiographics 1998:18;1125-1136). Radiation-induced sarcomas demonstrate rapid progression of lesion. (Dahnert)
4. Chondrosarcoma most common F – most common type is osteosarcoma, also fibrosarcoma and malignant fibrous histiocytoma. (Dahnert)
5. Does not occur in absence of radiation necrosis F – by definition a radiation-induced sarcoma arises within the radiation field and is histologically different from the original lesion.

141
Q

141.Shoulder tear / rotator cuff, which is false:

  1. <7mm acromiohumeral distance
  2. Hill Sacks seen best on external rotation
  3. Supraspinatus calcification best seen on external rotation
  4. Os acrominale not associated with
  5. Posterior supraspinatus tear most ?common
A
  • LW: Option 2: Hill Sachs on ext rotation - best on internal rotation.
    1. <7mm acromiohumeral distance: true
    2. Hill Sachs seen best on external rotation: False: best seen on Internal rotation, brings Posterolateral humeral head more into profile, next followed by lateral or Stryker view.. (Radiopedia).
    3. Supraspinatus calcification best seen on external rotation: True, best seen in external rotation position.
    4. Os acrominale not associated with

5.Posterior supraspinatus tear most ?common: Conflicting evidence regarding this: Chronic tears posterior, acute tears anterior (as discussed below)
TRUE, regarding chronic tears: Degenerative rotator cuff tears most commonly involve a posterior location, near the junction of the supraspinatus and infraspinatus. (https://www.ncbi.nlm.nih.gov/pmc/articles/PMC2945926/). Critical zone is distal supraspinatus tendon approx. 1cm proximal to insertion of foot print, which is most hypo vascular area being the articular surface; hence why articular tears more common than bursal surface tears.

However: conflicting literature, Helms states anterior most fibres are where tears usually begin, and one would assume academics who write tests worship Helms…

Previous answers:
Answer:2.Hill Sachs seen best on external rotation - F - posterolateral defect. AP arm in internal rotation best yield. Ext rotation can obscure defect (Garth view (apical oblique) and Stryker-Notch view is also good).

  1. Shoulder tear / rotator cuff, which is false: (TW)
  2. <7mm acromiohumeral distance - T - humeral head appears superior to its normal articulation with the glenoid and the space b/w glenoid and humeral head <7mm.
  3. Hill Sachs seen best on external rotation - F - posterolateral defect. AP arm in internal rotation best yield. Ext rotation can obscure defect (Garth view (apical oblique) and Stryker-Notch view is also good).
  4. Supraspinatus calcification best seen on external rotation - T - (Frontal views). Note that with US internal rotation is better to visualise SS tendon.
  5. Os acrominale not associated with - T - rare cause of rotator cuff tears and impingement syndrome
  6. Posterior supraspinatus tear most common - F - articular surface
142
Q

142.MRI shoulder, which is true:

  1. Increased signal in the supraspinatous tendon seen only on proton density indicates a tear
  2. Fluid in the subdeltoid bursa indicates a tear
  3. The subdeltoid fat plane is abnormal in significant tears of the rotator cuff
  4. The majority of significant tears are not assoc with a joint effusion
  5. Supraspinatus tendon on US better assessed on external rotation
A

answer: 3.The subdeltoid fat plane is abnormal in significant tears of the rotator cuff - T - subacromial and subdeltoid peribursal fat changes - fat may be replaced by either low signal-intensity granulation tissue or scar or bright-signal-intensity fluid. Alterations in the peribursal fat plane and proximal musculotendinous junction are present in up to 92% of complete tears (MRI in ortho and sports med, Stoller).
76. MRI shoulder, which is true: (TW)
1. Increased signal in the supraspinatous tendon seen only on proton density indicates a tear - F - DDx tendinopathy / tendinitis; intratendinous cyst; magic angle (short TE)
2. Fluid in the subdeltoid bursa indicates a tear - F - fluid with any inflam process. Rotator cuff impingement, glenoid labrum abnormality, bursitis, supraspinatis tendinitis (Skeletal radiol)
3. The subdeltoid fat plane is abnormal in significant tears of the rotator cuff - T - subacromial and subdeltoid peribursal fat changes - fat may be replaced by either low signal-intensity granulation tissue or scar or bright-signal-intensity fluid. Alterations in the peribursal fat plane and proximal musculotendinous junction are present in up to 92% of complete tears (MRI in ortho and sports med, Stoller).
4. The majority of significant tears are not assoc with a joint effusion - F - Often found assoc with tears, but as an isolated finding has low sensitivity and specificity. When joint effusion combined with fluid in the subacromial/subdeltoid bursa - highly specific with high PPV for rotator cuff tears (AJR).
5. Supraspinatus tendon on US better assessed on external rotation F – internally rotate shoulder - exposes the SST by bringing it out from underneath the ACJ. Subscap best seen on external rotation.CME 03.74 (8)

143
Q
  1. Tripod fracture, which is false:
  2. Depressed lateral canthus
  3. Inferior rectus mm trapped in upwards diplopia
  4. Decreased / absent sensation inferior orbital nerve
  5. Zygomatic temporal fracture excludes Le Fort III
  6. # orbital floor as part of
A

Answer:4.Zygomatic temporal fracture excludes Le Fort III - F - includes. LF III “craniofacial disjunction” - fracture line - horizontal course through nasofrontal suture, maxillofrontal suture, orbital wall, zygomatic arch (ie extends through lateral orbital wall through zygoma / zygomaticofrontal suture).

  1. Tripod fracture which is false: (TW)
  2. Depressed lateral canthus - T - lateral canthus attached to the lateral free-fragment. Inferior displacement of the lateral canthal angle may indicate inferior migration of the fractured zygomatic bone (eMed).
  3. Inferior rectus mm trapped in upwards diplopia - T - entrapment of the inferior rectus & inferior oblique muscles can cause diplopia on upward gaze, and cause limited downgaze ability. Dahnert 6th ed / Handbook of Ocular disease management (Note that for some reason this answer (inferior oblique instead of rectus) was considered false in the 1999 CME)
  4. Decreased / absent sensation inferior orbital nerve - T - fracture along ZM suture through lateral maxillary buttress almost invariable traverses the infraorbital nerve foramen (Radiographics 2006)
  5. Zygomatic temporal fracture excludes Le Fort III - F - includes. LF III “craniofacial disjunction” - fracture line - horizontal course through nasofrontal suture, maxillofrontal suture, orbital wall, zygomatic arch (ie extends through lateral orbital wall through zygoma / zygomaticofrontal suture).
  6. # orbital floor as part of - T - fractures of lateral wall of maxillary sinus, orbital rim close to infrorbital foramen, floor of orbit, zygomaticofrontal suture / zygomatic arch. CME 99.40
144
Q

144.Which is false re: bone infections:

  1. Neonatal viral infection with rubella produces vertically orientated radiolucent metaphyseal bands
  2. Brodies abscess is diaphyseal
  3. Osteomyelitis in infants extends to the epiphysis
  4. Sclerosing Osteomyelitis of Garre is most often in the mandible
  5. Echonococcus typically produces expansile lytic bone lesions
A

ANSWER:2.Brodies abscess is diaphyseal F – Typically occurs in the ends of tubular bones, metaphysis but can be in epiphysis in children and infants. (Dahnert)

  1. Which is false re: bone infections: (JS)
  2. Neonatal viral infection with rubella produces vertically orientated radiolucent metaphyseal bands T – “celery stalk” sign – metaphyseal irregular margins and coarsened trabeculae extending longitudinally from epiphysis. (Dahnert)
  3. Brodies abscess is diaphyseal F – Typically occurs in the ends of tubular bones, metaphysis but can be in epiphysis in children and infants. (Dahnert)
  4. Osteomyelitis in infants extends to the epiphysis T – see above
  5. Sclerosing Osteomyelitis of Garre is most often in the mandible T – Sterile osteomyelitis, with a low-grade nonnecrotic nonpurulent infection, typically in mandible, focal bulge of thickened cortex with sclerosing periosteal reaction. (Dahnert).
  6. Echinococcus typically produces expansile lytic bone lesions T – (Chapman & Nakielny)
145
Q

145.Which is true re: rotator cuff tears:

  1. Commonly associated with type 1 acromion
  2. Most often in post third of supraspinatus tendon
  3. Typically echogenic
  4. Full thickness tears found in at least 10% of asymptomatic individuals >80 years
  5. Absence of fluid in subacromial / subdeltoid bursa excludes diagnosis
A

ANSWER:4.Full thickness tears found in at least 10% of asymptomatic individuals >80 years T – can’t find a reference for this!

  1. Which is true re: rotator cuff tears: (JS)
  2. Commonly associated with type 1 acromion F – A higher prevalence of rotator cuff tears is seen with Type 3 (hooked downward anteriorly) and possibly Type 2 (curved downward) acromia. (Radiographics 2006;26:1045-1065).
  3. Most often in post third of supraspinatus tendon F - most cuff tears start in the anterior fibres: *LW False according to Helms (states starts anteriroly). Recent literature shows in chronic degenerative tears, it starts posteriorly. For exam would probably go with Helms….
  4. Typically echogenic F – tears are generally hypoechoic. Hyperechoic foci within tendons may represent calcification, scar tissue, synovitis or haemorrhage. (Radiographics 2006;26;589-604).
  5. Full thickness tears found in at least 10% of asymptomatic individuals >80 years T – can’t find a reference for this!
  6. Absence of fluid in subacromial / subdeltoid bursa excludes diagnosis F – presence of fluid in the SA/SD bursa indicates a full-thickness tear, absence of fluid can be seen with a partial thickness tear.
146
Q

146.Which is true re: BMD estimation using DEXA:

  1. T score compares individuals BMD to age matched mean
  2. BMD abnormal if at least 1.5SD below the mean of the young normal population
  3. T score -2.2 indicates osteoporosis
  4. For each SD decrease in BMD there is a doubling of fracture risk
  5. A decrease in hip BMD of 3% over one year is considered statistically significant
A

True = 2 + 4 (AJL and LW agree)

  1. Which is true re: BMD estimation using DEXA: (JS)
  2. T score compares individuals BMD to age matched mean - F – compares the individual against peak young normals of the same sex in standard deviations.
  3. BMD abnormal if at least 1.5 SD below the mean of the young normal population - T implies osteopenia, which is abnormal. as T score between – 1 and -2.5 is osteopenia
  4. T score -2.2 indicates osteoporosis - F - T score 1-2.4 = osteopenia. T score = 2.5 or below = osteoporosis
  5. For each SD decrease in BMD there is a doubling of fracture risk - T – for each T score SD below normal, the fracture risk is increased by between 2 and 3 (Brandt and Helms and other journal articles)
  6. A decrease in hip BMD of 3% over one year is considered statistically significant - F – a change of up to 3% can be due to inherent measurement error
147
Q

147.Which is false re: MRI knee:

  1. Normal ACL perpendicular to Bloomenstat’s line
  2. Normal PCL subject to magic angle effect
  3. Tears of the posterior horn of medial meniscus more common than anterior
  4. The lateral facet of the patella is broader than the medial facet
  5. MCL tear and ACL disruption are associated
A

ANSWER:1.Normal ACL perpendicular to Bloomenstat’s line - F - parallel to.

  1. Which is false re: MRI knee: (TW)
  2. Normal ACL perpendicular to Bloomenstat’s line - F - parallel to.
  3. Normal PCL subject to magic angle effect - T - femoral side of PCL often oreintated close to magic angle.
  4. Tears of the posterior horn of medial meniscus more common than anterior - T - most commonly injured meniscus is medial. Lateral meniscus is less commonly injured as it has greater mobility (Primer). 60% of medial meniscus tears are posterior.(D 6th ed)
  5. The lateral facet of the patella is broader than the medial facet - T
  6. MCL tear and ACL disruption are associated - T - Longitudinal mensical tear assoc with ACL injury in up to 40% (D 6th ed) Magic angle phenomenon causes increased signal on short echo time (TE) images. It affects tissues with well-ordered collagen fibres in one direction (eg tendon or articular hyaline cartilage) which behave in an anisotropic manner in a magnetic field. The induced hyperintense signal is dependent on the orientation of the tissue to the main magnetic field (B0), with max signal intensity observed at the “magic angle” of 54.74 degrees to B0.
148
Q

148.In an infant with hyperparathyroidism, which is not a finding:

  1. Osteoporosis
  2. Fractures
  3. Periosteal elevation
  4. Subperiosteal resorption
  5. Resorption of the distal phalangeal tufts
A

All appear true:
https://www.sciencedirect.com/science/article/pii/S0378603X16301681.
Pathological fractures can occur secondary to marked osteoporosis.

Probably 2???

  1. Osteoporosis - T (D6th pg 3)
  2. Fractures - T - ?pseudo # looser zone
  3. Periosteal elevation - T - Can be seen in congenital primary hyperparathyroidism. Periosteal elevation is common, and when it is severe, the long bones may actually look cloaked with new bone (# in Children, Rockwood and Wilkins’)
  4. Subperiosteal resorption - T - subperiosteal bone resorption
  5. Resorption of the distal phalangeal tufts - T - Phalagenal tuft bone resorption is earliest involvement.
149
Q
  1. Which is not a cause of an Erlenmeyer flask deformity:
  2. Thalassemia
  3. Achondroplasia
  4. Hypophosphatasia
  5. Osteopetrosis
  6. Neurofibromatosis
A

ANSWER:5.Neurofibromatosis - F

83.Which is not a cause of an Erlenmeyer flask deformity: (TW)

  1. Thalassemia - T
  2. Achondroplasia - T
  3. Hypophosphatasia - T
  4. Osteopetrosis - T
  5. Neurofibromatosis - F Erlenmeyer flask deformity :Expansion of distal end of long bones, usually femur
Causes of Erlenmeyer Flask deformity 
- TOP DOG
Thalassemia, 
Osteopetrosis, 
Pyle disease, 
Diaphyseal aclasis (MHE), 
Ollier disease, 
Gaucher disease. 
Others Niemann-Pick. 
Hemolytic anaemia. 
Heavy metal poisoning. 
Rickets. 
Fibrous dysplasia, 
Down syndrome, 
Achondroplasia. 
Rheumatoid arthritis. 
Hypophosphatasia.
 Leukemia.
150
Q
  1. Which is false re: SUFE:
  2. Initial slip is posterior
  3. Bilateral in 20-40%
  4. AVN in 15%
  5. Chondrolysis is reversible
A

ANSWER:4.Chondrolysis is reversible - F - acute cartilage necrosis - occurs 7-10% = rapid loss of >50% of thickness of cartilage. Joint space <3mm

  1. Which is false re: SUFE: (TW)
  2. Initial slip is posterior - T - posteromedial displacement of the head (acute slip).
  3. Bilateral in 20-40% - T - 20-37% bilateral
  4. AVN in 15% - T - 10-15% AVN of femoral head: risk increases with advanced degree of slip, delayed surgery for acute slip, anterior pin displacement, lots of pins, subcapital osteotomy.
  5. Chondrolysis is reversible - F - acute cartilage necrosis - occurs 7-10% = rapid loss of >50% of thickness of cartilage. Joint space <3mm
151
Q

151.Regarding imaging of paediatric cervical spine, which is true:

  1. Atlantoaxial dislocation occurs less in children
  2. Anterior wedging of C3 can be a normal variant
  3. Normal CT excludes spinal cord injury
  4. Displacement of lateral mass of C1/2 of >6mm is normal
  5. Injury to the cervical spine is more common in the lower than the upper spine
A

Answer:2.Anterior wedging of C3 can be a normal variant - T – normal variant, however age dependent: eg, anterior wedging in a 3yo at C5 is likely normal variant. However wedged body at C5 in an 8yo this is likely pathologically compressed.

  1. Regarding imaging of paediatric cervical spine, which is true: (TW)
  2. Atlantoaxial dislocation occurs less in children - F - occurs 5x more common in children than adults.
  3. Anterior wedging of C3 can be a normal variant - T – normal variant, however age dependent: eg, anterior wedging in a 3yo at C5 is likely normal variant. However wedged body at C5 in an 8yo this is likely pathologically compressed.
  4. Normal CT excludes spinal cord injury - F - SCIWORA - most studies of traumatic myelopathy in children report an incidence of SCIWORA greater than 20% (Medscape). Range 5-65%.
  5. Displacement of lateral mass of C1/2 of >6mm is normal - F - >2mm bilateral is always abnormal. >1-2mm or unilateral displacement can be due to head tilt / rotation. Total offset (sum of both sides) of more than 6mm of the lateral masses of the atlas with respect to the odontoid is highly suggestive of rupture of the transverse ligament or avulsion of its attachments.
  6. Injury to the cervical spine is more common in the lower than the upper spine - F – in patients under 9yo, almost all injures are to the occiput-C2 region (Imaging of Spinal Trauma in Children, Kuhns). Differences between adult and paeds spinal injury (Spinal Trauma in Children, Paeds Radiol 2001) Fulcrum of movement located at C2-3 in the child, C5-6 in adult Relatively large head and weak neck muscles Ligamentous and joint capsule laxity Horizontal orientation of the facet joints in younger children Underdeveloped uncinate processes Mild physiological anterior ‘wedging’ of vertebral bodies Incomplete ossification of odontoid process
152
Q

152.Tripod fracture, which is most correct:

  1. The orbital roof is fractured
  2. The medial canthus of the eye is displaced inferiorly
  3. Majority of these fractures are due to punch injuries
  4. Diplopia on both upward and downward gaze suggests entrapment of the inferior rectus muscle
  5. Separation of the frontozygomatic suture excludes co existing Le Fort III
A

ANSWER:4.Diplopia on both upward and downward gaze suggests entrapment of the inferior rectus muscle - T - entrapment of the inferior rectus & inferior oblique muscles can cause diplopia on upward gaze, and cause limited downgaze ability. Dahnert 6th ed / Handbook of Ocular disease management (Note that for some reason this answer (inferior oblique instead of rectus) was considered false in the 1999 CME)

  1. Tripod fracture, which is most correct:? (TW)
  2. The orbital roof is fractured - F - fractures of lateral wall of maxillary sinus, orbital rim close to infrorbital foramen, floor of orbit, zygomaticofrontal suture / zygomatic arch
  3. The medial canthus of the eye is displaced inferiorly - F - lateral canthus. Medial canthus involved in nasoethmoidal fractures. Inferior displacement of the lateral canthal angle may indicate inferior migration of the fractured zygomatic bone.
  4. Majority of these fractures are due to punch injuries - F - studies show that 80% of these injures are due to motor vehicle accidents (eMedicine).
  5. Diplopia on both upward and downward gaze suggests entrapment of the inferior rectus muscle - T - entrapment of the inferior rectus & inferior oblique muscles can cause diplopia on upward gaze, and cause limited downgaze ability. Dahnert 6th ed / Handbook of Ocular disease management (Note that for some reason this answer (inferior oblique instead of rectus) was considered false in the 1999 CME)
  6. Separation of the frontozygomatic suture excludes co existing Le Fort III - F - includes. LF III “craniofacial disjunction” - fracture line - horizontal course through nasofrontal suture, maxillofrontal suture, orbital wall, zygomatic arch (ie extends through lateral orbital wall through zygoma / zygomaticofrontal suture).
153
Q

153.Unicameral bone cyst, which is most correct:

  1. Are more common in the femur than the humerus
  2. Involve the epiphysis in about 30% of cases
  3. Contain fluid fluid levels on MRI in greater than 50% of cases
  4. Arise in the metaphysic
  5. Fallen fragment sign may be seen in half of cases
A

ANSWER:4.Arise in the metaphysis - T - see 2

87.Unicameral bone cyst, which is most correct: (TW)

. 1.Are more common in the femur than the humerus - F - Majority occur in the proximal femur and humerus (60-75%) (Dahnert 6th ed.). 55-65% SBCs occur in humerus, and 25-30% in the femur. In patients older than 20yo, SBCs are found more commonly in flat bones (iliac, calcaneus) (eMedicine).

  1. Involve the epiphysis in about 30% of cases - F - intramedullary, central , metaphyseal. Migrate into diaphysis with growth. Does not cross epiphyseal plate. (Dahnert 6th ed).
  2. Contain fluid fluid levels on MRI in greater than 50% of cases - F - Not best answer, however not sure about this statement. Says fluid-fluid levels prob occur with path #, of which Dahnert says 65% have path fracture however, that’s presuming every SBC in a population is identified. (D 6th ed.)
  3. Arise in the metaphysis - T - see 2.
  4. Fallen fragment sign may be seen in half of cases - F - see if fractured. 20% cases. Presently, the most popular theory is that of venous outflow obstruction in bone, as proposed by Cohen, which results in an elevated intraosseous pressure. Increased pressure - leads to osteoclastic activation for resorption of bone, presumably from the elevated cytokines (prostaglandins, interleukin 1[beta], and gelatinase) released by the endothelial cells lining the cyst wall.
154
Q

154.With regards to red marrow, which bones converts to fatty marrow first:

  1. Spine
  2. Ribs
  3. Pelvis
  4. Proximal humerus
  5. Femoral diaphysis
A

5.Femoral diaphysis - T Red or cellular marrow is hematopoietcally active. Hematopoietically inactive yellow marrow is composed of fat cells. RM signal iso- / slightly hyper to muscle T1 and T2. YM iso with subcutaneous fat T1, hyperintense to muscle on T2, and iso- / slightly lower than subcut fat. Birth - marrow predominantly hematopoietically active cells. Orderly and predictable conversion - begins in appendicular/peripheral skeleton and progresses to the axial/central skeleton. In long bones, marrow conversion first in diaphysis, then distal metaphyses, and finally proximal metaphyses. Adult marrow pattern reached ~25y - at this time see red marrow in axial skeleton (skull, spine, sternum, flat bones) and prox ends of humeri and femurs), yellow marrow elsewhere. May see subchondral islands of red marrow in proximal humeral epiphyses.

155
Q

155.Parosteal osteosarcoma, which is false:

  1. Commonly third and fourth decades
  2. Diaphysis of long bones is commonest site
  3. Commonly in femur
  4. Better prognosis than other osteosarcoma
  5. Has a radiolucent region between tumour and bone
A

ANSWER:2.Diaphysis of long bones is commonest site - F – metaphysis in 80-90%. Typical site is posterior aspect of distal femur (50-65%). (PERIosteal sarcoma - anteromedial diaphysis of proximal tibia + middle / distal femur).

  1. Parosteal osteosarcoma, which is false: (TW)
  2. Commonly third and fourth decades - T – peak age is ~40yo. 50% > 30yo.
  3. Diaphysis of long bones is commonest site - F – metaphysis in 80-90%. Typical site is posterior aspect of distal femur (50-65%). (PERIosteal sarcoma - anteromedial diaphysis of proximal tibia + middle / distal femur).
  4. Commonly in femur - T - posterior distal femur most common site. Either end of tibia, proximal humerus, fibula. Rare in other long bones.
  5. Better prognosis than other osteosarcoma - T - Parosteal is best > Periosteal > Conventional OSA.
  6. Has a radiolucent region between tumour and bone - T - “String sign” - initially fine radiolucent line separating tumor mass from cortex (30-40%). Dahnert 6th ed.
156
Q

156.Osteosarcoma: which one is most correct

  1. Of the jaw occurs in an older age group than osteosarcoma elsewhere
  2. Is extraskeletal in 10-20% of cases
  3. Arises in the diaphyseal medullary canal in the majority of cases
  4. Of the telangiectatic variety manifests fluid fluid levels on MR in about 50% of cases
  5. Occurs around the knee in 90%
A

ANSWER:1.Of the jaw occurs in an older age group than osteosarcoma elsewhere - T (best option) - gnathic osteosarcoma - avg age 34yo. Is generally said 3rd-4th decade. Of note Dahnert says parosteal osteosarcoma peak age 38yo. This answer is T as per the CME’s.

  1. Osteosarcoma: (TW)
  2. Of the jaw occurs in an older age group than osteosarcoma elsewhere - T (best option) - gnathic osteosarcoma - avg age 34yo. Is generally said 3rd-4th decade. Of note Dahnert says parosteal osteosarcoma peak age 38yo. This answer is T as per the CME’s.
  3. Is extraskeletal in 10-20% of cases - F - 4% of osteosarcomas.
  4. Arises in the diaphyseal medullary canal in the majority of cases F – conventional osteosarcoma - origin in metaphysis 90-95% / diaphysis 2-11%.
  5. Of the telangiectatic variety manifests fluid fluid levels on MR in about 50% of cases - F - fluid-fluid levels in 90%.
  6. Occurs around the knee in 90% - F - conventional OSA 50-55%, other subtypes vary and although low-grade intraosseous osteosarcoma if often about the knee, still ~50% of all osteosarcoma occur near the knee.
157
Q

157.Which of the following statements regarding rib notching is false:

  1. Unilateral rib notching with aberrant right subclavian
  2. In hyperparathyroidism, it is typically on the upper surface of one or more ribs
  3. NF upper or lower
  4. Superior rib erosions may be seen in normal individuals with increasing age
  5. Rib notching regresses following successful repair of coarctation in children
A

ANSWER:1.Unilateral rib notching with aberrant right subclavian - F - Can be unilateral when assoc with coarctation.Unilateral left rib nothcing when there is coarctation proximal to aberrant right SCA. Unilateral right sided if coarctation proximal to left SCA.

  1. Which of the following statements regarding rib notching is false: (TW)
  2. Unilateral rib notching with aberrant right subclavian - F - Can be unilateral when assoc with coarctation.Unilateral left rib nothcing when there is coarctation proximal to aberrant right SCA. Unilateral right sided if coarctation proximal to left SCA.
  3. In hyperparathyroidism, it is typically on the upper surface of one or more ribs - T (D 6th)
  4. NF upper or lower - T - Chapman & Nak
  5. Superior rib erosions may be seen in normal individuals with increasing age - T (CME)
  6. Rib notching regresses following successful repair of coarctation in children - T (CME) Variation of CME 01.04
158
Q

158.In zygomaticomaxillary ‘tripod’ fracture which is false:

  1. The orbital floor is fractured
  2. The lateral canthus of the eye may be displaced inferiorly
  3. Numbness over the ipsilateral cheek indicates infraorbital nerve damage
  4. Diplopia on upward gaze suggests entrapment of the inferior rectus muscle
  5. Fracture of the frontozygomatic suture exludes Le Fort III
A

5.Fracture of the frontozygomatic suture exludes Le Fort III

159
Q

159.Regarding shoulder imaging: which one is true?

  1. Rotator cuff calcification commonest plain film finding of rotator cuff impingement
  2. Acromio humeral distance normally <7mm
  3. Os acromiale not associated with impingement
  4. Supraspinatus calcification is best seen on external rotation
  5. Hill Sachs defect best seen on films in external rotation
A
  • LW:
    4. Supraspinatus calcification best seen on external rotation: True, best seen in external rotation position.

note that with US internal rotation is better to visualise SS tendon.

  1. Regarding shoulder imaging: (TW)
  2. Rotator cuff calcification commonest plain film finding of rotator cuff impingement F - radiographic finding usually normal in acute setting. Impingement can be suggested radiographicallly by abnormalities that encroach on the suprspinatous outlet, eg spur formation on undersurface of ACJ or reduced acromioclavicular distance / hooked acromion.
  3. Acromio humeral distance normally <7mm - F - SS tear: can’t stabliize humerus and oppose upward traction exerted on humerus by deltoid - humerus eventually migrates superiorly and is high-riding (<7mm acromiohumeral distance) on AP view. Also may be = 2mm on active abduction views.
  4. Os acromiale not associated with impingement - F - rare cause of rotator cuff tears and impingement syndrome
  5. Supraspinatus calcification is best seen on external rotation - T - note that with US internal rotation is better to visualise SS tendon.
  6. Hill Sachs defect best seen on films in external rotation - F - posterolateral defect. AP arm in internal rotation best yield. Ext rotation can obscure defect (Garth view (apical oblique) and Stryker-Notch view is also good).
160
Q

160.Regarding tarsal coalition, which is false:

  1. It typically presents in adolescent patients
  2. Talo navicular coalition is the most frequently occurring variety
  3. Talo calcaneal coalition most commonly involves the middle facet of the subtalar articulation
  4. Bone marrow oedema is demonstrated adjacent to fibrous coalitions on MRI
  5. The coalition may be cartilaginous
A

ANSWER:2.Talo navicular coalition is the most frequently occurring variety - F - CN 45%, TC 45% (remainder are talo-nav, calc-cuboid, cubo-nav)

  1. Regarding tarsal coalition, which is false: (TW)
  2. It typically presents in adolescent patients - T - CN coalition 8-12yo, TC coalition 12-16yo.
  3. Talo navicular coalition is the most frequently occurring variety - F - CN 45%, TC 45% (remainder are talo-nav, calc-cuboid, cubo-nav)
  4. Talo calcaneal coalition most commonly involves the middle facet of the subtalar articulation -T - middle facet at level of sustentaculum tali most frequently
  5. Bone marrow oedema is demonstrated adjacent to fibrous coalitions on MRI -T - reactive periarticular bone changes and bone marrow oedema along fused joint.
  6. The coalition may be cartilaginous - T - osseous, cartilaginous, fibrous coalition.
161
Q

161.Os tibiale externum, which is incorrect?

  1. Os peroneum is more common
  2. Symptoms most common during adolescence
  3. Surgical correction is an option
  4. Lateral to the navicular
A

ANSWER:4.Lateral to the navicular - F - Medial. Type I - true sesamoid within TP tendon. Type II - triangular or heart shaped. Type III - enlarged medial extension of navicular.

  1. Os tibiale externum, which is incorrect? (TW)
  2. Os peroneum is more common - T - Os peroneum is most common, Os tibiale externum (ie accessory navicular) is 2nd most common.
  3. Symptoms most common during adolescence - T -Symptoms after 5y of age. Os tibiale externum ossifies age 9-11y. Symptoms most common during adolescence (prob as occur post local trauma to foot and ankle)
  4. Surgical correction is an option- T - Surgical treatment (Type I - rmove ossicle from tib post tendon; Type II - resect entire non-ossified synchondrosis or bone graft + arthordesis; Type III surgical remodeling / recontruction). Conervative Rx (orthotic, shielding, shoe modifications)
  5. Lateral to the navicular - F - Medial. Type I - true sesamoid within TP tendon. Type II - triangular or heart shaped. Type III - enlarged medial extension of navicular.
162
Q

162.The following statements are correct concerning unilateral facet joint dislocation:

  1. The mechanism of injury (vector force) is simultaneous hyperextension and rotation
  2. There is disruption of the anterior longitudinal ligament
  3. There is forward subluxation of the involved vertebra of a distance equal to or greater than half of the AP diameter of the body
  4. The optimum plain radiographic view for diagnosis is the ipsilateral oblique
  5. The injury is mechanically unstable
A

ANSWER:4.The optimum plain radiographic view for diagnosis is the ipsilateral oblique - T - oblique views. May see ‘bow-tie’ sign on lateral projection. Oblique projection would show interruption of the normal ‘shingles on a roof’ appearance.

  1. The following statements are correct concerning unilateral facet joint dislocation: (TW)
  2. The mechanism of injury (vector force) is simultaneous hyperextension and rotation - F - flexion rotation injury. Most common levels C5-6, C6-7.
  3. There is disruption of the anterior longitudinal ligament - F
  4. There is forward subluxation of the involved vertebra of a distance equal to or greater than half of the AP diameter of the body - F - anteriolisthesis <1/4 vertebral body width (cf bilateral facet lock where anteriolisthesis by 1/2 vertebral body width - Dahnert 6th ed). Pocket Radiol ER-trauma - says <1/2 vertebral body width for unilat facet dislocation.
  5. The optimum plain radiographic view for diagnosis is the ipsilateral oblique - T - oblique views. May see ‘bow-tie’ sign on lateral projection. Oblique projection would show interruption of the normal ‘shingles on a roof’ appearance.
  6. The injury is mechanically unstable - F - stable injury (Dahnert 6th ed).
163
Q
  1. In fractures of the petrous temporal bone, which is false:
  2. Longitudinal fracture (parallel to petrous ridge) with conductive hearing loss
  3. Transverse fracture (perpendicular to petrous ridge) with complete facial nerve palsy
  4. Longitudinal fracture with low incidence of CSF leak
  5. Transverse fracture with sensorineural hearing loss
A

ANSWER:3.Longitudinal fracture with low incidence of CSF leak - F - 15% of temporal bone fractures have associated CSF leak (Dahnert 6th ed). CSF otorhinorrhoea is common, but usually temporary (eMedicine).

  1. In fractures of the petrous temporal bone, which is false: (TW)
  2. Longitudinal fracture (parallel to petrous ridge) with conductive hearing loss - T - Longitudinal fractures 75% - dislocation of auditory ossicles. No neurosensory hearing loss. Facial nerve palsy 20% due to oedema / fracture of facial canal near geniculate gangion (frequent recovery) - (Dahnert 6th ed)
  3. Transverse fracture (perpendicular to petrous ridge) with complete facial nerve palsy - T - Transverse fracture 25% - neurosensory hearing loss (fracture across apex IAC), facial nerve palsy 50% (IAC injury) with less frequent spontaneous recovery due to disruption of nerve fibers.
  4. Longitudinal fracture with low incidence of CSF leak - F - 15% of temporal bone fractures have associated CSF leak (Dahnert 6th ed). CSF otorhinorrhoea is common, but usually temporary (eMedicine).
  5. Transverse fracture with sensorineural hearing loss - T - see ans 1.
164
Q

164.With regard to imaging the scaphoid bone, which is false:

  1. In the majority of normal subjects the periscaphoid fat plane is present on plain film radiography
  2. The optimal timing of scinitgraphy to detect a fracture is less than 24 hours from injury
  3. Approximately 70% of fractures occur through the waist
  4. MR imaging is the most sensitive modality to assess for avascular necrosis
A

ANSWER:2.The optimal timing of scinitgraphy to detect a fracture is less than 24 hours from injury - F - Bone scan PPV 93% after 2-3d.

  1. With regard to imaging the scaphoid bone, which is false: (TW)
  2. In the majority of normal subjects the periscaphoid fat plane is present on plain film radiography - T - absence or displacement of navicular fat stripe should raise suspicion of scaphoid #.
  3. The optimal timing of scinitgraphy to detect a fracture is less than 24 hours from injury - F - Bone scan PPV 93% after 2-3d.
  4. Approximately 70% of fractures occur through the waist - T - Prox 1/3 20% (failure to reunite 90%), middle 1/3 70% (failure to reunite 30%), distal 1/3 20% (usually fragments reunite).
  5. MR imaging is the most sensitive modality to assess for avascular necrosis - T - MRI more sensitive and specific than radiography or CT. Evaluates for proximal pole T2 hypointensity (AVN), and T1 C+ can document viability of proximal pole with enhancement.
165
Q

165.Melorrheostosis, which is false:

  1. Joint contractures
  2. Skin periarticular calcification
  3. Muscular wasting
  4. Increased risk of malignancy
A

ANSWER:4.Increased risk of malignancy - F - Can be associated with osteopoikilosis, osteopathia striata, tumors / malformations of blood vessels (eg hemaingioma, vascular nevi gloums tumor, AVM,). Doesn’t seem to be increased risk of malignancy. Benign, slowly progressive process.

  1. Melorrheostosis, which is false: (TW)
  2. Joint contractures - T - Flexion contractures of hip and knee. Limited joint motion (bone may encroach on nerves, blood vessels, or joints). May cross joint with joint fusion.
  3. Skin periarticular calcification - T - ossified soft-tissue masses (27%)
  4. Muscular wasting - T - muscle atrophy is frequent.
  5. Increased risk of malignancy - F - Can be associated with osteopoikilosis, osteopathia striata, tumors / malformations of blood vessels (eg hemaingioma, vascular nevi gloums tumor, AVM,). Doesn’t seem to be increased risk of malignancy. Benign, slowly progressive process.
166
Q

166.MRI and ACL injuries, which is false:

  1. Meniscocapsular separation of the posterior horn of the medial meniscus
  2. Dashboard injury associated with ACL tear
  3. Iliotibial band avulsion
  4. Bone contusion anterior aspect of the medial femoral condyle
  5. Posterior horn medial meniscus tear
A

ANSWER:2.Dashboard injury associated with ACL tear - F - PCL injury

  1. MRI and ACL injuries, which is false: (TW)
  2. Meniscocapsular separation of the posterior horn of the medial meniscus - T - see below
  3. Dashboard injury associated with ACL tear - F - PCL injury
  4. Iliotibial band avulsion - T
  5. Bone contusion anterior aspect of the medial femoral condyle - T
    * *LJS: I thought bone contusions were lateral femoral condyle and posterolateral tibia**
  • **LW:
  • most common ACL bone marrow oedema pattern is Pivot Shift, with lateral femoral condyle - postero-lateral tibia.
  • ACL injuries can be associated with other mechanisms, although less commonly, such as counter coup and valgus hyper extension, where there will be more variable bone oedema patterns, including medial femoral condyle, especially if other ligamentous injuries such as MCL.
    (https: //pubs.rsna.org/doi/full/10.1148/radiographics.20.suppl_1.g00oc19s135#F10A)

5.Posterior horn medial meniscus tear - T Some of the possible answers are a bit counterintuitive and are not in the main texts. May have come from Radiology “Bone contusions of the posterior lip of the medial tibial plateau….. 1999).

Typically - ACL injuries - assoc with meniscal injury (lateral > medial).
Bone contusion pattern lateral femoral condyle and posterior aspect of the lateral tibial plateau.
Article looked at pattern with medial contusion pattern + assoc injury: medial compartment bone contusion involving the posterior lip of the medial tibial plateau and, occasionally, the medial femoral condyle occur as contrecoup impaction injuries following ACL tears. these bone contusions are almost always assoc with far peripheral meniscal tears, meniscocapsular contusions, or separations affecting the posterior horn of the medial meniscus.

Radiographics 2008 Avulsion # of the knee - iliotibial band avulsion and retraction from its distal insertion on the Gerdy tubercle - associated injury to the ACL is a common finding in this entity (but vice versa? - still option2 is best choice).

167
Q

167.MRI Knee, which is true:

  1. PCL is normally hyperintense on T1 relatvie to the ACL
  2. ACL perpendicular to Bloomenstat’s line
  3. Both ligaments of Wrisberg and Humphry can be identified in most knees
  4. Sartorius, gracilius, and semitendinosis share a common tendon
A

NSWER:4.Sartorius, gracilius, and semitendinosis share a common tendon - T - pes anserinuus,”goose’s foot” - is the insertion of the conjoined tendons of the 3 muscles onto the anteromedial surface of the proximal tibia. Conjoined tendon lies superficial to the tibial insertion of the MCL. “Say Grace before eating Tea” – Sartorius – Gracilius – Semitendinosis (Med to Lat).

  1. MRI Knee, which is true: (TW) A
  2. PCL is normally hyperintense on T1 relative to the ACL - F - PCL is usually hypointense.
  3. ACL perpendicular to Bloomenstat’s line - F - parallel to Bloomenstat’s line
  4. Both ligaments of Wrisberg and Humphry can be identified in most knees - F - only about 10% of knees have both.
  5. Sartorius, gracilius, and semitendinosis share a common tendon - T - pes anserinuus,”goose’s foot” - is the insertion of the conjoined tendons of the 3 muscles onto the anteromedial surface of the proximal tibia. Conjoined tendon lies superficial to the tibial insertion of the MCL. “Say Grace before eating Tea” – Sartorius – Gracilius – Semitendinosis (Med to Lat).
168
Q

167.The following statement regarding MR of the Achilles tendon is true:

  1. When normal it has a convex anterior margin on axial T1 weighted images
  2. Fluid in the retrocalcaneal bursa is a normal finding
  3. Any intermediate signal intensity within the tendon on proton density images is abnormal
  4. It is typically ruptured at the calcaneal insertion site
  5. It contains foci of increased T2 signal in all cases of chronic tendinopathy
A

ANSWER:2.Fluid in the retrocalcaneal bursa is a normal finding T - teardrop-shaped structure normally located between the tendon and the posterior aspect of the upper calcaneus; it has little or no fluid within it when not inflamed.

  1. The following statement regarding MR of the Achilles tendon is true: (GC, TW)
  2. When normal it has a convex anterior margin on axial T1 weighted images F - normally flat or concave; if it becomes diffusely convex, an abnormally thickened tendon exits.
  3. Fluid in the retrocalcaneal bursa is a normal finding T - teardrop-shaped structure normally located between the tendon and the posterior aspect of the upper calcaneus; it has little or no fluid within it when not inflamed.
  4. Any intermediate signal intensity within the tendon on proton density images is abnormal F - see below PD image, intermediate SI reflects derangement in tendon fibre structure.
  5. It is typically ruptured at the calcaneal insertion site F - usually ~4cm above insertion
  6. It contains foci of increased T2 signal in all cases of chronic tendinopathy F - chronic tendinosis manifests as thickening of tendon without evidence of increased intrasubstance signal intensity; the latter (and clinical history) help differentiate from partial tear.
169
Q

168.Regarding shoulder imaging, which is true:

  1. Rotator Cuff tears are most commonly associated with Type 1 acromion
  2. Rotator Cuff tears Most often in posterior third of supraspinatus tendon
  3. Rotator Cuff tears are typically echogenic on ultrasound
  4. Absence of fluid in the subacromial subdeltoid bursa excludes Rotator Cuff tears
  5. Supraspinatus calcification best seen on films in external rotation
A

ANSWER:5.Supraspinatus calcification best seen on films in external rotation T - CME 03.78

  1. Regarding shoulder imaging, which is true: (GC)
  2. They are most commonly associated with Type 1 acromion F - Type III has a significantly higher incidence of rotator cuff tears (see Qu 1, Aug 06).
  3. Most often in posterior third of supraspinatus tendon F - “critical zone” is 1cm medial to tendon insertion (area of relative hypovascularity), tears usually start in anterior portion and propagate posteriorly. *LW agrees with this by according to Helms, so would assume this correct for exams. More recent literature shown for chronic tears starts posteriorly.
  4. They are typically echogenic on ultrasound F - typically hypoechoic area, represents thickened bursa filling the defect (Dahnert)
  5. Absence of fluid in the subacromial subdeltoid bursa excludes the diagnosis F - most sensitive sign (Dahnert) but may not be seen in articular-surface partial thickness tears.
  6. Supraspinatus calcification best seen on films in external rotation T - CME 03.78
170
Q

169.MRI findings associated with anterior cruciate ligament tears include the following except:

  1. Bone contusion of the posterolateral tibial plateau
  2. Meniscocapsular separation of the posterior horn of the medial meniscus
  3. Medial collateral ligament injury
  4. Fracture of the fibular tubercle
A
  • LW: All answers appear true.
    1. Bone contusion of the posterolateral tibial plateau: true, classic for pivot shift mechanism.
    2. Meniscocapsular separation of the posterior horn of the medial meniscus: True. Radiopedia states an uncommon injury.
    3. Medial collateral ligament injury: true, O’Donoghue unhappy triad: ACL, MCL, medial meniscus.
  1. Fracture of the fibular tubercle
    * LW: unsure what the fibular tubercle is:
    - If referring to styloid process, this can be avulsed in cruciate ligament injuries (commonly PCL but also ACL injuries) and is referred to the arcuate sign, representing PLC injury. Thus, would favour this to be possibly true
  • If referring to small anterior and posterior tubercles at distal tib-fib joint forming anterior and posterior syndesmotic boundaries, then this is obviously incorrect.
171
Q

170.Pigmented villonodular synovitis, which is true:

  1. Ankle joint is most commonly involved
  2. Recurrence in up to 20% following synovectomy
  3. Scattered calcifications occur
  4. Hypertrophic bone formation is a feature
  5. May see areas of high signal intensity on MRI
A

ANSWER:5.May see areas of high signal intensity on MRI T - predominantly low signal on all sequences (due to presence of iron), but often heterogenous/high SI on T2 due to fat, effusion, oedema, inflammation.Ref. Dahnert

  1. Pigmented villonodular synovitis, which is true: (GC)
  2. Ankle joint is most commonly involved F - knee involved in 80%
  3. Recurrence in up to 20% following synovectomy F - 50% recurrence rate
  4. Scattered calcifications occur F – no calcification
  5. Hypertrophic bone formation is a feature F
  6. May see areas of high signal intensity on MRI T - predominantly low signal on all sequences (due to presence of iron), but often heterogenous/high SI on T2 due to fat, effusion, oedema, inflammation.Ref. Dahnert
172
Q

172.Spinal cord injury without radiographic abnormality, which is true:

  1. Occurs in 2 - 5% of spinal inuries
  2. Is commoner in older children
  3. Can have a delayed onset, usually within up to 48hrs
  4. Is thought to be due to lack of elasticity of ligaments resulting in cord distraction and compression
A

ANSWER:3.Can have a delayed onset, usually within up to 48hrs T - delayed onset of neurological damage is reported to occur in 6-54% of cases, usually within 48h

  1. Spinal cord injury without radiographic abnormality, which is true: (GC)
  2. Occurs in 2 - 5% of spinal inuries F - overall incidence of SCIWORA varies from 5-65%
  3. Is more common in older children F - SCIWORA is commoner in younger children and in this group is often severe with a poor prognosis for recovery
  4. Can have a delayed onset, usually within up to 48hrs T - delayed onset of neurological damage is reported to occur in 6-54% of cases, usually within 48h
  5. Is thought to be due to lack of elasticity of ligaments resulting in cord distraction and compression F - is postulated that excessively elastic ligaments and other developmental features of the immature spine allow transient excessive movement during trauma, resulting in cord distraction or compression. (Pediatr Radiol 2001)
173
Q

173.Regarding paediatric bones, which is true:

  1. Neonatal rubella infection produces vertically orientated radiolucent metaphyseal bands
  2. Abnormal collagen formation in scurvy results in metaphyseal irregularity and lucency
  3. A ground glass pattern of osteoporosis and metaphyseal changes are characteristic of rickets
  4. Symmetric periosteal reaction is pathognomonic for NAI
  5. Involucrum refers to the portion of necrotic cortical bone seen in osteomyelitis
A

ANSWER:1.Neonatal rubella infection produces vertically orientated radiolucent metaphyseal bands T - “celery stalk sign” seen in 50% of cases; distal femur > proximal tibia

  1. Regarding paediatric bones, which is true: (GC)
  2. Neonatal rubella infection produces vertically orientated radiolucent metaphyseal bands T - “celery stalk sign” seen in 50% of cases; distal femur > proximal tibia
  3. Scurvy results in metaphyseal lucency at the zone of provisional calcification due to abnormal collagen formation F - pathogenesis is correct (failure in formation of stable collagen with subsequent defective formation of osteoid matrix), but the metaphyses are dense (white line of Frankel). Trummerfeld’s zone is a lucent area seen proximal to Frankel’s line (site of subepiphyseal infraction).
  4. A ground glass pattern of osteoporosis and metaphyseal changes are characteristic of rickets F - rickets has coarse trabeculation (ground glass pattern is seen in scurvy). Metaphyses show cupping and fraying in rickets; Frankel’s white line in scurvy.
  5. Symmetric periosteal reaction is pathognomonic for NAI F - may be seen in child abuse but also in a number of other conditions: Dahnert uses PERIOSTEAL SOCKS. Physiologic, scurvy, rickets, Caffey’s (infantile cortical hyperostosis), congenital syphilis, PgE therapy, neoplasm (neuroblastoma, acute leukaemia), juvenile RA (Chapman).
  6. Involucrum refers to the portion of necrotic cortical bone seen in osteomyelitis F - involucrum is the shell of periosteum around the infected bone (develops after 20 days); sequestrum is the detached necrotic cortical bone surrounded by granulation tissue (develops after 30 days).
174
Q

174.The following causes inferior rib notching:

  1. Blalock Taussig shunt
  2. Marfan syndrome
  3. Rheumatoid arthritis
  4. Scleroderma
  5. SLE
A

ANSWER: 1 .1.Blalock Taussig shunt T - systemic-to-pulmonary artery shunt used for palliation or prior to corrective surgery in cyanotic cardiac malformations (eg. tetralogy of Fallot, tricuspid atresia, pulmonary atresia, hypoplastic left heart syndrome). See unilateral rib notching of the upper 3 or 4 ribs on the operation side. On the side of the shunt there is no distal SCA flow - so no internal mammary flow, and get resultant retrograde flow from the aorta.

108.The following causes inferior rib notching: (GC, TW)

.1.Blalock Taussig shunt T - systemic-to-pulmonary artery shunt used for palliation or prior to corrective surgery in cyanotic cardiac malformations (eg. tetralogy of Fallot, tricuspid atresia, pulmonary atresia, hypoplastic left heart syndrome). See unilateral rib notching of the upper 3 or 4 ribs on the operation side. On the side of the shunt there is no distal SCA flow - so no internal mammary flow, and get resultant retrograde flow from the aorta.

  1. Marfan syndrome superior
  2. Rheumatoid arthritis superior
  3. Scleroderma superior
  4. SLE superior

Causes of superior rib notching:
Connective tissue diseases
- RA, SLE, scleroderma, Sjorgren’s Marfan’s, Osteogenesis imperfecta Hyperparathyroidism NF, restrictive lung disease, poliomyelitis, progeria

Causes of inferior rib notching: Enlarged collateral vessels aorta - coarctation (ribs 4-8 bilaterally), aortic thrombosis (bilateral lower ribs) subclavian a. - post Blalock-Taussig shunt (upper 3 or 4 ribs on side of op) pulmonary a. stenosis or absence venous - AVM of intercostals, SVC obstruction 2. Neurogenic tumours - intercostal neuroma, NF (ribbon ribs), poliomyelitis, paraplegia 3. Osseous - hyperparathyroidism, thalassaemia Dahnert pg 19, Chapman pg 54

175
Q
  1. Regarding congenital spinal abnormalities, which is false:
  2. Diastematomyelia is typically associated with a congenital scoliosis
  3. Dorsal dermal sinus most frequently occurs in the lumbosacral
  4. Caudal regression syndrome is associated with maternal diabetes
  5. Neuroenteric cysts most commonly occur posterior to the spinal cord
  6. Myelomeningocele is associated with hydrocephalus in 70-90% of patients
A

ANSWER:4.Neuroenteric cysts most commonly occur posterior to the spinal cord F - incomplete separation of foregut and notochord with persistence of a canal btwn yolk sac and notochord; cyst connected to meninges through midline defect. Located anterior to spinal canal on mesenteric side of gut.

  1. Regarding congenital spinal abnormalities, which is false: (GC)
  2. Diastematomyelia is typically associated with a congenital scoliosis T - sagittal division of cord into two hemicords, each of which contains a central canal, one dorsal horn and one ventral horn; result of adhesions btwn ectodermand endoderm. Congenital scoliosis in 50-75%. 5% of patients with congenital scoliosis have diastematomyelia.
  3. Dorsal dermal sinus most frequently occurs in the lumbosacral region T - epithelium-lined dural tube extending from skin surface to intracanalicular space, frequently communicating with CNS and its coverings; due to a focal point of incomplete separation of cutaneous ectoderm from neural ectoderm during neurulation. Lumbosacral 60%, occipital 25%, thoracic 10%. Half end in a dermoid/epidermoid cyst.
  4. Caudal regression syndrome is associated with maternal diabetes T - midline closure defect of neural tube with a spectrum of abnormalities including complete/partial sacral agenesis, defromities of lumbar vertebra and pelvis. Occurs in up to 1% of pregnancies of women with diabetes; 200-400x risk in women who are insulin-dependent.
  5. Neuroenteric cysts most commonly occur posterior to the spinal cord F - incomplete separation of foregut and notochord with persistence of a canal btwn yolk sac and notochord; cyst connected to meninges through midline defect. Located anterior to spinal canal on mesenteric side of gut.
  6. Myelomeningocele is associated with hydrocephalus in 70-90% of patients T - sac covered by leptomeninges containing CSF and variable amt of neural tissue; herniated through a defect in posterior/anterior elements of spine. Hydrocephalus in 70-90%; 25% of patients with hydrocephalus have spina bifida. Other associations include: Chiari II (99%), vertebral anomalies, chromosomal anomalies, cord tethering. Ref. Dahnert
176
Q

176.Pagets disease characteristics

  1. large multinucleated osteoclasts in the lytic phase
  2. characteristic woven bone
  3. caused by a slow virus - rotavirus
A

1.large multinucleated osteoclasts in the lytic phase

Postulated to be myxo or paramyxovirus.

**LW: although Pagets does form woven bone, the classic descriptor is of mosaic pattern Lamellar bone per Robbins.

177
Q

177.Chondrosarcoma - differentiating from other chondroid tumours

  1. Peripheral skeleton
  2. Intramedullary chondroid matrix surrounding trabeculae
  3. Cellular atypia
  4. Chicken wire appearance
A

ANSWER:
LJS: In chondrosarcoma, cartilage infiltrates BM, causing entrapment and destruction of surrounding pre-existing normal trabecular bone. This surrounding of normal trabeculae is classic histological finding according to Robbins. Cellular atypia and mitotic activity are variable depending on grade (1-3) of chondrosarcoma, so doesn’t help differentiate. Hence I think 2 is correct

  1. Chicken wire in chondroblastoma.
    * LW - chicken wire appearance is classic for chondroblastoma, thus differentiating it from chondrosarcoma, but its absence doesnt confirm chondrosarcoma, hence less correct.
178
Q
  1. Suspect of NAI is unusual in a history of:
  2. SIDS in a sibling of 13 months
  3. Healing fracture of the clavicle at one month
  4. Brachial plexus injury (home birth)
A

lots of controversyProbably 1 . clavicle fracture can be due to birth defect

179
Q
  1. Synovial sarcoma (Most correct statement)
  2. most common in tendon sheaths of fingers/toes
  3. Metastasises to regional LN’s, lung and bones
A

2.Metastasises to regional LN’s, lung and bones

180
Q
  1. Osteoblastoma vs Osteoma
  2. no such thing as an osteoblastoma
  3. spinal involvement
A

2.spinal involvement

181
Q
  1. Regarding SLE, least likely associated with
  2. male > female
  3. Ab to double stranded DNA and Smith Ag
  4. Venous and arterial thrombosis
  5. ??% associated with renal disease
A
  1. female > male 10: 1
182
Q
  1. Liposarcoma. Most correct statement
  2. occurs in retroperitoneum and proximal limbs
  3. 2-8 years of age
  4. 50-80 years of age
A

1.occurs in retroperitoneum and proximal limbs

183
Q
  1. Liposarcoma. Most correct statement
  2. occurs in retroperitoneum and proximal limbs
  3. 2-8 years of age
  4. 50-80 years of age
A

ANSWER:1.occurs in retroperitoneum and proximal limbs (normally retroperitoneum and proximal thigh)

  1. common age 40-60
184
Q
  1. Meniscal cysts
  2. lateral > medial
  3. associated with vertical mensical tears
A

*AJL - tricky question to answer. Seems like they are approximately equal.
RP says:
Surgical and arthroscopic series report lateral meniscal cysts as a more frequent occurrence as compared to their medial counterparts. A systematic review of MR literature says that the frequency of medial and lateral meniscal cysts is almost equal. In ~4% of cases, meniscal cysts may involve both menisci within the same knee

Previous answer:
Both wrong???Stat dx says- Cystic mass in continuity with horizontal meniscal tear- median > lateral

185
Q
  1. Posterolateral complex injury
  2. popliteus
  3. medial meniscus
  4. lateral collateral ligament
A

*LW: posterolateral corner injury involves popliteus, LCL and poplieofibular ligament, with additional structures of: biceps femoris muscle, arcuate ligament, meniscopopliteal fascicles, and fabellofibular ligament.

Thus options 1 and 3 are correct.

If question was worded which is false: as medial meniscus is not part of the anatomical PLC, this would be favoured option

186
Q
  1. Rotator Cuff associations (false):
  2. acromio-humeral distance
  3. Os acromiale
  4. calcium in supraspinatus best seen on external rotation
  5. Hills Sachs best seen on external rotation
A

ANSWER:4. hill sac best on internal rotation not external rotation.

  1. Abnormal acromiohumeral distance < 7mm
  2. Os Acromiale can be a cause of rotator cuff tear.
  3. Supraspinatus calcium best seen on external rotation (xray), internal rotation (ultrasound): true.
  4. hill sac best on internal rotation
187
Q

188.Osteoid osteoma. which is true

  1. Shows no bony sclerosis if intracapsular.
  2. Shows no bony sclerosis if intramedullary.
  3. Causes no pain in a phalangeal lesion.
  4. More commonly affects males.
  5. Is associated with leg length discrepancy.
A

4.More commonly affects males. 3:1

  • *LJS can also cause leg length discrepancy
    e. g. https://radiopaedia.org/cases/osteoid-osteoma-15?lang=gb
188
Q
189.The following are multilocular lucent solitary lesions
.1.Odontogenic keratin cyst.
2.Dentigerous cyst.
3.Plasmacytoma.
4.Cementoma.
5.Ameloblastoma.
A

*LW: 5.Ameloblastoma.

.1.Odontogenic keratin cyst: False - expansile solitary unilocular lesion extending longitudinally in the posterior portions of the mandible.
*Promtheius says can have daughter cysts being multi locular, although can also be uni locular. so although True, I still favor ameloblastoma as most correct.

  1. Dentigerous cyst: False - well-defined and unilocular radiolucency surrounding the crown of an unerupted or impacted tooth within the mandible.
  2. Plasmacytoma: False - Plasmacytoma (as with multiple myeloma) are typically seen as well-defined, “punched-out” lytic lesions with associated extraosseous soft-tissue masses
  3. Cementoma: false - radiodense or mixed-density with a rounded or sunburst appearance, and a relatively radiolucent rim. Loss of the periodontal ligament space along with root resorption/loss of root outline are common
  4. Ameloblastoma: ture - multicystic
189
Q
  1. More than 60% of Ewings rib lesions. which is false
  2. Have a large soft tissue mass.
  3. Occur in males.
  4. Have lytic destruction.
  5. Produce expansion.
  6. Have extensive periosteal reaction.
A

4.Produce expansion.

190
Q

191.The following are indications for an MDP scan

  1. Primary tumour versus osteomyelitis.
  2. Consolidation of a three month old fracture.
  3. Stress fractures.
  4. Loosening of prosthesis two years after insertion.
  5. Patello-femoral arthritis.
A

4 is probably correct answerstat dx:2-3 years after placement, negative NM uptake has very high negative predictive value.

  1. stress fracture indication is not right -> as high sensitivity (75-95%) but low specificity -> positive on all 3 phase-> positivity less conspicuous after 2-4 weeks
191
Q

192.Osteochondritis dissecans.which is wrong

  1. More commonly affects the medial talus.
  2. Affects the lateral femoral condyle in 15-20%.
  3. The asymptomatic knee is affected in 25% of cases.
  4. Non-attached versus attached fragments can be distinguished by a surrounding hypodense rim on T-2 images.
A

4.Non-attached versus attached fragments can be distinguished by a surrounding hypodense rim on T-2 images.

192
Q

193.The following have ossified by eight years. which one is right
Multiple answers

  1. The medial epicondyle of the humerus.
  2. Lateral epicondyle of the humerus.
  3. Acromion.
  4. Calcaneum.
  5. Pisiform.
A
  1. The medial epicondyle of the humerus.

4. Calcaneum.

193
Q

194.PVNS

1.Is a common cause of painless passive reduction in range of movement.
2.May show ill defined margins on plain film.
3.The diffuse form more commonly affects the hips.
4.The focal form more commonly affects the hand and wrist
5.Can be distinguished from rheumatoid arthritis on heavily T-2 weighted images
Multiple answers

A
  • LW:
    4. The focal form more commonly affects the hand and wrist: True as local / focal form refers to tenosynovial giant cell tumor, which occurs most commonly in hands / wrist.

5.Can be distinguished from rheumatoid arthritis on heavily T-2 weighted images: likely true.

PVNS:
1.Is a common cause of painless passive reduction in range of movement: False - painful.

  1. May show ill defined margins on plain film: False. XR appearances are relatively nonspecific with appearances mainly being those of a joint effusion. Bone density and joint space are preserved until the late stages. No calcification is seen. Extrinsic marginal pressure erosions may be present, but it is not possible to distinguish PVNS from synovial chondromatosis
  2. The diffuse form more commonly affects the hips: knee most common.
  3. The focal form more commonly affects the hand and wrist: True, as above.
  4. Can be distinguished from rheumatoid arthritis on heavily T-2 weighted images: likely true.

On MR images, PVNS masslike proliferative synovium has a lobulated margin, and it may be extensive in diffuse PVNS or limited to a single nodule in the focal form. The lesions tend to bleed, causing hemosiderin deposition and a characteristic low signal intensity with all pulse sequences (,12). Areas of high signal intensity on T2-weighted images may be present and are likely caused by inflamed synovium or joint effusions

Rheumatoid MR imaging demonstrates intermediate- to low-signal-intensity pannus on T1- and T2-weighted images in the affected joints

194
Q

129.ABC’S
Multiple answers

  1. Long bones are affected more commonly than vertebral bodies.
  2. Have a well defined margin.
  3. Show a fluid level on C.T. & M.R..I.
  4. Can extend from one vertebral body to the contiguous vertebral body.
  5. Are predominantly diaphyseal in long bones.
A
  1. Long bones are affected more commonly than vertebral bodies.
  2. Have a well defined margin.
  3. Show a fluid level on C.T. & M.R..I.
195
Q

196.Fibrous dysplasia
Multiple answers

.1.May contain regions of calcification.

  1. Has a higher incidence of malignancy than Paget’s disease.
  2. Shows increased tracer distribution on flow and delayed scans.
  3. A rind of sclerosis is more commonly seen than with ABC’S.
  4. Stops growing after epiphyseal fusion.
A
  1. Shows increased tracer distribution on flow and delayed scans.
  2. Stops growing after epiphyseal fusion.
196
Q
  1. Pseudoarthrosis occurs in
  2. Osteogenesis imperfecta
  3. Neurofibromatosis
  4. Ehlers Danlos syndrome
  5. Marfan’s syndrome
  6. Fibrous dysplasia
A

2.Neurofibromatosis

197
Q

132.With ultrasound of the venous system of the lower limbs

  1. The walls are usually well visualised
  2. Vein compressibility is best tested with the image plane transverse to the vein axis
  3. The diameter of the larger veins decreases with inspiration
  4. Recent thrombus is rarely anechoeic
  5. Venous reflux is poorly assessed by colour doppler
A
  1. Vein compressibility is best tested with the image plane transverse to the vein axis
  2. Recent thrombus is rarely anechoeic

***LJS - reduced vein diameter on inspiration also true?
Increased intrathoracic volume = increased venous return

**LW: i tried looking vein diameter up, got into some deep physiology papers, that generally said vein diameter was NOT reduced on inspiration, depsite above first principles, as intra abdominal pressure goes up, which in turn reduces venous return from limb into abdomen, hence vein not reducing.

198
Q

133.Aneurysmal bone cysts , which is true

  1. May regress spontaneously
  2. Are typically central in the metaphysis
  3. Are more vascular than surrounding tissue on angiography
  4. Usually present in the third decade
  5. Are more common in males
A
  1. Are more vascular than surrounding tissue on angiography - lots of feeding vessels
  2. Spontaneous regression may occur, including following partial removal, but this is not the typical natural history
  3. Normally eccentric
  4. more vascular - has extensive feeding vessels
  5. Aneurysmal bone cysts are primarily seen in children and adolescents, with 80% occurring in the patients less than 20 years of age 8.5. M = F
199
Q
  1. Acro-osteolysis may be seen in
  2. Syringomyelia
  3. Pitriasis versicolor
  4. Burns
  5. Psoriasis
  6. Neurofibromatosis
A
  1. Burns

4. Psoriasis

200
Q

135.MRI of the knee
Multiple answers

  1. Normal menisci apear as triangular regions of increased signal on T1 weighted sagittal scans
  2. Synoviai fluid is of increased signal of T 2 weighted images
  3. Anatomic detail is best depicted on T1 weighted images
  4. Cortical bone is of increased signal on Tl and T2 images
  5. Meniscal tears in children usually occur in the medial meniscus
A
  • LW: Correct answers:
    2. Synoviai fluid is of increased signal of T 2 weighted images
    3. Anatomic detail is best depicted on T1 weighted images

5.Meniscal tears in children usually occur in the medial meniscus: False - more commonly laterally, as does discoid menisci which are prone to tear. (https://posna.org/Physician-Education/Study-Guide/Meniscus-Tears)